■ちょっとした疑問はここに書いてね26■

このエントリーをはてなブックマークに追加
1ご冗談でしょう?名無しさん
前スレ
■ちょっとした疑問はここに書いてね25■
http://science.2ch.net/test/read.cgi/sci/1064830533/

【質問する前に】
教科書や参考書をよく読もう。
http://www.yahoo.co.jp/ とか http://www.google.com/ を利用し自分で調べること。
宿題を聞くときは、どこまでやってみてどこが分からないのかを書くこと。
丸投げはダメだからね。
(レスする人も丸投げに答えるのは自粛してねっていうかや・め・ろ。
 ていうか丸投げ君を諭そうとするのも自粛きぼんぬ。無駄だから)

質問に対する返答には、何かしらの返答ちょうだいね。

★過去スレ、関連スレはここ>>2よ。
★それから、書き込む前に>>3の注意事項を読んでね。
★数式の書き方(参考)はこちら>>4
(予備リンク:>>2-10
荒らし厳禁、煽りは黙殺、忘れないうちに定期age。
単発質問スレを発見したらこのスレへの誘導をよろしくね。>ALL
2ご冗談でしょう?名無しさん:03/11/05 17:28 ID:???
過去スレ
Part1:http://cheese.2ch.net/sci/kako/968/968116983.html
Part2:http://cheese.2ch.net/sci/kako/981/981661589.html
Part3:http://cheese.2ch.net/sci/kako/991/991491349.html
Part4:http://cheese.2ch.net/sci/kako/997/997037229.html
Part5:http://cheese.2ch.net/sci/kako/1003/10030/1003082106.html
Part6:http://cheese.2ch.net/sci/kako/1006/10064/1006423654.html
Part7:http://cheese.2ch.net/sci/kako/1011/10111/1011199123.html
Part8:http://cheese.2ch.net/sci/kako/1014/10149/1014955593.html
Part9:http://science.2ch.net/test/read.cgi/sci/1019557050/
Part10:http://science.2ch.net/test/read.cgi/sci/1024330084/
Part11:http://science.2ch.net/test/read.cgi/sci/1025840212/
Part12:http://science.2ch.net/test/read.cgi/sci/1027694658/
Part13:http://science.2ch.net/test/read.cgi/sci/1029946423/
Part14:http://science.2ch.net/test/read.cgi/sci/1032629984/
Part15:http://science.2ch.net/test/read.cgi/sci/1035913464/
Part16:http://science.2ch.net/test/read.cgi/sci/1038224562/
Part17:http://science.2ch.net/test/read.cgi/sci/1041622629/
Part18:http://science.2ch.net/test/read.cgi/sci/1043668297/
Part19:http://science.2ch.net/test/read.cgi/sci/1045448430/
Part20:http://science.2ch.net/test/read.cgi/sci/1048741188/
Part21:http://science.2ch.net/test/read.cgi/sci/1052820825/
Part22:http://science.2ch.net/test/read.cgi/sci/1055317668/
Part23:http://science.2ch.net/test/read.cgi/sci/1057733284/
Part24:http://science.2ch.net/test/read.cgi/sci/1060960040/
Part25:http://science.2ch.net/test/read.cgi/sci/1064830533/
3ご冗談でしょう?名無しさん:03/11/05 17:29 ID:???
書き込む際の注意

1.)
板の性格上、UNIX端末からの閲覧も多いと推察されます。
機種依存文字(ローマ数字、丸数字等)は避けて下さい。

2.)
以下のような質問に物理板住人は飽き飽きしているので、たぶん無視されます。
しないで下さい。
「相対性理論は間違っています」「量子力学は間違っています」
「宇宙論は間違っています」「シュレディンガーの猫は変です」
「永久機関を作りました」「タイムマシンについて教えて」
「どうして〜?」関連(→「どのようにして〜?」と質問すること)
「なぜ〜?」関連(物理学の対象ではないため)

「どうして・なぜ」:http://academy.2ch.net/philo/
(哲学板・雑談板のほうがむいている場合が多いです。)
雑談は雑談スレで:http://science.2ch.net/test/read.cgi/sci/973536997/

3.)
宿題を聞くときは、どこまでやってみてどこが分からないのかを書くこと。
丸投げはダメよ。丸投げに答えるのもダメよ。
せめてポインタを示す程度に留めましょう。

4.)
厨房を放置できない人も厨房です。
4ご冗談でしょう?名無しさん:03/11/05 17:29 ID:???
掲示板での数学記号の書き方例(数学板より拝借)】
●スカラー:a,b,c,...,z, A,B,C,...,Z, α,β,γ,...,ω, Α,Β,Γ,...,Ω, ...
●ベクトル:x=[x[1],x[2],...], |x>, x↑, vector(x) (← 混同しない場合はスカラーと同じ記号でいい.通常は(成分を横で書いても)縦ベクトルとして扱う.)
●行列(1成分表示):A[i,j], I[i,j]=δ_(ij)
●行列(全成分表示):A=[[A[1,1],A[2,1],...],[A[1,2],A[2,2],...],...]=[a1,a2,a3,...], I=[[1,0,0,...],[0,1,0,...],[0,0,1,...],...] (←ここでは列ごとに表示(縦ベクトルを横に並べる).行ごとに表示しても構わないが,統一して使わないと混同するので注意.)
●関数:f(x), f[x]
●数列:a(n), a[n], a_n
●足し算:a+b
●引き算:a-b
●掛け算:a*b, ab (← 通常"*"を使い,"x"は使わない.)
●割り算・分数:a/b, a/(b+c), a/(bc) (← 通常"/"を使い,"÷"は使わない.)
●複号:a±b=a士b, a干b (← "±"は「きごう」で変換可.他に漢字の"士""干"なども利用できる.)
●平方根:√(a+b)=(a+b)^(1/2) (← "√"は「るーと」で変換可.)
●指数・指数関数:a^b, x^(n+1), exp(x+y)=e^(x+y) (← "^"を使う."exp"はeの指数.)
●対数・対数関数:log_{a}(b), log(x/2)=log_{10}(x/2), ln(x/2)=log_{e}(x/2) (← 底を省略する場合,"log"は常用対数,"ln"は自然対数.)
5ご冗談でしょう?名無しさん:03/11/05 17:30 ID:???
●三角比・三角関数:sin(a), cos(x+y), tan(x/2)
●内積・外積・スカラー3重積:a・b=(a,b), axb=[a,b], a・(bxc)=(axb)・c=[a,b,c]=det([a,b,c])
●行列式・トレース:|A|=det(A), tr(A)
●絶対値:|x|
●ガウス記号:[x] (← 関数の変数表示などと混同しないように注意.)
●共役複素数:z~
●階乗:n!=n*(n-1)*(n-2)*...*2*1, n!!=n*(n-2)*(n-4)*...
●順列・組合せ:P[n,k]=nPk, C[n.k]=nCk, Π[n,k]=nΠk, H[n,k]=nHk (← "Π"は「ぱい」で変換可.)
●微分・偏微分:y', dy/dx, ∂y/∂x (← "∂"は「きごう」で変換可.)
●ベクトル微分:∇f=grad(f), ∇・A=div(A),∇xA=rot(A), (∇^2)f=Δf (← "∇"は「きごう」,"Δ"は「でるた」で
変換可.)
●積分:∫[0,1]f(x)dx=F(x)|_[x=0,1], ∫[y=0,x]f(x,y)dy, ∬_[D]f(x,y)dxdy (← "∫"は「いんてぐらる」,"∬"は「きご
う」で変換可.)
●数列和・数列積:Σ[k=1,n]a(k), Π[k=1,n]a(k) (← "Σ"は「しぐま」,"Π"は「ぱい」で変換可.)
●極限:lim[x→∞]f(x) (← "∞"は「むげんだい」で変換可.)
●図形:"△"は「さんかく」,"∠"は「かく」,"⊥"は「すいちょく」で変換可.

●その他
・関数等の変数表示や式の括弧は,括弧()だけでなく[]{}を適当に組み合わせると見やすい場合がある.
・ギリシャ文字はその読み方で変換可.
・上記のほとんどの数学記号や上記以外の数学記号"⇒∀≠≧≒∈±≡∩∽"などは「きごう」で順次変換できる.
61:03/11/05 17:31 ID:???
関連スレは消えてたので落としました

7ご冗談でしょう?名無しさん:03/11/05 17:31 ID:???
>>1
乙葉
8ken:03/11/05 23:39 ID:7WHeMoIO
突然すみません。
二輪のレースをやってるものですが、キャブレターセッティングを出すのに空気中の酸素濃度が大切で、
気温(℃)、湿度(%)、気圧(hpa)から空気中の酸素濃度がわかるソフトが欲しいのですが、どこかにないですか?
知っている方教えて下さい。
9ご冗談でしょう?名無しさん:03/11/06 00:32 ID:???
酸素濃度は気温、湿度、気圧だけからは求めることができんだろ
10ご冗談でしょう?名無しさん:03/11/06 01:25 ID:bWrA34RM
いわゆる21%じゃだめなのか?
有効数字何桁までひつようなんだ?
11前スレの978:03/11/06 01:27 ID:32/6bnhT
一様磁場中の荷電粒子の運動
基本解 mv↑'=q(v↑×B↑)
初期条件 t=0 の時、r↑(0)=(x_0,y_0,z_0),v↑(0)=(v_0x,v_0y,v_0z)
また、B↑=(B,0,0) とする。


という問題で、>>908さんがこからこうアドバイスしてもらったのですが


成分ごとに計算しなさい。
dmv_z/dt=-qv_yB
dmv_y/dt=qv_zB
から
d^2v_z/dt^2=-(qB/m)^2 v_z
解くと
v_z=exp(iqBt/m+iθ) iは虚数、θは初期位相v_yも同じ形で初期位相が異なる
これで周期qB/mの円運動が表されてる


私は
v_z=exp(iqBt/m+C_1)+exp(-iqBt/m+C_2)
となると思うのですが、
v_z=exp(iqBt/m+iθ)
というように式変形する過程がわかりません。
どなたかお教えいただけないでしょうか?
12ご冗談でしょう?名無しさん:03/11/06 01:28 ID:???
酸素濃度って日によってどれくらい変わるの?
13ご冗談でしょう?名無しさん:03/11/06 01:33 ID:???
高度、つまり気圧によって空気の量が減るために酸素の量も減るってことはあっても
酸素濃度、つまり空気中に含まれる酸素の割合は中間圏以下ほとんど変わりません。
14フシアナしろよ:03/11/06 02:29 ID:???
      /VVVVVVVVVVVVVVVVVVVVVVVVVVVVVVN\
( ・∀・)∩ ウンコビ━━━━━━━━━━━━━━━━━ム  >εε=ヽ( `Д´)ノ ウワァァァァン      
    ⊃  VVVVVVVVVVVVVVVVVVVVVVVVVVVVVVVN/

                               人人人人人
( ・∀・) ニヤニヤ                      ≪ヽ( `Д´)ノ ≫ ビリビリ
                               ^Y^Y^Y^Y^Y 

ヽ( ・∀・)ノ ウンコー                      ヽ(・∀・ )ノ ウンコー
15ご冗談でしょう?名無しさん:03/11/06 02:40 ID:???
酸素の体積濃度のことを言ってるんだったら、
(酸素濃度) = (θ ℃、P hPaにおける酸素濃度) × ((圧力) − (水の蒸気圧) ×(湿度)÷100)/P × (θ + 273)/(気温 + 273)
温度θ ℃、圧力P hPa(ヘクトパスカル)は、水の蒸気圧、酸素濃度を理科年表等の文献で調べて、その文献値に適合しているものを使えばよい。
16ご冗談でしょう?名無しさん:03/11/06 02:42 ID:???
>>8へのレスでした。
現在の気温圧力湿度は(気温)(圧力)(湿度)に代入すること。
17フシアナしろよ:03/11/06 02:57 ID:???
      /VVVVVVVVVVVVVVVVVVVVVVVVVVVVVVN\
( ・∀・)∩ ウンコビ━━━━━━━━━━━━━━━━━ム  >εε=ヽ( `Д´)ノ ウワァァァァン      
    ⊃  VVVVVVVVVVVVVVVVVVVVVVVVVVVVVVVN/

                               人人人人人
( ・∀・) ニヤニヤ                      ≪ヽ( `Д´)ノ ≫ ビリビリ
                               ^Y^Y^Y^Y^Y 

ヽ( ・∀・)ノ ウンコー                      ヽ(・∀・ )ノ ウンコー
18ご冗談でしょう?名無しさん:03/11/06 07:09 ID:???
>>12
酸素濃度は光合成の影響で、
昼増・夜減・夏増・冬減になると思います。
理科年表に書いてあったような気がしてたんだけど、
どうも見当たりません。
日本国勢図絵のどこかに書いてないでしょうか?
19ご冗談でしょう?名無しさん:03/11/06 07:30 ID:???
ってか、前スレのいくつかの質問、スルーか(w
20ご冗談でしょう?名無しさん:03/11/06 07:47 ID:xHUyae+O
ウラン鉱石の採掘できる鉱山はなぜ爆発しないの?

すごいヒントだと思うのだが。
21ご冗談でしょう?名無しさん:03/11/06 08:00 ID:???
ケプラー第3法則って、なんでグラフは直線になるって説明するんでしょうね。
どう考えても曲線になりそうですよね。
どうしてか教えてください。
22ご冗談でしょう?名無しさん:03/11/06 08:09 ID:xHUyae+O
なめて角を滑らかにすると曲線。
23ご冗談でしょう?名無しさん:03/11/06 08:29 ID:xHUyae+O
世の中を「自然」に見ることができる人は通り過ぎるだけ。
24ご冗談でしょう?名無しさん:03/11/06 08:49 ID:xHUyae+O
「もしかして自分が一番乗り?」そういう考えが頭にある人しか論文を書かない。

「うちの歴代の先祖はみんな通っているはず。」
25ご冗談でしょう?名無しさん:03/11/06 10:19 ID:xHUyae+O
新しい考え、確信的な技術は世の中全体の意識が追いつくまでは、どれだけあがいても出ない。

存在に気づいた人間すべてが結界になり、気づいた人間自身も封印される。

教えなくても自然にできる人が一定数を越えたとき自然に出ていく。
26k2:03/11/06 10:25 ID:UZLrh3on
お絵描きチャット  おまいらの憩いの場。

思う存分、暴れて来い。

http://www.takamin.com/oekakichat/user/oekakichat3.php?userid=9729

                          以上
27ご冗談でしょう?名無しさん:03/11/06 10:31 ID:xHUyae+O
みんな、まだお子様用初心者実験キットでやってるんじゃないの?
28ご冗談でしょう?名無しさん:03/11/06 11:48 ID:???
一言ノートのスレはここですか?
29フシアナしろよ:03/11/06 12:06 ID:???
      /VVVVVVVVVVVVVVVVVVVVVVVVVVVVVVN\
( ・∀・)∩ ウンコビ━━━━━━━━━━━━━━━━━ム  >εε=ヽ( `Д´)ノ ウワァァァァン      
    ⊃  VVVVVVVVVVVVVVVVVVVVVVVVVVVVVVVN/

                               人人人人人
( ・∀・) ニヤニヤ                      ≪ヽ( `Д´)ノ ≫ ビリビリ
                               ^Y^Y^Y^Y^Y 

ヽ( ・∀・)ノ ウンコー                      ヽ(・∀・ )ノ ウンコー
30フシアナしろよ:03/11/06 12:47 ID:???
      /VVVVVVVVVVVVVVVVVVVVVVVVVVVVVVN\
( ・∀・)∩ ウンコビ━━━━━━━━━━━━━━━━━ム  >εε=ヽ( `Д´)ノ ウワァァァァン      
    ⊃  VVVVVVVVVVVVVVVVVVVVVVVVVVVVVVVN/

                               人人人人人
( ・∀・) ニヤニヤ                      ≪ヽ( `Д´)ノ ≫ ビリビリ
                               ^Y^Y^Y^Y^Y 

ヽ( ・∀・)ノ ウンコー                      ヽ(・∀・ )ノ ウンコー
31ご冗談でしょう?名無しさん:03/11/06 16:45 ID:hWxvR1X9
量子力学で有名な物理学者ってハイゼンベルグと誰でしたっけ?
32ご冗談でしょう?名無しさん:03/11/06 16:46 ID:???
ボーアでした・・・
33ご冗談でしょう?名無しさん:03/11/06 17:35 ID:???
シュレーディンガーやディラックの立場は?
34ご冗談でしょう?名無しさん:03/11/06 18:53 ID:???
ちょっと質問です。
A↑・(B↑×C↑)は
どうしてA↑、B↑、C↑を三稜とした平行六面体の体積に
なるんでしょうか。幾何学的に理解することは可能でしょうか。
35ご冗談でしょう?名無しさん:03/11/06 19:07 ID:???
>>34
BとCのなす角をθ、AとB×Cのなす角をφとすると
BとCが作る平行四辺形の面積の大きさは|B||C|sinθ
B×C⊥B、Cより、B×Cは平行六面体の高さに平行だから、その高さは|A|cosφ。
よって平行六面体の体積は|A||B||C|sinθcosφ
他方
A・B×C=|A||B×C|cosφ=|A||B||C|sinθcosφ
36ご冗談でしょう?名無しさん:03/11/06 19:15 ID:???
>>35
ありがとうございました!
37ご冗談でしょう?名無しさん:03/11/06 19:34 ID:WRojrbrQ
全く同じ条件でコップを地面に落としすと、
割れ方は同じになるのでしょうか?
38ご冗談でしょう?名無しさん:03/11/06 20:13 ID:???
>>37
ガラスの破壊機構を調べな。
39ご冗談でしょう?名無しさん:03/11/07 07:40 ID:???
すみません、相対論の問題になると思うんですが、次の問題はどうなるでしょうか?

ものすごく広い宇宙船があって、それが1G加速を続けてると仮定する。
進行方向に垂直な面を通るような光は、宇宙船の中の人からはどう見えるか?

私としては系の中からは直進するように見えると思うんですが…
むろん、系の外からは曲がって見えるんでしょうが。
40ご冗談でしょう?名無しさん:03/11/07 08:10 ID:???
>>39
宇宙船の中(加速系)からは光が曲がっているように見え、
宇宙船の外の系からは直進しているように見える。って
一般相対論の基本前提「等価原理」の話なんだが・・・・
41ご冗談でしょう?名無しさん:03/11/07 11:54 ID:cfMZsaRt
大学の電磁気学の教科書で始めから磁荷というものは存在しない、と
書かれてあるものと磁荷の存在を仮定して説明をしていくもの(途中で
電流が本質だとは書かれてあるのですが)があるのですが、磁荷の存在
を仮定して便利な事はあるのでしょうか?
42ご冗談でしょう?名無しさん:03/11/07 12:17 ID:???
>>41
特にメリットはないんじゃない?
43ご冗談でしょう?名無しさん:03/11/07 12:20 ID:???
>>41
電場と磁場の対称性がよくなる
量子論的考察をすると、もし磁荷が存在すれば電荷の量子化が自然に出てくる
大統一理論では磁荷の存在は必然
44ご冗談でしょう?名無しさん:03/11/07 12:24 ID:z82GFxqO
ナゼリサージュ図形の位相角はSin0=B/A
であたえられるんですか?
45ご冗談でしょう?名無しさん:03/11/07 13:13 ID:???
他でも書き込んでしまったんですが、
つまらない質問です。
「時空は曲がってる」の反対って、「時空は平らである」というのでしょうか、
それとも、「時空は均一である」というのでしょうか?
「曲がっていない」というの以外でどういったらいいのでしょうか?
親切な方、お願いします。
46ご冗談でしょう?名無しさん:03/11/07 13:14 ID:???
>>45
「平ら」とか「平坦」とかいいますね。
「均一」では均一に曲がってるのも含まれそう
4741:03/11/07 13:26 ID:cfMZsaRt
なるほど、ありがとうございます。
48ご冗談でしょう?名無しさん:03/11/07 20:00 ID:???
>>40
あぅ。逆でしたか。ありがとうございました。
49サードマン:03/11/08 17:58 ID:IWrm8uBG
水を垂直に吸い上げようとしても10メートル付近で水の重さと真空の圧力がつり合って、これ以上吸い上げれない。じゃあ高さ10メートル以上の大木はどうやって先まで吸い上げてるんですか?誰か教えてください。
50ご冗談でしょう?名無しさん:03/11/08 18:00 ID:oesEfyyi
もーさいかん
51ご冗談でしょう?名無しさん:03/11/08 18:08 ID:???
>>49
ぐぐったら出てきた。
「樹木 圧力 吸い上げ」で、一番上のやつ。
まとめると
20-30mまで:能動輸送&毛細管現象
どの高さでも:蒸散による気孔からの吸い上げ
52ご冗談でしょう?名無しさん:03/11/08 18:21 ID:???
質問があります。
------------------
たとえば、N=6個のローターがあったとして、

   ○
●    ●

●    ●
   ○

●サンプルあり
○サンプルなし
-----------------
生物系のスレで、遠心機を回す際、上図の状態ではバランスが取れていない、との記載があったのですが、理由がわかりません。
教えて下さい!!
53ご冗談でしょう?名無しさん:03/11/08 19:14 ID:pNh6u5Zv
下図で、点電荷Bが固定されていても、固定されていなくても、
点Aに作用する力の大きさはFだよね。なんで?

だって、固定されてたら2Fとかになりそうじゃない?

http://www.geocities.co.jp/SiliconValley-PaloAlto/9562/densi.gif
5453:03/11/08 19:18 ID:pNh6u5Zv
55ご冗談でしょう?名無しさん:03/11/08 20:06 ID:???
>>53
自由落下するあなたにかかる重力と、地面に立っているあなたにかかる重力は変わる?
56ご冗談でしょう?名無しさん:03/11/08 20:58 ID:???
混み合っている電車の中で二人の人が言い合いをしています。
A「押すなよ」
B「押してねぇよ。反対側の奴が押してるんだよ。」
A「そんな言い訳しても、お前が押してるのは変わらないだろ」
B「なんだよ」
・・・・

しかし、BがAを押しているように見えても、二人が「動かない」状況では
実はAも同じ力でBを押し返しているのです。
もし、AがBを押していなかったら、どんどんAの方に動いていって
アッという間に電車から落ちてしまうでしょう。
57ご冗談でしょう?名無しさん:03/11/08 21:49 ID:???
>>56 比喩ヘタ杉。混乱に拍車をかけてどーする。
5856:03/11/08 22:48 ID:???
そっかなぁ〜。力と運動は「直接」関わりはない、っていう例の
つもりだったんだけど。スマソ。
いずれにせよ、少なくとも文才がないのは認める。理才もないかも。
59ご冗談でしょう?名無しさん:03/11/08 23:50 ID:???
>>58
力と運動は直接関わりあるだろ。
6056:03/11/08 23:56 ID:???
>>59
再びスマソ。「力のウチの一つ(一部)を取り出した物」と「運動」。
うーん、ホントにごめん。もう寝るわ。
61ご冗談でしょう?名無しさん:03/11/09 01:03 ID:???
>>56
結局どこがちょっとした疑問なんでしょうか?
62ご冗談でしょう?名無しさん:03/11/09 01:29 ID:???
内力?
63ご冗談でしょう?名無しさん:03/11/09 09:26 ID:30H1gvVQ
「ニュートラリーノ」ってなんですか?
64ご冗談でしょう?名無しさん:03/11/09 10:34 ID:kXHMM00K
65ご冗談でしょう?名無しさん:03/11/09 10:47 ID:MHFbWcXW
こないだアンサンブル部の友達がやってたんですが、
トロンボーンを吹いてる時に、ベル(朝顔みたいな音の出るところ)に紙コップとかグーにした手を
近づけていくとベルを完全にふさいでしまうまで、音が"連続的"に下がっていくように聞こえます。
これは、高校物理の波動の知識では定常波の端が口元に近づいて短くなって、音は高くなるはずですが、
どういう理由で連続的に下がるとおもいますか?さらに、この現象はフルートとかリコーダーでも起こるみたいです。
ちなみにトロンボーンは開管楽器だそうです。物理版の住民の力を貸してください。

参考(トロンボーン):http://www.yamaha.co.jp/product/wind/brass/tb/tb-xeno.html
こんな奴です。
66ご冗談でしょう?名無しさん:03/11/09 13:04 ID:1lxQ5s2W
>>65
回線が遅い方に迷惑ですので↓だけにしたほうがいいです。
http://www.yamaha.co.jp/product/wind/imgs/pic/tb/pic-ysl882v.gif
67Hydrofluoric Acid:03/11/09 13:34 ID:llnuk6FF
前から思っていたんですけどリチウム5やヘリウム5の半減期がかくしゅひょう
に載ってないのは何でですか?
6865:03/11/09 14:05 ID:???
>>66
気が利かなくてスイマセンでした。
69ご冗談でしょう?名無しさん:03/11/09 14:47 ID:???
>>67
非常に寿命が短いものは崩壊幅Γで載っている
寿命τとはτ=hbar/Γの関係

5Liなら崩壊幅1.5MeVでこれは4.4×10^(-22)secということになる
70ご冗談でしょう?名無しさん:03/11/09 15:04 ID:CJ0TpHnn
電磁場の揺らぎとしてみたとき、熱雑音と黒体輻射の共通点と相違点ってなんですか?
雑音ってモロ揺らぎじゃない?
でもさ、黒体輻射って放射スペクトルそのものじゃない?どこが揺らぎなのかと。
71ご冗談でしょう?名無しさん:03/11/09 16:13 ID:???
ケプラー第3法則って、なんでグラフは直線になるって説明するんでしょうね。
どう考えても曲線になりそうですよね。
どうしてか教えてください。
72ご冗談でしょう?名無しさん:03/11/09 16:36 ID:???
>>71
T^2 = (4π^2/GM) a^3
両辺の対数をとってごらん。そうすればわかります。
73ご冗談でしょう?名無しさん:03/11/09 17:04 ID:???
>>69
エネルギーの次元なのになぜ崩壊「幅」というんですか?
74ご冗談でしょう?名無しさん:03/11/09 17:07 ID:RaoTzYSd
化学式、物理法則にすると曼陀羅図みたいな複雑式になる。

「自然」から削りだすこと自体が間違い。

「自分の人生この程度」を認められない女にはまず無理。
75ご冗談でしょう?名無しさん:03/11/09 17:14 ID:ZTEMEXWg
単位長さあたりの質量がρ、全体の長さがLの鎖がまとめられて机にある。
一端が机から落ち始め、鎖の端が机から長さsだけ落ちたときの
鎖の端の速度、加速度を求めよ。

この問題ですが、ρやLはいつ使えばいいのでしょうか?
私は速度が√(2gs)と出しましたが勘違いをしていると思いますので
ご指摘等よろしくお願いします。
76ご冗談でしょう?名無しさん:03/11/09 18:17 ID:???
>>73
不確定性原理からの直接の帰結(ていうか上の式)で、
粒子の質量エネルギーは、一定の値をとりません。
その幅の事。
77ご冗談でしょう?名無しさん:03/11/09 19:30 ID:???
>>76
散乱断面積が長さの2乗の次元を持つように、崩壊幅は長さの1乗の次元を
持つのではないんですか?

それと、エネルギーに不確定性があるのはいいとしても「質量」は一定でしょう。
78ご冗談でしょう?名無しさん:03/11/09 20:38 ID:???
>>77
>>69の定義を見て、もう一度考え直しましょう。
ついでに、一定とやらの「質量」の定義も教えて下さい
79ご冗談でしょう?名無しさん:03/11/09 20:38 ID:???
>>77
長さの幅ではなくエネルギーの幅だからエネルギーの次元

厳密には質量が確固とした値になるのは安定なものだけ
不安定なものは有限な幅を持つ
8077:03/11/09 22:51 ID:???
>>79
>長さの幅ではなくエネルギーの幅だからエネルギーの次元
わかりました。
>厳密には質量が確固とした値になるのは安定なものだけ
そうなんですか。不安定粒子についてはまだ勉強不足なのですが
すると例えば中性子の質量939.5656MeVというのも幅をもっているのですか?
81ご冗談でしょう?名無しさん:03/11/10 00:10 ID:???
>>80
>例えば中性子の質量939.5656MeVというのも幅をもっているのですか?
そうですよ。幅が小さすぎて測定にはかかりませんけどね。
>>69の式から幅を計算してみ?

82某研究者:03/11/10 00:49 ID:/Zm2o1Z2
宇宙膨張が僅かにでも永久に続けば
何れ人間を構成している物質も散逸せざる負えぬと言うなら
人類は絶滅する訳だろうし
自然が人間原理等で人間の存続を永久に望むなら
何れ宇宙膨張は止まり平衡状態と成る可能性も
有るのだろうか
83ご冗談でしょう?名無しさん:03/11/10 04:17 ID:???
何の研究者か知りませんが物理関係では無いようですね
84ご冗談でしょう?名無しさん:03/11/10 06:03 ID:plXpL9mD






【だれかタフなやつ、無真系スレを立てろ】




85ご冗談でしょう?名無しさん:03/11/10 09:52 ID:???
>>82
>自然が人間原理等で人間の存続を永久に望むなら
そんな人間原理はない
86ご冗談でしょう?名無しさん:03/11/10 12:46 ID:???
>>82はよその板から来たアレなので相手にしないであげて下さい
87ご冗談でしょう?名無しさん:03/11/10 22:54 ID:???
>>81
10^-24MeVですから確かに小さすぎて測定できませんね。
どうもありがとうございました。
88某研究者:03/11/11 00:25 ID:GnnQDzmI
>自然が人間原理等で人間の存続を永久に望むなら
>そんな人間原理はない

まあ人間を作る為に物理法則が調整されたのなら
人間が永久に存在出来る様に
物理法則が調整されている可能性も有るだろうか
89ご冗談でしょう?名無しさん:03/11/11 00:36 ID:???
>>88
人間原理を勉強しなおせ

90某研究者:03/11/11 00:43 ID:GnnQDzmI
ホーキングが宇宙が平衡状態と成り人間が永久に存在出来るかも知れぬなら
其れは人間原理が有るからだと言う様な発言をしていた様な
記憶も有るが
どうだっただろうか
91ご冗談でしょう?名無しさん:03/11/11 01:08 ID:???
 倍速で再生しているにもかかわらず
音声が普通に聞こえるビデオデッキ
があったけど(所さんのCMの奴)
あれはどうやってたんだろう?
92ご冗談でしょう?名無しさん:03/11/11 01:25 ID:???
>>91
音素子を一つ置きに再生
93ご冗談でしょう?名無しさん:03/11/11 01:29 ID:???
>>92
 あ、なるほど!
 ありがとう御座いました。
94高2 :03/11/11 01:37 ID:???
縦波の疎、密な部分を決めるにはどうすればいいんでしょうか?
9592:03/11/11 01:38 ID:???
>>93
いや、冗談。
9691:03/11/11 01:48 ID:???
ガーンw

>>94
 決めるじゃなくて出来るだな。
 
97ご冗談でしょう?名無しさん:03/11/11 03:04 ID:nZxjolL+
宇宙に興味があり、図解雑学・宇宙論という本を読みました(ナツメ社刊)
http://www.amazon.co.jp/exec/obidos/ASIN/4816324240/qid=1068487462/sr=1-1/ref=sr_1_2_1/249-3305330-4620301

わかりやすく、面白かったです。
もっと興味がわいてきたので、もう1ステップ上の本を読みたいのですが、
お薦めはありませんか?
物理の知識は、高校で習った基本的なことを覚えてる程度です。
98ご冗談でしょう?名無しさん:03/11/11 06:51 ID:???
S. Weinberg の Gravitation and Cosmology はどう?
9997:03/11/11 10:53 ID:/x8qsjYx
それって洋書ですよね。
できれば和書でお願いします。洋書だと和訳だけで精一杯なので・・・
100ご冗談でしょう?名無しさん:03/11/11 12:26 ID:???
電気って何だよ。しびれるよな。
101ご冗談でしょう?名無しさん:03/11/11 13:16 ID:???
空気、電気、磁気、湿気・・・・


 気の意味がわからなくなった
102学部生です。名無しさん:03/11/11 15:05 ID:???
今、研究室をどこにするかで迷っているのですが、
まだまだ発展性のある研究(研究の余地が十分にある)
分野ってありますか?
もし親切な方がおられましたら、教えてください
お願いしますm(_ _)m
103ご冗談でしょう?名無しさん:03/11/11 15:23 ID:???
>>102
常温核融合
104ご冗談でしょう?名無しさん:03/11/11 15:26 ID:zXu6NaCU
★★★無修正DVDの老舗★★★ ★★★復活★★★
ペントハウス10月号で徹底検証 ★加藤iの★ 全裸盗撮DVD鮮明画像原版入荷
新宿歌舞伎町店頭価格10,000円の品 他商品と同価格 ★★★4枚10,000円★★★
   商品映像確認後の★完全アト払い★ ダマしよう無し
同時入荷  ★朝河 蘭 新作  ★D-MODE20 白石ひより  
      ★援交ハメ撮り 品薄入手困難なNo.3・5・8
      ★女子高生拉致監禁飼育シリーズ 小雪・桃井望 
  
http://go.fc2.com/yamazaki/

     及川奈央 堤さやか 長瀬愛 も依然バカ売れ
 サンプル画像充実 日本一安い、安心、確実。歌舞伎町店頭販売10年の
 実績と信用と★検挙回数★の多さ?  ご覧下さい。 ★★★自信有ります。★★★
            
http://go.fc2.com/yamazaki/

105ご冗談でしょう?名無しさん:03/11/11 16:07 ID:ylqEAOc8
>>102
出来れば大学名を書いた方が助言が増えると思うが。

個人的には発展性の有無で進路を選ぶってのはアレだと思うぞ。
そういう野心があるなら、民間企業とかの方が将来得するかも。

ついでに言うと、研究の余地がない分野ってのは、例えば
(18世紀における)物理学とかか?
106ご冗談でしょう?名無しさん:03/11/11 16:15 ID:T6AOm71a
2つのチャンネルから取り入れた別の波形がお互いに干渉するので、干渉しない
措置を取ったら、今度はノイズが出だしたのだけどどうしたらいいでしょうか?
107ご冗談でしょう?名無しさん:03/11/11 16:37 ID:???
どこからどこに「取り入れた」の?
「干渉しない措置」ってのはどうやったの?

物理の勉強の前に、日本語の勉強をしなさい。
108ご冗談でしょう?名無しさん:03/11/11 17:44 ID:???
>>102
将来性を気にする位だから研究者志望のようだが、
興味の無い分野に行っても身を持ち崩すだけだよ。
好きじゃなければ、この業界やってらんない。
109ご冗談でしょう?名無しさん:03/11/11 17:45 ID:???
>>106
ノイズが出ない措置を取ればよろしいんじゃないかと思います。
110ご冗談でしょう?名無しさん:03/11/11 18:29 ID:j6BjYTv0
Jacob's ladderの放電回路おせーてw
111ご冗談でしょう?名無しさん:03/11/11 21:30 ID:UNh4lDB2
力学的エネルギー保存の法則の実験をしたんですけど
実験の考察で行き詰っています。
自分では高さを2倍にすると飛び出す重りの速さは○倍。
水平にどれぐらい飛んでそれが○倍だった。
それとその理由を書く。あとおもりの質量を2倍にすると
水平に飛ぶ距離はどれぐらいになるかを書こうかなと思ってます。
これだけでは実験の考察としては不向きですか?
112ご冗談でしょう?名無しさん:03/11/11 21:31 ID:???
>>106 マルチ
113ご冗談でしょう?名無しさん:03/11/11 21:39 ID:???
>>111
実験目的は何よ?
114111:03/11/11 21:57 ID:UNh4lDB2
>>113
実験目的は
おもりの高さを2倍にすると飛び出すおもりの水平到達距離は何倍か
を調べるです。
115ご冗談でしょう?名無しさん:03/11/11 22:09 ID:???
理論・予測と実験結果が(誤差の範囲で)一致してるかどうか
一致してなければ、それは何故か

に対する考察が求められてるんじゃないかな。

116111:03/11/11 22:15 ID:???
>>115
でも実験前に結果の予測とか何もしませんでしたけど・・・。
117ご冗談でしょう?名無しさん:03/11/11 22:20 ID:???
>>116
おいおい、>>115が的確に答えているのに読み取れないのかよ。
そんなんじゃろくな考察かけないぞ。
言葉そのものを受け取るんじゃなくて本質を見抜かなきゃ。
118111:03/11/11 22:33 ID:???
>>117
言葉ではなく本質を見るんですね。
>>115の言いたい事がなんとなく分かったような気がします。
自分で勝手に予測とかしろって事ですよね・・・?
119ご冗談でしょう?名無しさん:03/11/11 22:34 ID:0mLLmmim
抵抗を測る機械って英語で何メーターだったかな?
120ご冗談でしょう?名無しさん:03/11/11 22:36 ID:fd9ySA1f
>>111
後でもいいんだよ、予想するのは。
結果を当たり前だと思えば、その理由を、
結果がおかしいと思えば、その理由を。
121ご冗談でしょう?名無しさん:03/11/11 22:38 ID:???
オームメータ
122111:03/11/11 22:43 ID:???
>>120
ありがとうございます。
やっと意味がわかりました。
多分結果がおかしいっぽいのでその理由と
正しい結果を書きますね。
123ご冗談でしょう?名無しさん:03/11/11 23:00 ID:???
>>122
念のために言っておくが、これまで習っている単純な式に
合わないからといって「おかしい」とは限らない。
摩擦とか空気抵抗などの式に入っていない要素も多いはず。
むしろ、結果が常に正しいと考えるべき。
124ご冗談でしょう?名無しさん:03/11/12 02:24 ID:???
>>123
実験目的に添わない無意味な実験をしていたらどうする?
例えば、誰かが悪戯で装置に磁石をしかけていたとか。
所詮2chだしねぇ。
125ご冗談でしょう?名無しさん:03/11/12 04:20 ID:???
>>124
言いたい事はわかるが>>123の意図も汲んでやろうぜ。
実験手法と適用理論の妥当性についての考察は確かに常識だが、
>>122は実験をしくじったという結論に安易に飛びつきそうだったから、
実験を「教科書通りに行なったという前提で」まずモデルの洗い直しを
してみよう、と言ってるんじゃないかなあ。

にわとりたまごのような気もしてきたが、
普通はコントロールサンプル等で実験の妥当性を主張→理論を検証
だろうな…
126ご冗談でしょう?名無しさん:03/11/12 18:57 ID:???
シリコーン油中での音速及び音波吸収係数を教えてください
127126:03/11/12 19:07 ID:gDWrJvAt
あ、シリコーン油の種類はジメチコンでも何でもいいです

もし分からなければシリコーン油に似た性質の物質を教えてください
128ご冗談でしょう?名無しさん:03/11/12 20:46 ID:HIU4LEfn
やっぱ初心者スレあったほうがよくない?
129ご冗談でしょう?名無しさん:03/11/12 20:58 ID:bUjqoCnk
質問です
http://www.sh.rim.or.jp/~mark/winchesterbaton-02.htm
ここを見てください。剛性は同じ材質肉厚の場合
外径の3乗とありますが肉厚が同じっていうことは
外径が10cmで厚さ1mmの鉄パイプと外形径が5cmの厚さ1mmの
同じ材質のものでは前者が合成が高いということですよね?
これは明らかにおかしいと思いますが。
130ご冗談でしょう?名無しさん:03/11/12 21:03 ID:9gt5YQQN
131ご冗談でしょう?名無しさん:03/11/12 21:15 ID:???
>>129
たとえば上から重みがかかる場合に、円周が長いほうが重みが分散されるから強くなると思わない?
132ご冗談でしょう?名無しさん:03/11/13 13:16 ID:8MTxpr4l
ある本に

光速の半分の速さで地球の側を通りすぎる宇宙飛行士は
世界を一周する信号の正確な遅れを決定するうえで、地上にいる観測者と
深刻に食い違うはずだ。

と書いてありますがなぜか教えて下さい。
133ご冗談でしょう?名無しさん:03/11/13 18:57 ID:WWJcAdZ8
>>131 そうですかね?割れやすいと思いますが。
鉄と紙とでは勝手が違いますかね?
チラシで試してみますが。
134ご冗談でしょう?名無しさん:03/11/13 18:58 ID:???
>>132
(少なくともその引用部分だけでは)日本語が変だよ。
食い違うって、何と何が?
135ご冗談でしょう?名無しさん:03/11/13 18:59 ID:???
それは正確な引用じゃないだろ?
だいたい引用するなら出典も書け。なぜ隠す必要があるんだ?
136ご冗談でしょう?名無しさん:03/11/13 18:59 ID:WWJcAdZ8
単なる重さではなく衝撃の場合も同じですか?
たとえば物を殴るときなど
137ご冗談でしょう?名無しさん:03/11/13 20:54 ID:???
ちょっとお伺いしたいです。文系なんですが、大学で相対性理論を受講しまして。
それでちょっとマイケルソン・モーレーについてぐぐってみたら、偉大な失敗って言われている一方、
何かいろいろ疑問視もされているようで。
それで、一般にMM実験を疑問視する人たちが誤りとして指摘する部分を、
オリジナルと比較して、分かりやすくご教授していただきたいのですが・・・。
どうかお願いします。
138ご冗談でしょう?名無しさん:03/11/13 20:55 ID:ZSpy/7eO
タイムマシンをつくろう!という本です。なぜ地球を一周する光の時間の遅れが地上の人と光速の半分の速度で地球を通る人とで時間の遅れが異なるか?です。
139ご冗談でしょう?名無しさん:03/11/13 21:07 ID:???
文系にしてはわかりにくい日本語を書くな
140ご冗談でしょう?名無しさん:03/11/13 21:19 ID:???
「線密度λの無限に長い直線状電荷があるとき、
この電荷から距離Rでの電場E(R)を求めよ。」

ガウスの法則をどう適用すればよいかわかりません・・・_| ̄|○
141ご冗談でしょう?名無しさん:03/11/13 21:21 ID:ZSpy/7eO
ホーキング、未来を語るという本はどうよ?人類の未来が予想されてるみたいでかなり興味があります。
142ご冗談でしょう?名無しさん:03/11/13 21:23 ID:???
>>140
電荷のある直線を軸とする半径R高さ1の円柱の表面上で面積分する
143ご冗談でしょう?名無しさん:03/11/13 21:38 ID:???
バカチョンカメラのフラッシュはなんで「キィーーーーン」というカッコイイ音がするんですか?
144ご冗談でしょう?名無しさん:03/11/13 21:57 ID:ZSpy/7eO
↑それはお前がバカチョンだからw
145ご冗談でしょう?名無しさん:03/11/13 22:31 ID:lzB2b/14
鉄の熱膨張係数は、結晶構造がfccのときとbccの時で異なりますか?
fccは高温のとき、γ鉄。bccは低温のとき、α鉄。
fccのほうがbccよりも熱膨張しない。
147ご冗談でしょう?名無しさん:03/11/13 22:46 ID:???
>>137
疑問視する人って、具体的に誰?
その人がどこをどう疑問視してるのかを書かないと、「オリジナルと比較」が出来ないっしょ。

おそらく疑問視する人ってのは、反相対論な主張をしてる人なんだろうな。
148ご冗談でしょう?名無しさん:03/11/13 22:50 ID:WWTCwjwd
>>143
コンデンサに電気をためる音
149ご冗談でしょう?名無しさん:03/11/14 05:18 ID:WFrK3jKE
他板住人です。
話の流れで出たのですが、質量保存の法則が働かないことがあるって本当ですか?
また、それはどんな時ですか?
150ご冗談でしょう?名無しさん:03/11/14 06:40 ID:???
他板とはどこか。話の流れとは何か。
必要な内容なら全部かけ。不要なら書くな。

食べた飯より、ウンコの方が軽いのは自明。
ネゲントロピーに関係があるので、自分で調べろ。
151ご冗談でしょう?名無しさん:03/11/14 07:19 ID:???
>149
核反応
152ご冗談でしょう?名無しさん:03/11/14 10:23 ID:ITs5mTFq
宇宙論についての本で、初心者向けのお勧めを教えてくんろ
153ご冗談でしょう?名無しさん:03/11/14 11:36 ID:ddauOEJF
単振りこの運動方程式 d2θ/dt2=-g*sinθ/l のdθ/dtを両辺にかけて
dt=〜〜dθの形に解くやり方を教えてください。お願いします。
154ご冗談でしょう?名無しさん:03/11/14 11:55 ID:Ih9eZUIT
左辺 = d/dt{(1/2)*(dθ/dt)^2}
右辺 = d/dt(gcosθ)
になるから
(dθ/dt)^2 = 2gcosθ+ C (Cは定数)
dθ/dt = ±√(2gcosθ+ C)
dt = dθ/{±√(2gcosθ+ C)}
こんなもんでいいですか?
155ご冗談でしょう?名無しさん:03/11/14 12:01 ID:ddauOEJF
>>154
わかりました。ありがとうございました。
156ご冗談でしょう?名無しさん:03/11/14 12:35 ID:kuNumJW/
152<<宇宙論,一般相対論前提.
http://www.pearsoned.co.jp/washo/physics/wa_phy11-j.shtml
157ご冗談でしょう?名無しさん:03/11/14 14:33 ID:/MR4N8a3
質問です。
量子論において、慣性質量というのはヒッグス粒子によって持たされていると考えられて
いるそうですが、では重力質量は一体どういう由来によるのでしょう?

等価原理によって「慣性質量=重力質量」なのだから、重力質量もヒッグス粒子と何らかの
関係があると考えるのが自然な気がしますが、そのあたりがどうもよくわかりません・・・。
158157:03/11/14 14:35 ID:/MR4N8a3
…はっ! 今ふと思ったのですが、もしかして等価原理って言うのは、
必ずしも「慣性質量=重力質量」と言っているわけではなくて、
単に「慣性質量と重力質量は比例している(重力定数によって)」
ってことを言ってるだけなのでしょうか? 
(いや、それでも関係があることに変わりはありませんが…)
159ご冗談でしょう?名無しさん:03/11/14 14:39 ID:5S7D4Wme
音波って光みたいに屈折しますか?
160ご冗談でしょう?名無しさん:03/11/14 14:55 ID:???
>>158
そうですが、比例定数分は重力定数Gに押しつけてしまえば
慣性質量と重力質量の比例定数は1にできます。
161ご冗談でしょう?名無しさん:03/11/14 16:10 ID:???
固体にあたると反射するわけだが>音波。
162ご冗談でしょう?名無しさん:03/11/14 16:47 ID:???
>>159
します。
163ご冗談でしょう?名無しさん:03/11/14 18:37 ID:???
>>159
地震波も屈折するだろ?それと同じ。
164タカヤ:03/11/14 18:44 ID:qeYkHLwd
すみません。太陽光発電て、どうして可能なんですか?
彼女に聞かれて、答えられなかったもんで、、
165ご冗談でしょう?名無しさん:03/11/14 18:53 ID:g+mXNGAM
太陽から出た光が地球に届くには何分何秒?って問題が出て、
地球と太陽の距離は1天文単位(1.496*10^11)
光速は3.0*10^8m/s

数学わからないんで計算式教えてください。お願いします…。

166ご冗談でしょう?名無しさん:03/11/14 19:00 ID:???
>>164
PN接合部はフェルミ面に「坂」が出来てて、そこに太陽光のエネで
電子ーホール対が発生すると電子とホールがそれぞれに分かれる。
そして電圧が発生する。たしかこんな感じ。
167ご冗談でしょう?名無しさん:03/11/14 19:07 ID:???
>>165
1.496*10^11÷3.0*10^8
168ご冗談でしょう?名無しさん:03/11/14 19:14 ID:???
>>165
距離も速度もわかってるのにかかる時間がわからん、と。
小学校に戻れ
169ご冗談でしょう?名無しさん:03/11/14 19:27 ID:ddauOEJF
すみません。第1種楕円積分というらしいのですが、
   (√(L/2g(cosθ-cosA))dθ
の解き方を教えてください。お願いします。
170ご冗談でしょう?名無しさん:03/11/14 19:41 ID:???
楕円積分は解析的にはできません
171ご冗談でしょう?名無しさん:03/11/14 19:43 ID:ddauOEJF
ではどうやって値を出すのでしょうか?
172ご冗談でしょう?名無しさん:03/11/14 20:20 ID:???
数値積分
173ご冗談でしょう?名無しさん:03/11/14 20:26 ID:ddauOEJF
つまり、実際に数値を代入して細かく分けてやっていくやり方ですか?
174ご冗談でしょう?名無しさん:03/11/14 20:30 ID:???
第一種楕円積分F(θ/2 、 2/(1−cosA))
175ご冗談でしょう?名無しさん:03/11/14 20:39 ID:ddauOEJF
調べてみてわかりました。
ありがとうございました。
176ご冗談でしょう?名無しさん:03/11/14 23:49 ID:iL96/X6T
力学的な量はM、L、Tの三次元、
電磁気学的な量はIをこれに加えて四次元で表現されると聞きましたが、
物理量全体は何次元で表現されるものなんですか?
177ご冗談でしょう?名無しさん:03/11/15 00:13 ID:???
>>179 無次元
178ご冗談でしょう?名無しさん:03/11/15 01:17 ID:???
さて、179は質問考えるのが大変だぞ。
179ご冗談でしょう?名無しさん:03/11/15 04:56 ID:???
ルパンと不二子と五衛門で、合わせて何次元?
180ご冗談でしょう?名無しさん:03/11/15 05:46 ID:???
次元ちゃ〜ん
181ご冗談でしょう?名無しさん:03/11/15 14:35 ID:???
質問です

一次元のシュレーディンガー方程式
(-h^2/2m d^2/dx^2+V(x))ψ(x)=Eψ(x) (hはプランク定数×(1/2π))
の解としてψ1(x)とψ2(x)が得られた場合
Aψ1(x)+Bψ2(x)も解となりますか?
182ご冗談でしょう?名無しさん:03/11/15 14:52 ID:fpkHUkQ4
>>181
実際に左辺に代入して解いてみればいいじゃん。
183ご冗談でしょう?名無しさん:03/11/15 14:58 ID:???
>>182
解いてみたら
左辺=A(-h^2/2m d^2/dx^2+V(x))ψ1(x)+B(-h^2/2m d^2/dx^2+V(x))ψ2(x)
=AEψ1(x)+BEψ2(x)
=E(Aψ1(x)+Bψ2(x))
となったので解になっているとしてよさそうなのですが、
なんとなく騙されているような気がしてなりません。
184ご冗談でしょう?名無しさん:03/11/15 15:01 ID:???
>なんとなく騙されているような気がしてなりません。

どうして??どう見たって解になっとるじゃん。
185ご冗談でしょう?名無しさん:03/11/15 15:07 ID:???
>>184
考えてみたところ
V(x)を波動関数によらない関数としていいのか?
と思ってしまうためにそんな気がするようです。
古典的にはポテンシャルはxのみに依存するとしていいと思うのですが・・・
波動関数のイメージがよくつかめないのでちょっと疑問に思ってしまうのです
186ご冗談でしょう?名無しさん:03/11/15 15:46 ID:???
>>185
ポテンシャルは位置のみによるものしか考えないとしたら
分かるの?ならそれでいいよ。深く考えるな( ´ー`)
187ご冗談でしょう?名無しさん:03/11/15 16:00 ID:???
>>186
位置のみによるものしか考えないとしたらわかります。
なので、深く考えないようにしますw。ありがとうございました。
188大学2年生:03/11/15 16:41 ID:HMtzI7E3
高校の物理で原子分野ですが、ボーアの理論の問題で
クーロン力と遠心力がつりあうとして式を立てますが、
あれって位置と運動量がはっきりしてて、量子力学での
不確定性原理には反しないのですか?
189anyunyu:03/11/15 16:44 ID:COdahafh
グリセリンの粘性は温度が高くなるほど小さくなるのですが、
この温度依存性が定量的にどうであるか、即ち温度の何乗に
反比例しているか、あるいはexpで減衰していくのかなどを
ご存知な方はいらっしゃいますか?
190ご冗談でしょう?名無しさん:03/11/15 16:49 ID:???
>>188 ボーアの理論は量子力学じゃないから・・・
191大学2年生:03/11/15 16:55 ID:HMtzI7E3
>>190
量子力学でしょう。量子条件で電子の速さは観測不可能
だから速さは消去するんでしょ。
言いたいのはその前に釣り合いの式は厳密か?ということ。
192ご冗談でしょう?名無しさん:03/11/15 17:05 ID:???
間違えている上に態度が悪い
193ご冗談でしょう?名無しさん:03/11/15 17:08 ID:???
ああ・・・
194ご冗談でしょう?名無しさん:03/11/15 17:09 ID:???
ということで次の質問いってみよう
195大学2年生:03/11/15 17:10 ID:HMtzI7E3
前期量子論であって、厳密ではないんですね。
仮説式として立てたと解釈してよろしいのでしょうか?
196ご冗談でしょう?名無しさん:03/11/15 17:12 ID:???
大学2年生さんは高校生?それとも大学2年生?
197大学2年生:03/11/15 17:16 ID:HMtzI7E3
そのままです。
198ご冗談でしょう?名無しさん:03/11/15 17:20 ID:???
>>197
だったら運動方程式でなくシューレーディンガー方程式を解きましょう。
199大学2年生:03/11/15 17:22 ID:HMtzI7E3
はい。塾で教えててふと不確定性では?
と思った次第。
200200:03/11/15 17:22 ID:???

200ゲットーーーー!!
201ご冗談でしょう?名無しさん:03/11/15 17:24 ID:???
>>195
ボーアが何を説明しようとしてあの理論にたどり着いたか調べろよ。
大学生なら。

その時代の物理学者がいきなり量子論で物を考えたとでも思ってるのか?
202大学2年生:03/11/15 17:34 ID:HMtzI7E3
いえ、つりあいを考えるのは古典論でしょ。
だから、厳密には正しくないが、そこを高校生に
教えていいものかどうか。
203ご冗談でしょう?名無しさん:03/11/15 17:43 ID:???
>>202
なぜ厳密には正しくないか説明できるなら量子論と比べて教えてやれば。
204ご冗談でしょう?名無しさん:03/11/15 17:44 ID:???
>>202
じゃぁ厳密には正しくないニュートン力学も教えていいものか悩まないの?
205ご冗談でしょう?名無しさん:03/11/15 17:47 ID:ovYV2df2
物理の剛体のつりあいが、まったくわからないのですが
誰か説明して下さらんか
サインとかコサインとかでごちゃごちゃされると
頭が混乱するんですよ


206ご冗談でしょう?名無しさん:03/11/15 17:49 ID:???
>>205
理系から足を洗うべきです
207ご冗談でしょう?名無しさん:03/11/15 17:56 ID:???
>>204
私も塾でバイトしてるけど、そういうことがあるから高校物理は教えたくない。
どうしてもってときはしょうがないけど。そういうときは
「これは古典論であって、入試向けの問題だから云々」
でごまかす。そこでくいいてきたやつには[ある程度]説明するし、
そうでないやつは軽く流す。

理系志望なのにくいついてこないやつは「この程度か」と思ってしまうんだがw
208ご冗談でしょう?名無しさん:03/11/15 18:26 ID:saDWe8Ht
西暦4420年の世界から来たオレから見れば、
お前らのやってる物理は全部間違ってるんだがな。
209ご冗談でしょう?名無しさん:03/11/15 18:29 ID:???
>>208
そうか、がんばるよ。我々の努力の上に君が立っているのだからね。
210ご冗談でしょう?名無しさん:03/11/15 18:32 ID:???
「全部間違ってる」という考えは極めて文系的
211ご冗談でしょう?名無しさん:03/11/15 18:38 ID:???
>>208
モーニング娘は何人になってますか?
212ご冗談でしょう?名無しさん:03/11/15 18:38 ID:???
>>207
なにえらそうなこと言ってんだか。
自分がたまたま生まれた年が早くて先まで知ってるだけでしょ。
初学者にそういう知識を得意げにひけらかしてどうすんだよ。
全く物を教えるセンスないな。
213ご冗談でしょう?名無しさん:03/11/15 18:41 ID:???
↑同意
214ご冗談でしょう?名無しさん:03/11/15 18:43 ID:???
まあ、小学生に「1+1は本当に2なのか?」から始めると発狂するだろうな…
215ご冗談でしょう?名無しさん:03/11/15 19:09 ID:U56l0iad
四谷辺りを隕石が直撃したとして、
どのくらいの大きさだと
山の手線内の地域は壊滅しますか?
216ご冗談でしょう?名無しさん:03/11/15 19:14 ID:???
>>215
そんなの知るかボケ
217215:03/11/15 19:24 ID:???
ボケですいません。
直径5センチの穴に直径5センチの棒を入れることは
可能なのだろうか?と考えるほどボケてます。
218ご冗談でしょう?名無しさん:03/11/15 20:29 ID:???
>>204
そういう問題じゃないだろ。ニュートン力学は理論としては完成してる。
ボーアの理論は未完成。

>>208
先人の思考をたどるのは無意味だと? 一点の曇りもない理論しかやる価値がないと?
一点の曇りもない理論が存在すると?
219ご冗談でしょう?名無しさん:03/11/15 20:38 ID:???
ボーアの理論だってその適用範囲内では完成された理論だが?
220ご冗談でしょう?名無しさん:03/11/15 21:04 ID:fIWM+dr6
まったく。あんなやつが塾の講師をしているとは、教わるほうがかわいそうだよ。
221ご冗談でしょう?名無しさん:03/11/15 22:09 ID:???
>>219
 はぁ?「完成された理論」て、どういうものを指して言ってるんだ?
222222:03/11/15 22:09 ID:???

222ゲッツ!!
223223:03/11/15 22:33 ID:???
223ゲッツ!!
224ご冗談でしょう?名無しさん:03/11/15 22:35 ID:???
224ゲッツ!!
225ご冗談でしょう?名無しさん:03/11/15 22:48 ID:een9eJBB
音のエネルギーって何を指すんですか?
226223:03/11/15 22:49 ID:???
>>225
あなたを指します
227ご冗談でしょう?名無しさん:03/11/16 00:03 ID:???
あん
228ゆうひ:03/11/16 00:17 ID:uH2D6XZ7
地球から変形しない長い棒で月を押し動かせるとしたら、力が光より速く伝わって
手と「同時に」月も動くの?それともタイムラグが発生する?
229ご冗談でしょう?名無しさん:03/11/16 00:25 ID:???
力は光速以下の速さで伝わる。
230ご冗談でしょう?名無しさん:03/11/16 00:29 ID:FsILPhSE
普段物理とは円も縁もない人間ですが、教えてください。
月の上でボールを落としたとき、地面に接している時間は地球よりも長いですか?同じですか?
私は素人考えで長いとおもうのですが。
後、感感俺俺とは何ですか?この板で結構はやっているようです。
震空内では、重いもののほうがなぜか早く落ちるって本当ですか?MMRに書いてありました。
231230:03/11/16 00:34 ID:FsILPhSE
ええと、なんというか説明下手ですいません。
ボールを落としてポーンと跳ね返るとき、その跳ね返る瞬間のボールがひしゃげて上に跳ね上がるまでの時間の事です。
232ご冗談でしょう?名無しさん:03/11/16 00:34 ID:???
感感俺俺って何?
233ご冗談でしょう?名無しさん:03/11/16 00:37 ID:???
>>228
「変形しない長い棒で月を押し動かせるとしたら」という前提なら、
光より速く伝わる。しかし、実際には「変形しない棒」は存在せず、
前提が間違い。
234ご冗談でしょう?名無しさん:03/11/16 01:17 ID:tf29FgTV
核融合の原理を世界で初めて発見したのは誰、そして何年に発見されたの?
235ご冗談でしょう?名無しさん:03/11/16 01:42 ID:???
>>221
適当な原理の元に自己完結した自己矛盾のない論理体系ってとこか。
ニュートン力学が完成した理論っていうのもそういう意味合いではないの?
そうでないなら、ニュートン力学とボーア理論での違いがわかるように
「完成した理論」の意味を説明してほしいな。納得いく説明なら賛成するよ。
単に「完成」の解釈の違いというだけだから、どちらが間違いという
わけではないからね。

先に言っとくけど、単に量子力学の視点から見て未完成とかいう説明だと
納得しないからね。それだったらニュートン力学だって相対論から
見れば未完成と言ってやらないと筋が通らないからね。
236ぽん:03/11/16 01:52 ID:wEs8P1Ie
「電気」に詳しい方、居ますかぁ???
237ご冗談でしょう?名無しさん:03/11/16 02:17 ID:???
 円運動の問題で、
垂直に円運動をさせる条件の話なんですが

 支点から紐でぶら下げている場合と、棒でぶら下げている場合、
とで円の真上まで到達するための条件がわかりません
 紐の場合
 円の真上でのことを考えて
 紐の張力T、遠心力F、物体にかかる重力Gで式を立てたとき
 F−T−G=0、とT>0
 でいいのかな・・・

 棒の場合
 棒にかかる力S、遠心力F、物体にかかる重力Gでどういう式
になるんでしょうか。
238???:03/11/16 02:19 ID:???
???
239ご冗談でしょう?名無しさん:03/11/16 02:53 ID:???
>>237
物体の方程式立てるのに、なんで棒にかかる力Sが出てくるのよ?w

下端での運動エネルギー=上端での位置エネルギー
って式立てとけ。棒から受ける力は運動と垂直方向なので仕事しない
からエネは保存するわけだ。分かったか坊主?
240ご冗談でしょう?名無しさん:03/11/16 02:58 ID:???
>>239
 ということは、紐の場合でもエネルギー保存則で一発ということでしょうか?

 後出しで申し訳ないのですが、問題としては
一番下での初速度をどれだけにすれば一番上にいけるか
なのですけど・・・
241ご冗談でしょう?名無しさん:03/11/16 03:47 ID:oiorRxb8
>>228
「変形しない」というのがそもそも非相対論的。
原子レベルで考えると、剛体というのは端から端まで「一瞬で」クーロン斥力の効果が伝播するという風に近似している。
つまり端から端まで疎密波が0秒で伝播するとしているのが間違い。

>>225
そもそも音って何だっけ?
242ご冗談でしょう?名無しさん:03/11/16 03:59 ID:oiorRxb8
>>237
いや。棒はたるまないから一番上で速度0でもイイ。
でも紐はたるむから一番上で円運動が成立するだけの速度が必要。
つまり実効的な遠心力が正でなくては鳴らない。
棒の場合はたるまないので0以上であればよい。
>一番下での初速度をどれだけにすれば一番上にいけるかなのですけど・・・
君は問題文を紹介するセンスに書ける。そこが一番重要なファクターだ。
243ご冗談でしょう?名無しさん:03/11/16 06:39 ID:RlkNN4Hi
Velocity: 300 feet per second は、何ジュールですか?
244ご冗談でしょう?名無しさん:03/11/16 06:42 ID:???
運動エネルギーなめんなよ
245ご冗談でしょう?名無しさん:03/11/16 06:43 ID:???
イエッサー!
246243:03/11/16 07:02 ID:RlkNN4Hi
すみませんでした。
重さ0.90gです。
247ご冗談でしょう?名無しさん:03/11/16 07:07 ID:6ZplIPp+
>>243=>>246
300feetは何mですか?
0.90gは何kgですか?
変換できれば、わかると思います。
248???:03/11/16 07:26 ID:???
そうだそうだ変換しろよボケが!
249ご冗談でしょう?名無しさん:03/11/16 08:01 ID:AKJMIrQw
塗膜中のポリエチレンをポリプロピレンに変更すると極端に滑りが悪く
なるんだが、この理由がわかる香具師おる?
250ご冗談でしょう?名無しさん:03/11/16 09:00 ID:???
>>249
それは物理の範囲の問題なのか?

勿論、自然現象が「・・・の範囲」と別れているワケではないが、
分子形状とか重合の様子とかって話だと板違いカモだしね。
251ご冗談でしょう?名無しさん:03/11/16 09:04 ID:???
>>249
物理の範囲だったら、動摩擦係数がわかれば良いと思いますが、
そうでなければ化学、特に高分子の人に聞いたほうが早いと思います。
252ご冗談でしょう?名無しさん:03/11/16 09:06 ID:???
>>236
電気といっても、どういう分野で詳しい人ですか?
質問があれば、ぜひ質問してください。
253ご冗談でしょう?名無しさん:03/11/16 10:13 ID:???
>>235
221じゃないが、
荷電粒子である電子が古典力学の法則に従って円運動すれば、その周囲の電場が
変化する。それにも関わらず、電磁波は放出されないと考える点、ヘリウムの振舞を
説明できない(適用範囲の異常な狭さ)点、などなど。
254ご冗談でしょう?名無しさん:03/11/16 11:55 ID:???
赤い星に向かってスピードを上げて、近づけば
ドップラー効果で青に見えることはあるんですか?

赤が周波数でないならば、何なのですか?
おしえてくりくり。
255ご冗談でしょう?名無しさん:03/11/16 11:59 ID:???
>>254
ドップラーで青に見えるし、赤は周波数だろ。
256ご冗談でしょう?名無しさん:03/11/16 12:08 ID:g0QFfce5
実験的に測定された重力加速度(の平均値)が時代によってドリフトするっていう話を
何かの本で読んだのですが、忘れてしまいました。
何に載っていたかどなたかご存じでしょうか。

話の趣旨は、
実験結果を吟味するときに過去のデータ(結果的に間違っていても)を
参照し、ほぼ一致していればその実験結果に満足し、ズレがあれば
その理由を探して再試をする。次第に実験の精度が上がるので
だんだん正しい(と思われる)値に近づいていく。
257厨房:03/11/16 12:29 ID:e3cO97bA
空気(酸素?)と水の元素記号ってなんでしたっけ?
258ご冗談でしょう?名無しさん:03/11/16 12:33 ID:???
この中のどれか。自分で思い出しなさい。
N2,O2,CO2,NH3,H2O,H2,Ne,Ar
259ご冗談でしょう?名無しさん:03/11/16 12:33 ID:y43zA3ey
ああ、なんかで読んだことがあるな。
ただし重力加速度じゃなくて素電荷だった気がする。
ミリカンのやつね。
260ご冗談でしょう?名無しさん:03/11/16 12:35 ID:g0QFfce5
>>259
そうだったかも知れません。学部の前半くらいで読んだ本だった気がする。
この話の重要なのは、真の値の周辺に分布するのではなく、
過去の平均値の周辺に分布することです。
261厨房:03/11/16 12:39 ID:e3cO97bA
>>258
ありがとう。H2O、O2、CO2あたりだっけ、、、?空気読めてなかったか。ごめん。
262ご冗談でしょう?名無しさん:03/11/16 13:50 ID:???
文字通りな。って言ったら負けなんだろうな。でも言う。
263ゆうひ:03/11/16 14:52 ID:uH2D6XZ7
>241
てことはダイヤのお箸を押したときも、押しつぶされる変化が波のように伝わるってこと?
変形しない物体は存在しないってことになる?
動く物は全て変形しながら動いているということ?
264ご冗談でしょう?名無しさん:03/11/16 16:10 ID:???
>>263
そうでつ。
それぞれの分子なり原子なりの相互作用の結果、物質を形作っている。
あらゆる相互作用は光速を越えることが出来ません。
265225:03/11/16 16:21 ID:???
>>241
気圧が伝わると表現としていいのでしょうか、疎密波ですよね?
振幅は音の大きさだし、振動数は音の高さだし・・・。
音のエネルギーって何だろう、って思ったんです。
266ご冗談でしょう?名無しさん:03/11/16 16:41 ID:aLoTUGqY
>>265
そんなに哲学的に悩まなくても分かりますよ。
要するに低気圧と高気圧があるだけ。
高気圧の部分から空気分子w が拡散しようとする力がエネルギーの源。
細い管の中を音波が通る様子を分子レベルでイメージしてみる。
267ご冗談でしょう?名無しさん:03/11/16 16:45 ID:oEHVX7w7
原子量表から得た分子量と、実験で理想気体として求めた分子量で
圧縮係数を求めたいのですが、この場合は1となるのでしょうか?
教えてください
268ご冗談でしょう?名無しさん:03/11/16 16:54 ID:VVNzM1DO
>>267
実験結果を聞いているのですか?
それは実験してみないと分かりません。
269yuki:03/11/16 17:07 ID:S4UPQI9f
こんにちは。
いつも大変興味深く皆さんの書き込み読ませて貰ってます。
ちょっとわからなくて困ってます。皆さんの知恵を貸して下さい。

日常生活において、強い電磁界(電磁場)の例をみつけなさい。どのような電磁界になっているか想像し、簡単に説明せよ、(図もできれば・・)

このような問題がでて私の乏しい知恵ではわかりません。。泣) 参考になるサイトや、皆さんの知恵があれば、是非メール下さい。お願いします。
270ご冗談でしょう?名無しさん:03/11/16 17:09 ID:???
マイケルソン干渉計を使って気体の屈折率を測定する実験をしました。

一方の光路の途中にポンプで真空にした管を設置しておいて
その管に気体を徐々に入れていくと干渉縞に変化が現れます。
その変化の回数から屈折率を求める、という実験でした。

干渉縞の変化の回数をN、管の長さをL、光の波長をλとするときその気体の屈折率nは
n = 1 + (Nλ/2L)で求められる、という考え方は分かりました。
そして、気体の種類によって屈折率が異なります。

以下の質問の解答をご教授ください。

1:気体によってnの値(Nの値)が異なる理由を説明できないので教えてください。
2:nは気体の持つ物理量の何に依存しますか?
271ご冗談でしょう?名無しさん:03/11/16 17:58 ID:BhiuIkaW
格子ベクトルと基本並進ベクトルの違いを教えてください
272ご冗談でしょう?名無しさん:03/11/16 18:24 ID:IA35UmoO
強い電磁場? 電線があるじゃん。高圧電線の周りに
磁場が発生します。妊娠中の女性は奇形児が産まれる
可能性があるので、その周りに住んではいけないらしい。
273ご冗談でしょう?名無しさん:03/11/16 18:40 ID:???
質問願います。
タバコのけむりはどうして風もないのに
まっすぐのぼっていかないんですか?
レイノルズ数とか流体力学のうわっぺらは
習ったんですがそれだけじゃわかりませんよね?
>>272
高圧電線って交流?何本もあるけど全部同時に同方向に流れてるの?
ウマク相殺してそうだけど。
275ご冗談でしょう?名無しさん:03/11/16 18:48 ID:???
>>273
ブラウン運動と対流でググって分からなければまた聞いてね。
276ご冗談でしょう?名無しさん:03/11/16 18:51 ID:???
>>249
定性的な答えでいいなら、多分ポリエチレンのほうが結晶性が高いため、
ゴム的な性質がより少なくなるからだね。
277ご冗談でしょう?名無しさん:03/11/16 18:56 ID:???
>>276
と思ったが、塗料の話だったら、ポリプロピレンのほうが溶媒に溶けやすくて、
つぶつぶの効果がより少なくなるからという可能性もあるな。
278ご冗談でしょう?名無しさん:03/11/16 19:05 ID:???
>>270
媒質の誘電率、透磁率、屈折率らへんでしらべてみれば?
279ご冗談でしょう?名無しさん:03/11/16 19:51 ID:HLMjQ7Rj
ポインティングベクトルが0になるときってどういう時ですか?
280ご冗談でしょう?名無しさん:03/11/16 20:05 ID:???
>>279
ポインティングベクトルは何を表しているのでしょう?
それ調べてまたきて!
281ご冗談でしょう?名無しさん:03/11/16 20:27 ID:???
エネルギーが伝播しな
282ご冗談でしょう?名無しさん:03/11/16 20:44 ID:???
>>279 電場が0のとき
283ご冗談でしょう?名無しさん:03/11/16 20:52 ID:HLMjQ7Rj
>>280-281
ググったらポインティングベクトルの面積分は、単位面積あたりの電磁波が単位時間に持ち運ぶエネルギーを表していることと一緒に
↓みたいなことも書いてあったんですけど電磁波以外だとポインティングベクトルは何を表すんですか?
それとも電磁波以外にはポインティングベクトルは定義されないんですか?

>ポインティングベクトルの解釈について誤解のないようにしておかなければならない。
>電場と磁場が直交して存在していれば確かにポインティングベクトルが定義できる。
>しかしポインティングベクトルがあるからと言って、必ずしもエネルギーの移動があるとは考えられないことに注意しておこう。
>例えば、2枚の金属板を帯電させて並行に置くことでその間に静電場を作り出す。
>そしてそれに対して磁石を横向きに置けばこの空間内でポインティングベクトルが計算できることであろう。
>果たしてこの空間内をエネルギーが一定方向へ流れていると言えるだろうか?
>この範囲から出て行くポインティングベクトルは計算できるし、反対側からは入ってきているように計算できるだろう。
>しかし実際にはこの空間には外部とのエネルギーの出入りはない。
>計算上、上の式が成り立っているだけである。
>ポインティングベクトルがエネルギーの流れを表すとの解釈は、この式を電磁波の場合に適用した場合にだけ出来ることなのである。
284254:03/11/16 20:56 ID:???
>>255
どうもです。
赤が周波数できまるなら、いったい
285ご冗談でしょう?名無しさん:03/11/16 21:18 ID:???
>>284
いったいなんだ?
286ご冗談でしょう?名無しさん:03/11/16 21:21 ID:???
>>284
いったいなんなんだ?
287254:03/11/16 21:43 ID:???
赤はどこにあるんだろう?
288ご冗談でしょう?名無しさん:03/11/16 21:48 ID:???
亜光速で飛行するスパゲッティにあります
289ご冗談でしょう?名無しさん:03/11/16 22:24 ID:???
>>275
ありがとうございました。いろいろ調べてみましたが
もうちょっと調べてみます。
290221:03/11/16 22:26 ID:???
>>235
 だからボーアの理論だと自己完結しとらんだろ。
 たとえば水素原子の電子模型だったら、ボーアの理論だとなぜ「定常状態」が
軌道長が「波長」の整数倍のものしかないのか説明できてない。これだと自由な
電子の運動との整合性がつかない。

 原理から始まって、観測と理論の数学体系との結びつきがきっちり定義できて
いなけりゃ、その理論は未完成だということでいいと思うが?
291ご冗談でしょう?名無しさん:03/11/16 22:30 ID:???
繰り込みの必要な標準模型も未完成な理論だという事でよろしいか?
292ご冗談でしょう?名無しさん:03/11/16 23:07 ID:???
>>291
 よくわからんけど、いいんじゃない?繰り込みが数学的にきちんと
決まってないのなら。
293ご冗談でしょう?名無しさん:03/11/16 23:13 ID:D0SWPWIU
関数u_1(x),u_2(x)がシュレディンガー方程式HΨ(x)=EΨ(x)の解であるときA_1u_1(x)+A_2u_2(x) (A_1,A_2は定数)
も解となるか?という問題ですが、
この場合 Eの値がA_1,A_2によって異なるかどうかで解になるかどうかがかわってくると思うんですが、答えはどうなるのでしょうか?
294ご冗談でしょう?名無しさん:03/11/16 23:42 ID:???
異なるエネルギー固有値に属するエネルギー固有関数を重ね合わせたら
エネルギー固有関数ではなくなるわな。
295ご冗談でしょう?名無しさん:03/11/16 23:50 ID:???
>>293
> Eの値がA_1,A_2によって異なるかどうかで

EとA1,A2は関係ない。 E1、E2が等しいかどうかだけが問題。
E1=E2ならA1,A2によらず重ね合わせ出来るし、そうでなければ出来ない。
296ご冗談でしょう?名無しさん:03/11/17 00:24 ID:XKcsvyYr
原子ってうんこするの?
297ご冗談でしょう?名無しさん:03/11/17 00:26 ID:???
するよ
298ご冗談でしょう?名無しさん:03/11/17 00:37 ID:COWkcsG/
角速度ベクトルωe{z}で回転している系って、z軸をコマの軸にしてxy平面が回転している感じですよね?
そこにt=0の時に(0,0,z)の点Aを設定して、系をぶん回してもAは動かない(系の外から見て)よーな気がするんですが、何か根本的に間違えていますか?
299ご冗談でしょう?名無しさん:03/11/17 02:33 ID:???
>>290
ボーア理論では量子化条件は説明を要しない原理のひとつなんだが?
繰り返しに近いけど、量子力学の立場から見れば、より基本的原理から
演繹されるから自己完結してないってなら、ニュートン力学だって自己
完結してない未完成な理論。
300ご冗談でしょう?名無しさん:03/11/17 02:35 ID:???
>>298
Aは動かない、で正しいけど、何を根本的に間違えてると感じてるの?
301ご冗談でしょう?名無しさん:03/11/17 02:58 ID:COWkcsG/
>>300
いや、宿題なんだけど・・・。

>慣性系Sと原点を同じにする非慣性系S'がある。S'は角速度ベクトルωe{z}で回転している。
>時刻t=0に両者の座標軸xyzおよびx'y'z'は一致している。
>1.S系において座標(0,0,a)に静止している質量mの質点をS'系で観測したときの座標を時間tの関数としてあらわせ。

問題の意図からして「動かない」じゃ変だと思うわけだが・・・漏れ、問題読み間違えてる?
302ご冗談でしょう?名無しさん:03/11/17 04:46 ID:???
固体物理についてググってたら飛び移り積分って言葉がでてきたんすけど
飛び移り積分ってなんですか?
303293:03/11/17 10:21 ID:???
>>294 >>295さん

遅くなりましたが、どうもありがとうございました。
304ご冗談でしょう?名無しさん:03/11/17 11:18 ID:???
>>302
分子軌道法で使う、電子の原子サイト間の移動のしやすさの指標みたいな
のだったと思う。数学的な積分計算法の一種とかではない。
305ご冗談でしょう?名無しさん:03/11/17 14:01 ID:Nzgg/jVZ
物質には必ず温度があるが
1個1個の電子、原子には温度があるの?
306ご冗談でしょう?名無しさん:03/11/17 14:10 ID:???
>>305
ではそもそも、温度とは何でしょーか。
307ご冗談でしょう?名無しさん:03/11/17 14:47 ID:???
>>305
高校に入ればすぐ分かるよ
308ご冗談でしょう?名無しさん:03/11/17 15:36 ID:jDk0TBQa
取り敢えず、揚げ。
309ご冗談でしょう?名無しさん:03/11/17 15:48 ID:???
>>298
>系をぶん回してもAは動かない(系の外から見て)よーな気がするんですが

系の外から見ちゃ駄目でしょ
S'系の任意の座標から観測した時どのように見えるのかを求める
310ご冗談でしょう?名無しさん:03/11/17 17:27 ID:NDPPjoMA
シルマナル磁石ってなんなんでしょうか?
長い間磁界やら磁束密度やらやってきたけど、いきなり講義で出てきて・・・
どなたか詳細おながいします(´・ω・`)
311ご冗談でしょう?名無しさん:03/11/17 17:35 ID:???
シルバー + ゲルマニウム ?
312301:03/11/17 18:46 ID:COWkcsG/
結局、出題ミスだったそうです(´・ω・`)
313ご冗談でしょう?名無しさん:03/11/17 18:59 ID:???
{an} = 正n面体の、頂点の数×辺の数×面の数 …失敗作ですた
314ご冗談でしょう?名無しさん:03/11/17 19:58 ID:tD/ovBjc
質問です。
核分裂反応の際に生じる、あの膨大なエネルギーというのは、反応した核物質の質量がエネルギーに
転換しているわけですよね。
それでは、他の反応…例えば化学反応などでエネルギーが生じた場合にも、一見質量保存の法則が
成立してるように見えて、実は(物凄く厳密に計測したら)反応前と後で総質量がほんっっっの少し
だけ減少していて、それがエネルギーになっているのでしょうか?

それとも、「質量がエネルギーに転換する」というのは、核分裂や核融合に特有な現象なので
しょうか。
315ご冗談でしょう?名無しさん:03/11/17 20:10 ID:???
化学エネルギーって結局なんだったんだろう・・・。
316ご冗談でしょう?名無しさん:03/11/17 20:10 ID:V1uG0uVb
>>314
特有
317ご冗談でしょう?名無しさん:03/11/17 20:13 ID:???
>>314
ある意味正解。というより、質量がエネルギーに代わっているのではなくて、
エネルギーが質量に見えているだけ。
化学エネルギーというのは結局のところ、原子同士の結合エネルギー。
そして原子同士が結合した段階でエネルギーが質量に変換されている。

ついでに言えば、運動エネルギーも質量に変換できる。
物体を加速するとその質量が増加するというのは、運動エネルギーが質量に変換されているため。
318ご冗談でしょう?名無しさん:03/11/17 20:13 ID:A8jJ6RDN
>>315
EM
319ご冗談でしょう?名無しさん:03/11/17 20:14 ID:???
>>314
>実は(物凄く厳密に計測したら)反応前と後で総質量がほんっっっの少し
>だけ減少していて、それがエネルギーになっているのでしょうか?
そのとーり
320ご冗談でしょう?名無しさん:03/11/17 20:14 ID:???
>>317
最後の2行は余計
321ご冗談でしょう?名無しさん:03/11/17 20:15 ID:A8jJ6RDN
>>317
まぁ乾電池も使用後の方が軽いからな
322ご冗談でしょう?名無しさん:03/11/17 20:16 ID:???
>>320
なんでよ
323ご冗談でしょう?名無しさん:03/11/17 20:21 ID:???
化学ポテンシャルって、粒子を一つ系に加えるエネルギーとかいうけど、
結局は電磁的なエネルギーに帰着するの?
324ご冗談でしょう?名無しさん:03/11/17 20:32 ID:???
原子核半径の遥か遠方では、核力・弱い力の影響はほぼゼロになり、
重力相互作用はもともと弱い。
従って、原子サイズより大きいスケールで働く作用はほとんど電磁気力に起因するものとなる。
(小惑星規模ともなると重力が無視できないほど効いてくるが)
325ご冗談でしょう?名無しさん:03/11/17 20:35 ID:???
っつーことは、複雑すぎて扱いきれない電磁ポテンシャルは全部化学エネルギーってことにして
お茶を濁しているわけか。なんだかなぁ・・・。
326ご冗談でしょう?名無しさん:03/11/17 20:37 ID:???
化学屋にとっては、そんなこたーどうでもいいし。
327ご冗談でしょう?名無しさん:03/11/17 20:38 ID:???
>>322
相対論的質量なんて使うなってこった
328ご冗談でしょう?名無しさん:03/11/17 20:39 ID:???
>>325
複雑すぎて扱いきれない核力ポテンシャルは全部結合エネルギーってことにして
お茶を濁しているわけだが、文句ある?
329ご冗談でしょう?名無しさん:03/11/17 20:41 ID:???
いや、文句を言うほど偉くはないけど精密科学を標榜する物理も結構いい加減な部分があるんだなって思っただけです。
330ご冗談でしょう?名無しさん:03/11/17 20:46 ID:???
>>329
そうですかテーラー展開二次で打ち切っちゃ駄目ですか
331ご冗談でしょう?名無しさん:03/11/17 20:48 ID:???
>>317
>運動エネルギーが質量に変換されている
これだと、変換された質量で考えるときは、運動エネルギーは0にしないと
ダブルカウントすることになるね。つまりその変換された質量は静止している
とみなすことになる。変でしょ
332ご冗談でしょう?名無しさん:03/11/17 20:48 ID:???
>>329
 まぁ必要なだけ精密であればいいわけだし。お茶濁して済ましたくなければ
そうすれば良い。労多くして益少ない仕事だが。
333ご冗談でしょう?名無しさん:03/11/17 20:48 ID:???
>>329
自然の面白いところは、そういう大雑把な近似をしても「結構うまくいっちゃう」ところですね。
PV=nRTは実用的に成り立ってるし、
惑星の運動も精密に予測できてますよね。
334ご冗談でしょう?名無しさん:03/11/17 20:49 ID:???
>>329
1.0000・・・(0が40個位)・・・0001 = 1 にして文句あっか?
原子での重力と電磁気力の差なんて10^-40乗のオーダーなわけだが。
そもそもそこまで測定精度は高くない。
335ご冗談でしょう?名無しさん:03/11/17 20:50 ID:???
>>331
運動エネルギーの差で勘定すればいいんでないかい?
336ご冗談でしょう?名無しさん:03/11/17 20:51 ID:???
二次までやらないでしょ?一次で打ち切ってなんでもかんでも線形にして不思議な踊りを踊るのが物理でしょ。
337ご冗談でしょう?名無しさん:03/11/17 20:51 ID:???
>>333
円周率3でもうまくいくもんな








ンなわけないか
338ご冗談でしょう?名無しさん:03/11/17 20:53 ID:???
>>337
オーダーの暗算なら3でやってますけど

>>331
不変質量…
339ご冗談でしょう?名無しさん:03/11/17 20:56 ID:???
まぁ目的によるわな。>近似
下手な近似すると意味の無い答え出てくるし。
340ご冗談でしょう?名無しさん:03/11/17 21:05 ID:???
まぁ必死こいて3次の計算してるのに別の場所で1次の近似で済ましているとゆー
無意味な努力はしばしばやってしまうものでつ
341ご冗談でしょう?名無しさん:03/11/17 21:06 ID:???
>>335
どう勘定しても、運動エネルギー(の差)を質量に転換させるなら、
そのぶん速度を変えて考えない限り二重カウントになって同じこと
342ご冗談でしょう?名無しさん:03/11/17 21:07 ID:???
マンドクセから1次の近似でやったら、全部消えちゃって2次の近時でやり直すという二度手間はしばしばやってしまうものでつ



>>340と漏れのタイプの違いがよくわかるな
343314:03/11/17 21:18 ID:tD/ovBjc
皆様ありがとうございました。おかげさまで疑問が解けました。

ところでもう一つ、ちょっとした質問なのですが、「核力」って
英語でどう綴るんでしょう?
344ご冗談でしょう?名無しさん:03/11/17 21:20 ID:???
>>343
ちったー自分で調べろよ。

nuclear force

マンマだ。
345ご冗談でしょう?名無しさん:03/11/17 21:21 ID:???
michio-force
346314:03/11/17 21:21 ID:tD/ovBjc
まんまなんですね…くだらない質問でスミマセンでした…。
347ご冗談でしょう?名無しさん:03/11/17 21:25 ID:???
 やっぱみんな近似計算には苦労してるのねぃw
348ご冗談でしょう?名無しさん:03/11/17 21:31 ID:???
近似計算は物理のキモの1つだと思うがな。

相対論大嫌いDQNの場合、たいてい近似がおかしくて妙な結論が出てくる。
349ご冗談でしょう?名無しさん:03/11/17 21:32 ID:???
つまらない近似のごくごくわずかな工夫を“発明”したつもりになって論文を書いてるヤツがほとんどでしょ。
物理はもう死にかけてるよ。
350ご冗談でしょう?名無しさん:03/11/17 21:35 ID:???
はいいしば → 配意芝
しってん → 失点
へんびぶん → 片微分
やこびあん → 屋媚案
とくいてん → 解く移転
かくそくど → 書く速度
かくうんどうりょう → 書く運動量

こんな珍変換にうんざりしているんだが、いい物理・化学・数学用辞書(フリー)って無いかね?
351ご冗談でしょう?名無しさん:03/11/17 21:44 ID:???
>>350
釣りじゃないと信じています!よろしくおねがいしまっす!!

「IME 化学 辞書」でググる。
例えば
http://lsd.pharm.kyoto-u.ac.jp/Others-J.html
352ご冗談でしょう?名無しさん:03/11/17 21:50 ID:???
OK言い方が悪かった。

ぐぐればいっぱい見つかるが、お前らのお勧めはどれですかい?ってことですがな
353ご冗談でしょう?名無しさん:03/11/17 21:54 ID:???
>>352
かたっぱしから単語登録してる。他人の作った辞書をとりこんでも、スピード落ちるだけ。
同じ職場、研究所にあるパソコン登録データをかき集めるのも手ですな。
354ご冗談でしょう?名無しさん:03/11/17 21:56 ID:???
スピード落ちるってお前何年前のパソコン使ってるんだよ
あ、DOS?
355ご冗談でしょう?名無しさん:03/11/17 23:29 ID:???
●慰謝料が気になったらageるスレ●
http://school.2ch.net/test/read.cgi/shikaku/1065261585/l50
356221:03/11/17 23:43 ID:???
>>299
 ボーアの量子論の原理である「量子化条件」がかなり曖昧だと言ってるの。
この原理から観測結果の予想ができないでしょうが。予想しようとするなら
注目する系によって恣意的な条件を持ち込まないといけない。
 何も「より基本的な原理」なんぞ持ち出そうなんて思っとらんよ。あくまで
その理論が閉じているかどうかが問題なんだから。
357ご冗談でしょう?名無しさん:03/11/18 00:12 ID:L5OUft7+
>>356
>この原理から観測結果の予想ができないでしょうが。
ハァ?
水素原子のスベクトル系列を見事に説明しましたが何か?
358ご冗談でしょう?名無しさん:03/11/18 00:27 ID:???
なぁ、光速に近い物体って時間が長くなってるの?それとも長さが縮んでいるの?
素粒子や何かの場合、時間が長くなっているから寿命が延びているのか、走るべき長さが短くなっているから寿命そのままでも到達できるのか区別つかんよね。
実際にはどっち?また、区別できるのなら区別つける方法を教えてくだちぃ。
359ご冗談でしょう?名無しさん:03/11/18 00:36 ID:???
ローレンツ収縮は実際には起きていないことを誰か証明しなかったっけ。
360yuki:03/11/18 01:06 ID:pLVOvLKS
力の種類を調べてるんですが、変わった力(あまり知られてない力)の種類について
教えて下さい♪その際、その力の種類に対する具体的な説明お願いします♪
361ご冗談でしょう?名無しさん:03/11/18 01:11 ID:???
厨房の宿題だな。

よろしい、教えてやろう

精神力:精神を集中することによって生じ、物的条件を超えて働く力。
忍耐力:苦しみ・つらさなどに耐える力。
気力:精神力に同じ。
精力:物事をやりぬく心身の元気。
暴力:乱暴な力。無法な力。
腕力:腕の力。
筋力:筋肉の力。
魔力:人間わざとも思えない不思議な力。
霊力:霊魂の力。精神の力。

こんなもんでいいか?
362ご冗談でしょう?名無しさん:03/11/18 01:12 ID:???
>>360
消防?

重力、電磁気力、強い力、弱い力、以上。

重力  →言わずもがな。
弱い力 →分子間力。水が水蒸気になると分子間力弱くなる。
強い力 →原爆。原子核エネルギー。物凄いよ。
電磁気力→他のもの。おぬしが動いてる力は電磁気力。
363ご冗談でしょう?名無しさん:03/11/18 01:18 ID:???
おいおいおいおいおい弱い力は分子間力じゃねーだろ。
364ご冗談でしょう?名無しさん:03/11/18 01:19 ID:???
>>362
アホ
365AJへぇ〜 ◆2chDQNAJHE :03/11/18 01:25 ID:ugqZIyRg
通は分子間力=β崩壊だろ?
366ご冗談でしょう?名無しさん:03/11/18 01:26 ID:???
まぁ厨房が消防の宿題教えてやるんだからこんなもんだろ。
367ご冗談でしょう?名無しさん:03/11/18 01:48 ID:???
あえてマジレスすると、何ちゃら力ってアホみたいにあるよな。

今さっと思いついただけでも
重力、核力、中間子力、電磁気力、ファンデルワールス力、ローレンツ力、
コリオリ力、湯川力、遠心力、浮力、揚力、圧力、摩擦力、慣性力、中心力、
向心力、斥力、反発力、引力、合力、張力、潮力、風力、原子力、火力

なんか色々と違うものが混ざってる気がするが
368ご冗談でしょう?名無しさん:03/11/18 02:10 ID:???
視力、聴力、抗力、光力、
369ご冗談でしょう?名無しさん:03/11/18 02:11 ID:???
>>365
いいから黙れ。煽りはいいが嘘はいけねぇ。
370ご冗談でしょう?名無しさん:03/11/18 02:12 ID:???
>>359
どういう事?「実際には起きていない」とはどういう事ですか。相間の人?
371ご冗談でしょう?名無しさん:03/11/18 02:21 ID:???
そういや、ちょっと前には
分子間力が重力由来だとか豪語してる香具師がいたな。
372ご冗談でしょう?名無しさん:03/11/18 02:30 ID:???
長州力
373ご冗談でしょう?名無しさん:03/11/18 02:31 ID:Ns1smISu
準静的変化によって儔を受け取ったとき、エントロピーの変化 儡=儔/T というのは、準静的変化じゃないときは
儡はどうなるんですか。高校の数学2割くらいの達成率と中学の数学で分かるような説明が存在したら教えてください。


あと、分子論的には S=klnW でWはミクロ状態の数というのは、ミクロ状態の数ってどういう意味ですか。高校の物理達成率3割のレベル
で教えてください
374ご冗談でしょう?名無しさん:03/11/18 02:43 ID:???
>>373
舐めた質問の仕方してんな。
375ご冗談でしょう?名無しさん:03/11/18 07:47 ID:???
>>358
観測者の視点。
傍観者は時間の伸びと見るし、
運動者は距離の縮みと見る。
376ご冗談でしょう?名無しさん:03/11/18 08:07 ID:???
>>370
江沢洋の「現代物理学」にはローレンツ短縮の項に
> ・・・(3.1)をローレンツ短縮というが、本当にそれがおこっているなら物体
> 内部にひずみができているはずだ。ガラスを圧縮すると光は複屈折をおこすが、
> 同じことがローレンツ短縮でも起こるか?電気抵抗は変わるか?物質による差は
> 本当に無いか?実験の結果は、すべて’ない!’であった。
と書いてまふ。だから漏れはローレンツ短縮は見かけの効果に過ぎないと
理解したんだが、間違ってる?
377ご冗談でしょう?名無しさん:03/11/18 08:30 ID:???
>>356
だからボーア理論の中では閉じてるでしょうが。
採用されてる原理が恣意的かどうかなんて問題にしてない。
キミこそニュートンとボーアの間に完全と不完全の境界を
引きたいあまり「原理が恣意的」などという主観的な理由を
恣意的に出して来たとしか思えんな。

ゾンマーフェルトの量子化条件くらい勉強した上で恣意的と
評してるのかね? それが恣意的でニュートンの運動の三法則は
恣意的でない? これまた恣意的で砂
378ご冗談でしょう?名無しさん:03/11/18 08:33 ID:???
>>376
内部にひずみができるような力学的圧縮ではない、としか言ってないね
379ご冗談でしょう?名無しさん:03/11/18 10:04 ID:???
>>376
質問です。空間(ものさし)が歪んでいる事を、ものさしを使って知る方法はありますか?
380ご冗談でしょう?名無しさん:03/11/18 10:27 ID:???
>>377
>>253御嫁。
381ご冗談でしょう?名無しさん:03/11/18 11:11 ID:ONZx0NdB
放射能と放射線てどう違うの?
またどういう関係なの?
382381:03/11/18 11:14 ID:???
すんません。
ちょっと調べたら出てきました。
383ご冗談でしょう?名無しさん:03/11/18 12:15 ID:???
>>380
ボーア理論では、電磁波は放出されないというのは原理であって
それ以上の説明は求めない。何度言わせるのかね

ボーア理論ではヘリウムは適用範囲外であって、それが
説明できないから不完全だと言うなら、ニュートン力学も
光速近くや大重力場での振舞を説明できないから不完全だと
言わないと筋が通らない
384ご冗談でしょう?名無しさん:03/11/18 13:02 ID:???
いやニュートン力学は不完全だけど、不完全だからこそ使いやすくていいんじゃねーの?
適材適所。
385ご冗談でしょう?名無しさん:03/11/18 13:51 ID:???
放射能を浴びるっていうのは、235Uなどの粉末をドバッっとかぶる
って意味で合ってますか?
386ご冗談でしょう?名無しさん:03/11/18 13:57 ID:???
>>385
放射能を浴びるという言い方は誤用
浴びるのは放射線
387ご冗談でしょう?名無しさん:03/11/18 14:04 ID:???
>>383
本当に読んだか? 古典力学の法則に従って運動しているのに電磁波を出さないんだぞ。
ボーア理論は水と油をくっつけているんだ。

適用範囲の「異常な」狭さと書いてあるだろ。
388ご冗談でしょう?名無しさん:03/11/18 14:15 ID:???
>>383は「原理」といえばなんでも許されると思っているみたいだね。
自己完結した理論であるためには、複数の原理間に矛盾があってはいけないのよ。
389ご冗談でしょう?名無しさん:03/11/18 14:39 ID:Ggp9hRtN
てか>>290の言い分で言えば、
現存するすべての物理理論は不完全なんだが。

もちろん、都合の悪い部分をすべて「原理」として棚上げし、
かつ適用範囲をしぼりこめば、いくらでも「論理的に」完全な理論は作れる。
そういう意味ではボーアの理論だって完全なものにすることはできる。

しかし問題は、そうやって完全な理論をいくつか作ったときに、
異なる理論のもつ「原理」が、たがいに矛盾することがありうる、ってことだ。
だが、これはそれぞれの理論が完全かどうかということには直接関係ない。
理論の間で矛盾があっても、個々の理論は完全でありうる。

つまり、物理の理論を作るときに、その理論の内部で完結しているのはもちろん望ましいが、
それと同時に(もしくはそれ以上に)、他の理論との整合性が重要だということだ。
要するに、ボーア理論が完全か完全じゃないかなんていう言いあいはどうしようもなく不毛だということ。
そもそも「完全」って言葉の意味もよくわからんままで低レベルな言いあいしてんじゃねーよ。
390ご冗談でしょう?名無しさん:03/11/18 14:46 ID:???
>>389
もともとは厳密に正しくない理論じゃないと物理が納得できないという怠け者からはじまったんだけどね

そんなこといったら化学や生物なんかやってらんないよな
391ご冗談でしょう?名無しさん:03/11/18 14:47 ID:???
みなさん、>>389はそっとしといてあげましょうね。
392ご冗談でしょう?名無しさん:03/11/18 16:21 ID:AtXlwz2B
クオークって何ですか?
393ご冗談でしょう?名無しさん:03/11/18 16:27 ID:+7pVZRSF
陽子中性子を構成しているのもある。
394ご冗談でしょう?名無しさん:03/11/18 21:16 ID:???
>>392
「クオークはあなたのための信販会社です。」だそうでつ
http://www.quoq.co.jp/
395ご冗談でしょう?名無しさん:03/11/18 21:54 ID:???
>>387
適用範囲が広かろうが狭かろうが、その範囲内での話をしている。
狭い広いは他と比較して初めて言えること。自己完結してるか
どうかの判断に他との比較を持ち込む理由はない。

そもそも、何をもって異常とするの? そのような主観的基準を持ち込んで
いいなら好きなように完全と不完全の線を引けるのは同意するよ。
そんな線引き、何の意味もないけどね。

>>389
>現存するすべての物理理論は不完全なんだが。
だからニュートンも不完全と主張するならわかると何度も言ってるに

>そもそも「完全」って言葉の意味もよくわからんままで低レベルな言いあいしてんじゃねーよ。
漏れは「完全」をどういう意味で使ったか明らかにし、首尾一貫してその意味に沿って
主張をしている。低レベルなのは、後出しジャンケンのように資意的とか適用範囲が
狭いとか、次々と条件を継ぎ足して取り繕おうとする姿勢だろ。

まぁ、どうしようもなく不毛だというのは同意するのでもうやめる
396ご冗談でしょう?名無しさん:03/11/18 22:17 ID:???
>>395>>387の前半と>>388には全く反論できないのね。
397ご冗談でしょう?名無しさん:03/11/18 22:28 ID:???
>>396
ボーア理論では電子は古典力学の法則に従って運動などしてない。
何のための量子化条件だ? そんな勘違いしてるから複数の原理間に
矛盾があるなどという主張になる。

これ以上続けたいなら、人の主張に文句付ける前にまずニュートンとボーアの
自然な線引きをしてみせれ。でなきゃ無視する。

これまでのおまいの線引きこそ資意的で適用範囲狭過ぎ(w
398ご冗談でしょう?名無しさん:03/11/19 00:50 ID:???
>>397
「定常状態において電子は通常の古典力学の法則に従って運動する」
ボーア理論のイロハのイも知らないとは・・・やめた。
399ご冗談でしょう?名無しさん:03/11/19 00:52 ID:+/Eebaun
400ご冗談でしょう?名無しさん:03/11/19 01:06 ID:TVeMOysO
400だし
401ご冗談でしょう?名無しさん:03/11/19 01:10 ID:???
>>399
上の電灯で生じる対流、ってマジレスできるのは、
きっと今酔っ払ってるせいだな。
402ご冗談でしょう?名無しさん:03/11/19 05:19 ID:???
>>401
良かったね。酔っ払ってて。
403ご冗談でしょう?名無しさん:03/11/19 08:39 ID:???
>>398
自分で「定常状態において」と書いておきながらその意味もわかってないのね。
話が通じなかったわけだ。これ以上意見をすり合わせる余地はないことが
はっきりわかったよ。じゃね
404ご冗談でしょう?名無しさん:03/11/19 08:42 ID:???
おまいら他所でやれ
405ご冗談でしょう?名無しさん:03/11/19 12:15 ID:LLma6xCP
他でやれー、第3者にあまり有益な討論とは思えないぽ
406259:03/11/19 13:32 ID:ECWe7ATS
>>256
ミリカンの話。
ご冗談でしょうファインマンさんの最後の章に載ってた。
407ご冗談でしょう?名無しさん:03/11/19 14:40 ID:Eu9GSJGD
ふと思ったのですが、位置エネルギーは何故「U」の文字で表すのでしょうか。?
408ご冗談でしょう?名無しさん:03/11/19 14:47 ID:M3JF70Sj
>>407
運動エネルギーをTで表すのは不思議に思わないのか?
409407:03/11/19 15:15 ID:Eu9GSJGD
>>408

はい、それも不思議に思います。

もしかしてテキトーに決められたものなのでしょうか?
410ご冗談でしょう?名無しさん:03/11/19 16:16 ID:XIVJ462b
理論物理を学ぶにはどこの国立大がいいっすかね?
411ご冗談でしょう?名無しさん:03/11/19 16:28 ID:???
ビックバンって何ですか?
412ご冗談でしょう?名無しさん:03/11/19 16:34 ID:???
もっと物理っぽい疑問持て
413412:03/11/19 16:37 ID:???
>>412
厨房的な質問でスマソ
414ご冗談でしょう?名無しさん:03/11/19 16:41 ID:???
>>410
理論物理っつっても範囲広過ぎ
415AJへぇ〜 ◆2chDQNAJHE :03/11/19 17:15 ID:PAwGwMPI
電磁気相対論etc
416ご冗談でしょう?名無しさん:03/11/19 17:16 ID:???
>>407 >>408
電圧、電位がVだから、その隣のUを使ったのではないか?
で、運動エネルギーはさらにその隣のTにした?
417ご冗談でしょう?名無しさん:03/11/19 17:40 ID:???
ちょっと質問させてください。

朝鮮半島はどういった経緯で南北に分けられることになったのでしょうか?
常識なのかもしれませんが、私は知らなかったので
どうかご教授ください。お願いします。
418ご冗談でしょう?名無しさん:03/11/19 17:43 ID:???
板違い
419ご冗談でしょう?名無しさん:03/11/19 17:43 ID:M3JF70Sj
>>417
その質問をなぜ物理板でしようと思ったのか
その動機を詳しく説明してくれたら答えてやってもいい
420ご冗談でしょう?名無しさん:03/11/19 18:35 ID:???
>>418 >>419
すいません。物理と分離を間違えました
421ご冗談でしょう?名無しさん:03/11/19 18:48 ID:???
ネタふりは面白いのを頼む
422ご冗談でしょう?名無しさん:03/11/19 21:35 ID:???
一人の人間を物理で証明ってできますか?
423ご冗談でしょう?名無しさん:03/11/19 21:38 ID:???
>>422
観測者が他にいて観測結果に再現性がないとダメです。
424ご冗談でしょう?名無しさん:03/11/19 22:42 ID:MPmzJApc
>>358のように、「光速に近い物体」とよく言うけど、

秒速29万qで移動する物体が仮にあったとしても、
その物体から観測できる光のスピードも秒速約30万qと変わらないんだったら、
どんな物質も光速に近づけないんじゃないの?

あ、それだと、秒速40万qで走る物質も光の速度を超えられないね。
何で?
425ご冗談でしょう?名無しさん:03/11/19 22:53 ID:???
激しく勘違いしているようだから教えてやると、
「光速」は速度の目安。「時速60km」と一緒だ。君が乗った車から、
常に時速60kmでボールを打ち出していたとしても、「時速60km」は出せるだろ?

>あ、それだと、秒速40万qで走る物質も光の速度を超えられないね。
>何で?
何でもこんでもそれが相対論からの帰結。
超光速物体は一生超光速のまま。光速度の壁は何人も越えられない。
426ご冗談でしょう?名無しさん:03/11/19 22:54 ID:I0MWIMyb
電子と同じ速さで動いている観測系での温度はゼロ?
このときの電子の速度がゼロ、よって運動エネルギーもゼロ
よって温度もゼロ
427ご冗談でしょう?名無しさん:03/11/19 22:58 ID:???
>>426
特殊相対論かじったら、次は量子論かじりましょう。
428ご冗談でしょう?名無しさん:03/11/20 00:36 ID:???
スレの趣旨とは違ってくるかもしれないのですが他に書くところがないのでここに書きます

今地帝のB3なんですが受けてる授業聞いててもサパーリわかりません
一応単位は全て取れているんですが教科書読んでも式追うのが精一杯で
それ以上の物理的考え方とかも全然わからなくて…
でもTAの院生を見てるとみんな教科書程度の内容はかなり理解しているように見えます
正直このまま院に上がってもかなり足手まといになるだけだと思うんですが
大学院に行けば研究活動や院試を通して
それなりの努力があれば学部でやった初歩的なことがわかるようになるんでしょうか?
429ご冗談でしょう?名無しさん:03/11/20 00:58 ID:???
>>428
院行ったら学部時代のことが理解できるかどうかより大変なことがたくさんあるよ。
学部内容の理解度など気にしているひまもない

ただ別に足手まといにはならないと思うぞ。
4年生や院に上がったばかりの人間に研究室の業績がたくされることなんかまず無いから
指導教官や先輩が多少いらつくくらいだよ
430ご冗談でしょう?名無しさん:03/11/20 01:11 ID:MOWy1IIe
干渉板として使用されているガラス板と普通のガラス板の違いはなんですか?
431ご冗談でしょう?名無しさん:03/11/20 01:22 ID:???
干渉板ってどんな干渉で何をみるもののこと?
俺、光学の実験やってるんだがいくつも思いあたって
どれを言いたいのかよくわからん
432ご冗談でしょう?名無しさん:03/11/20 09:38 ID:???
>>427
よく噛まないとお腹こわすので注意
433ご冗談でしょう?名無しさん:03/11/20 12:02 ID:???
誘電体表面における光の反射の際の位相のずれについてはどの教科書にも載っていますが、
金属表面での反射では、位相はどうなるのですか?
434ご冗談でしょう?名無しさん:03/11/20 18:19 ID:BT2AEDRY
一様なまっすぐな棒をその両端につけた糸でぶら下げる。
糸と水平の作る角がα、β(α>β)である。
このとき棒と水平の作る角が次のようになる。

tanθ=1/2 * (tanβ-tanα)

これを示すにはどうすればいいのですか?
435ご冗談でしょう?名無しさん:03/11/20 18:54 ID:i5oAXXv+
すいません、物理ではなく物理の本についての質問なのですが、
ファインマン物理学の特別記念版というのは原著ですか?
表紙の写真をみると、日本語でもタイトルが書いてあるので
どっちかなと迷ってしまうので注文できないのです。
原著なら買うつもりなのですが・・。
持っている方は教えてください。
436ご冗談でしょう?名無しさん:03/11/20 19:05 ID:???
>>434
それだけの条件では決められないと思うけど
437434:03/11/20 19:12 ID:BT2AEDRY
>>436
棒の重さや長さなど
必要に応じて様々な値を定義してください。
どうぞ、よろしくお願いします。
438AJへぇ〜 ◆2chDQNAJHE :03/11/20 19:15 ID:fqCqHffr
↑IDかこいい
439ご冗談でしょう?名無しさん:03/11/20 19:19 ID:???
>>437
丸投げかよ。>>434が成り立つような条件を自分で探してみ?
440434:03/11/20 19:22 ID:BT2AEDRY
丸投げと言われれば・・・確かにそうですね。

角運動量を利用して解くのでしょうか?
441ご冗談でしょう?名無しさん:03/11/20 20:10 ID:???
>>440
 力とトルクの釣り合い。後はがんばれ。
442ご冗談でしょう?名無しさん:03/11/20 20:13 ID:GesXXQB4
暇だから質問に着てやろう。
443434:03/11/20 20:41 ID:BT2AEDRY
わっかんねー
444ご冗談でしょう?名無しさん:03/11/20 21:12 ID:lOmBtYae
観測系によって速度が違うということは物体の温度も違う?
445ご冗談でしょう?名無しさん:03/11/20 21:18 ID:???
お前まだいたのか。一晩寝てもう一度考えれ
446ご冗談でしょう?名無しさん:03/11/20 21:22 ID:???
>>444
ミクロとマクロ。

あー、でもちょっと俺も思ったんだが、
背景放射を3゜Kと観測する系とは、みたいな。
447ご冗談でしょう?名無しさん:03/11/20 21:47 ID:lOmBtYae
おれはいつも一晩中考えているんだけどな
448ご冗談でしょう?名無しさん:03/11/20 21:59 ID:???
>>444
異なる観測系でどのような手段で温度を計測するの?
449ご冗談でしょう?名無しさん:03/11/20 22:02 ID:???
>>448
摩擦熱で
450ご冗談でしょう?名無しさん:03/11/20 22:07 ID:???
>>449
質問スレでボケるのは無しね
451ご冗談でしょう?名無しさん:03/11/20 22:15 ID:lOmBtYae
>>448
温度の計測方法は分かんないけどけど、運動エネルギーが観測系で
違うんだから、温度も違うのかな思って。
452ご冗談でしょう?名無しさん:03/11/20 22:17 ID:lOmBtYae
>温度の計測方法は分かんないけどけど ×
>温度の計測方法は分かんないけど   ○
453ご冗談でしょう?名無しさん:03/11/20 22:21 ID:???
質量みたいな示量変数が観測系で変わるのなら、

エントロピーと内部エネルギーが変わって
温度の方は同じになるんじゃないかなぁ?

推測だけど。
454ご冗談でしょう?名無しさん:03/11/20 22:44 ID:???
>>435
原著ってか、あれは、ファインマンの生講義を、他の人が活字にしたものだからな。
生にこだわるなら、講義の生音のCD-ROMが出てる。
http://www.amazon.co.jp/exec/obidos/ASIN/0201408252/qid=1069335720/sr=1-15/ref=sr_1_2_15/249-9079748-7537954
ただし、雑音が多いから、物理学の講義録としてはお勧めできない。
あくまでファインマンを感じるためのもの。
455ご冗談でしょう?名無しさん:03/11/20 22:59 ID:BcF9r5KK
あははーーーー、来年にはアポロ11号の月面着陸が捏造であることを
証明して差し上げますよ。

震えて眠れ旧新進党信者ども!!

456ご冗談でしょう?名無しさん:03/11/20 23:00 ID:???
>>455
誤爆ですか?
457ご冗談でしょう?名無しさん:03/11/20 23:01 ID:8kF5XZ82
>>454
本文は英語なんですね?
日本語訳のだったら買う意味がないんで・・。持ってるし。

別に生にこだわっているわけでは有りません。
まあ物理好きの趣味みたいなもんですよ。
458ご冗談でしょう?名無しさん:03/11/20 23:45 ID:???
>>457
表紙に日本語書いてあったら中身も日本語の可能性高いと思うなぁ。帯ならともかく。

確実にファインマン物理の原著が欲しいなら
http://www.amazon.co.jp/exec/obidos/ASIN/0201021153/ref=pd_ecc_rvi_f/250-9019056-1293026
特別記念版とやらは知らない
459ご冗談でしょう?名無しさん:03/11/21 00:37 ID:???
一般生活上の法律問題でお困りの方はこちらのスレにご相談ください。

やさしい法律相談その57【相談者はその1を読んで】
http://school.2ch.net/test/read.cgi/shikaku/1068898156/l50
460ご冗談でしょう?名無しさん:03/11/21 00:52 ID:???
>>451
運動エネルギーがそのまま温度に寄与するわけではない
461ご冗談でしょう?名無しさん:03/11/21 07:30 ID:I9UjYIml
扇風機などの風にあたると涼しく感じるのはなぜですか?
風によって感じる温度は変わるのでしょうか?
462ご冗談でしょう?名無しさん:03/11/21 07:50 ID:???
>>461
「体感温度」でぐぐってみ?
463ご冗談でしょう?名無しさん:03/11/21 08:05 ID:???
>>461
@体表面の水分が気化して熱を奪うため
A体の周囲の温まった空気が逃げるため
の二点。・・だと思う。
464ご冗談でしょう?名無しさん:03/11/21 09:36 ID:???
でもそれだけじゃ自然の風と扇風機の風とが
人体に同じ影響しか与えていないように聞こえる。
465ご冗談でしょう?名無しさん:03/11/21 10:06 ID:???
違うの?
466ご冗談でしょう?名無しさん:03/11/21 10:31 ID:qz120l/a
なぜ重力は時間の流れを遅くするの?
467ご冗談でしょう?名無しさん:03/11/21 10:35 ID:???
誰がそんな事言ったの?
468ご冗談でしょう?名無しさん:03/11/21 13:19 ID:???
アインシュタイン
469ご冗談でしょう?名無しさん :03/11/21 13:28 ID:BKcb5aTs
>>461
水分が気化して熱を奪うのを体感してみたかったら、
ぬれたタオルにジュース包んで、タオルの端もってブンブン回すと
缶ジュースがひんやり!
470ご冗談でしょう?名無しさん:03/11/21 14:41 ID:???
注射前のアルコール消毒でもいいけどな
471ご冗談でしょう?名無しさん:03/11/21 16:36 ID:vBq50BYn
この世界は何次元なのですか?
472ご冗談でしょう?名無しさん:03/11/21 16:53 ID:q2Ejc9N2
3次元だべさ
473ご冗談でしょう?名無しさん:03/11/21 17:30 ID:???
時間も入れて4次元
474ご冗談でしょう?名無しさん:03/11/21 18:09 ID:???
無限次元の位相空間でつ
475ご冗談でしょう?名無しさん:03/11/21 19:47 ID:rmbXx5dW
E=mc^2の意味がわからん。何で光速が関係あるの?
476ご冗談でしょう?名無しさん:03/11/21 20:21 ID:???
相対論
477ご冗談でしょう?名無しさん:03/11/21 21:18 ID:???
>>475
なぜなぜ坊やに一言で説明できる話じゃない。
478ご冗談でしょう?名無しさん:03/11/21 21:29 ID:rmbXx5dW
>>477
十行以内で説明お願い。
479ご冗談でしょう?名無しさん:03/11/21 22:12 ID:???
物理でルベーグ積分が必要になる場合はどのようなときでしょうか?
厨な質問ですいませんがお願いします。
480ご冗談でしょう?名無しさん:03/11/21 22:14 ID:???
>>434,440
角運動量は使わない…つうか運動してないじゃん。

>>441の通り、モーメントと力の釣り合いで出ると思うんだが…
俺もさっきまで計算してみたが、tanθを独立変数にとった二次方程式が出て来て
やる気が失せた(多分定義前後で間違った)
481ご冗談でしょう?名無しさん:03/11/21 22:20 ID:???
>>475
ローレンツ変換にcが入っているから
482461:03/11/21 22:46 ID:???
質問に答えてくださった皆さんありがとうございました。

>>469
おもしろそうですね、今度やってみたいと思います
483ご冗談でしょう?名無しさん:03/11/21 22:53 ID:???
>>479をお願いしまつ、、、
484ご冗談でしょう?名無しさん:03/11/21 23:06 ID:???
>>483
答えになっていないかもしれないけど、使ったことはありませんが。
おそらく、数学科の授業を受けていない限り、知っている人も少ないのでは・・・
485ご冗談でしょう?名無しさん:03/11/21 23:16 ID:???
>>484
では物理をやっている限りでは知らなくてもいいということですか?
486ご冗談でしょう?名無しさん:03/11/21 23:46 ID:???
ルベーグ積分は統計学にくるまれて必要になるとおもう。
統計学を極める気が無いなら、多分どうでもいい。
487ご冗談でしょう?名無しさん:03/11/21 23:48 ID:???
>>473
筒井康隆が、
3次元とは幅、奥行き、高さ
4次元はそれに低さが加わる
と書いていた。
488ご冗談でしょう?名無しさん:03/11/21 23:52 ID:???
安さだろ?
489ご冗談でしょう?名無しさん:03/11/22 00:36 ID:???
>>472
宇宙は有限で端がないから3次元をねじまげた4次元かそれ以上なんじゃない?
490高校生:03/11/22 00:58 ID:4NtYNUWO
「時間も何もない無になる」って言うこと良く聞くけどどういうこと?本当に無ってあるの?
無になってる時間が5秒あったとか言うふうには考えれないのかなぁ?
もっと深い考えがあるんだろうか?
491ご冗談でしょう?名無しさん:03/11/22 01:11 ID:???
>>486
岩波の現代物理学の基礎の量子力学にルベーグ積分がでてきてるんですが、、
492ご冗談でしょう?名無しさん:03/11/22 01:52 ID:???
>>489
おまえ、中1のときに「りんごがマイナス一個なんて変じゃないかぁ」
ってダダこねて先生を困らせたタイプだろ?
493492:03/11/22 01:53 ID:???
ずれた。>>490
494高校生:03/11/22 02:17 ID:4NtYNUWO
>>492
りんごがマイナスいっこてどういうことですか??
495ご冗談でしょう?名無しさん:03/11/22 03:25 ID:???
ダダこねて先生を困らせるな
496ご冗談でしょう?名無しさん:03/11/22 03:30 ID:???
>>491
おまいは統計学と量子力学との間に何の関係も無いと思ってるのか?
ヒルベルト空間マスターになりたければ、学んどけ。必要だ。
497ご冗談でしょう?名無しさん:03/11/22 10:29 ID:yq3sCvEA
体積を一定にして、温度を変化させたときの、
体系がした仕事量って0でいいんだよね?
498ご冗談でしょう?名無しさん:03/11/23 00:57 ID:???
>>496
ありがとうございます。
ルベーグ積分の入門書は何が良いでしょうか?
499ご冗談でしょう?名無しさん:03/11/23 13:14 ID:???
木村拓哉のドラマでの飛行機操縦士役の効果で
操縦士志望者がやたら増えたときく。
ではトリックの阿部寛の物理学者役によって物理学者ブームは来るのかどうか?
個人的には木村より阿部のほうがかっこいいと思うのだが。
500ご冗談でしょう?名無しさん:03/11/23 13:47 ID:???
俳優がかっこいいかより、役柄がかっこいいかどうかのほうが。
そういう意味ではトリックの物理学者は基本的にダメキャラの
設定なのでイマイチ
501ご冗談でしょう?名無しさん:03/11/24 01:21 ID:6VNmisny
(A*sin(wt-kx))+(B*sin(wt+kx+d))

上式をXsinYの形に直すことってできるの?
502ご冗談でしょう?名無しさん:03/11/24 03:42 ID:Efhi3tVW
>>501
 そりゃ何ぼでもかけるがw
 ホントに聞きたいのはXsin(wt+α)の形(Xとαはtに対して定数)にできるか、
つーことだろ?できるよ。

 Asinθ+Bsin(θ+δ)=Csin(θ+γ)にする公式が高校の教科書にもあるだろ?
見つからなけりゃ両辺をsinθとcosθの多項式に整理して恒等式作ってみ。
503ご冗談でしょう?名無しさん:03/11/24 04:32 ID:???
>>497
はい、ゼロです
あとそういう時は「体系」ではなく「系」のした仕事、と言います
504ご冗談でしょう?名無しさん:03/11/24 10:32 ID:Bj/GI3b0
[トムソンの原子模型]
原子は一様に分布している正電荷Qの、半径aの球から成り立っていて、
内部に負電荷の電子が一様に組み込まれている。
このとき、正と負の電荷の量は等しくて、原子の全電荷は0である。

[簡略化したラザフォード原子模型]
原子はほとんど空であり、正電荷Qを持つ原子核を中心として
負電荷を持つ電子-Qが原子核からの距離がaの球面上を回っている。

この2つの模型で、球の中心を原点Oとして、原点からの距離をrとしたとき、
0≦r≦a と a < r における電場をrの関数として求めよ。


この問題のガウスの法則を用いて求めたいのですが、
左辺はOを中心とした半径rの球面にガウスの法則を使えばよさそうなんですけど、
右辺をどう考えればいいか分かりません・・・教えていただけませんでしょうか?
505ご冗談でしょう?名無しさん:03/11/24 11:09 ID:???
>>504
我々は超能力者では無いので、君の頭の中の右辺と左辺を知ることは
出来ない。
506ご冗談でしょう?名無しさん:03/11/24 13:25 ID:???
>>505
申し訳ない。
∫[S] En dS = q /ε の、左辺と右辺の扱いでして・・・・。
507ご冗談でしょう?名無しさん:03/11/24 13:41 ID:???
>>504
半径rの電場E(r)×半径rの球殻の表面積=半径rの球殻の内部の電荷/ε_o
右辺は0<=r<=aとa<rで場合分け,トムソンは(半径aとrの球の体積比)×Qだろ
508ご冗談でしょう?名無しさん:03/11/24 15:05 ID:uZByNgBc
はじめまして FELを調べてたら
ウイグラーてでてきました・・・
教えてください。
509ご冗談でしょう?名無しさん:03/11/24 15:09 ID:???
>>507
ありがとうございます。


ラザフォードの場合はどうかんがえればいいのでしょうか?
510ご冗談でしょう?名無しさん:03/11/24 15:58 ID:???
>>508
馬鹿じゃねーの?FELを調べてたら普通に出てくるだろ
511ご冗談でしょう?名無しさん:03/11/24 16:37 ID:6VNmisny
>>502

>sinθとcosθの多項式に整理

すいません、具体的にはどうするんですかね?
どうがんばっても、うまく整理できないんですが…。

512ご冗談でしょう?名無しさん:03/11/24 17:06 ID:???
鉄の塊1dと綿の塊1d。重いのはどっちですか?


同じですよね?
513ご冗談でしょう?名無しさん:03/11/24 17:08 ID:???
空気の重さはどうしますか?
514ご冗談でしょう?名無しさん:03/11/24 17:28 ID:???
>>509
同じように、半径rの球殻の内部の電荷はいくらか、を考えるだけだろ
515ご冗談でしょう?名無しさん:03/11/24 18:48 ID:???
>>513
釣りなら釣りと断って下さいね
516513:03/11/24 20:01 ID:???
は?
517ご冗談でしょう?名無しさん:03/11/24 20:04 ID:???
>>515
むしろ、>>512にふさわしいせりふですな。
「塊」「重い」の語を使ってるから、案外マジかも。
518ご冗談でしょう?名無しさん:03/11/24 20:05 ID:???
む?
519ご冗談でしょう?名無しさん:03/11/24 20:07 ID:???
え?
520502:03/11/24 21:54 ID:???
>>511
 三角関数の加法公式のことなんだが。これがわからんとなればちとお手上げだなぁ。
521ご冗談でしょう?名無しさん:03/11/24 23:37 ID:avqx9abC
格子定数dとθの関係式
dsinθ=mλ m=0、±1、・・・
で、次数の高い(mの値を大きくする)回折像から求めたdの方が、正しい値に近づくのはなぜですか??
522ご冗談でしょう?名無しさん:03/11/25 00:13 ID:???
例えば100円硬貨の厚みを測る場合、1枚だけを測るのと、
10枚重ねて測定値を1/10倍するのとどちらが精確な値を得られるでしょうか。
(測定器、例えばノギスの精度が一定の場合)

う〜ん我ながら分かりにくいたとえだ死のう…
523ご冗談でしょう?名無しさん:03/11/25 04:15 ID:???
>>521
マルチいくない
524ご冗談でしょう?名無しさん:03/11/25 04:16 ID:???
>>522
実際に誤差を計算してみなよ
525ご冗談でしょう?名無しさん:03/11/25 05:06 ID:???
>>504は負電荷の量も考慮しなきゃいけませんよね?
だとしたら、トムソンの 0≦r≦a と a < r の右辺は両方とも0に
なるような気が・・・・・・・
526ご冗談でしょう?名無しさん:03/11/25 05:09 ID:???
文章省略しすぎた、
トムソンの 0≦r≦a と a < r における、ガウスの法則の右辺は
両方とも0になるような気が・・・・・・・
ですね、スマソ
527ご冗談でしょう?名無しさん:03/11/25 05:10 ID:???
>>524みたいのが出るくらいわかりにくいたとえだったみたいね>>522
528ご冗談でしょう?名無しさん:03/11/25 05:14 ID:???
>>525
なぜ0≦r≦aで0?
529ご冗談でしょう?名無しさん:03/11/25 05:18 ID:???
一様に分布してる正電荷に、一様に分布する電子が埋め込まれているって
書いてあるので・・・この場合の電子の扱いが良く分からないのです。
じゃあ、>>507氏の考えでよろしいのですか?
530528:03/11/25 06:16 ID:???
>>529
すまん。>>509からの流れでラザフォードについて聞いていると勘違いした
531ご冗談でしょう?名無しさん:03/11/25 06:32 ID:???
>>530
しばらく考えたけど、わからない・・・
トムソンは正電荷の中にある電子の電荷の和を
考慮しなければならないのでしょうけど・・・求め方が・・・
特に問題で指定がないから考える必要はないとかだったりして・・・
ラザフォードは、負電荷-Qと正電荷Qの電場ベクトルの総和を
考えなければならないのだけど、
これまた、求め方が・・・・わからないのですよ。
532ご冗談でしょう?名無しさん:03/11/25 07:04 ID:???
>>531
電子も正電荷と同様に連続的に一様分布してるとするなら0≦r≦aでも0で正しい。
ただ、トムソン模型といった場合は、電子は点状でレーズンパンのレーズンのように
正電荷の中に埋め込まれているとするので、その場合だと局所的には0でなくなる。
点状の電子の分布を与えれば原理的には電場を計算できるけど、簡単ではない。

ラザフォードのほうは何も考えずに半径rの球の中の電荷をカウントすればいいだけ
533ご冗談でしょう?名無しさん:03/11/25 07:29 ID:???
・トムソンの原子模型の電荷量
0≦r≦a : 0?(題意から負電荷の扱いをどうすればいいか分からないので)
a < r   : 0(正電荷kと負電荷の和が0であることから)
・簡略化したラザフォード原子模型]の電荷量
0≦r≦a のとき、 Q /4πε_o*r^2 (r内には正電荷Qが1つ)
a < r   のとき、 0? (r内には正電荷Qと負電荷-Qがひとつづつ)

こんなんでいいのでしょうか?
534ご冗談でしょう?名無しさん:03/11/25 09:09 ID:???
いいんじゃないの
535ご冗談でしょう?名無しさん:03/11/25 09:13 ID:hwgw62vK
数学科のものですが、将来的には建築関係の実家を継ぎたいと思っています。
しかし、大学、大学院で学んだ数学の知識やこれから学ぶであろう
研究の仕方、経験を生かして建築の会社を継ぎたいと考えてます。
何かいい考えはないでしょうか?また代数などやってても工学系にはやくだたないと思います。
やはり解析関係にウエイトをおいて勉強すべきなんでしょうか?
数学はいくら好きでも、医学や数学以外の理系科目、法学などと違い
それを生かした職業が少なすぎますし、院を卒業してもきょうしといったもの足りない
職業しかないので困ります。また会社経営をしながら個人塾の経営をしたいと考えています。
536ご冗談でしょう?名無しさん:03/11/25 09:27 ID:???
建築関係でどんな知識が要るのか、実家に聞いてみれ
537ご冗談でしょう?名無しさん:03/11/25 09:35 ID:GsVwqvdx
現在の建築をやっていく上で必要な知識だけでは、そこらの企業と差がでない。
だから数学の知識をつかって頭1つでたい
538ご冗談でしょう?名無しさん:03/11/25 09:58 ID:???
大学の知識を社会に生かすのは、はっきり言って難しい(レベルが違い過ぎる)
むしろ学習・研究の方法論を役立てるべきだ。
539ご冗談でしょう?名無しさん:03/11/25 10:27 ID:SncBis6P
>>538
アドバイスありがとうございます。
確かにおっしゃるとおりですね。では院へいって具体的に研究を
1度してみるのが良いのでしょうか?具体的なアドバイスをお願いします。
540ご冗談でしょう?名無しさん:03/11/25 11:53 ID:3HgMCZJT
大学生なんですが現在やっている実験の内容を知るには
どうしたらいいでしょう?又は現在どの様な理論が出ているかを
大学生で知るにはどうしたらいいですか?
(内容が理解出来る出来ないという事は抜きでお願いします)
541ご冗談でしょう?名無しさん:03/11/25 12:04 ID:???
>>540
ttp://prl.aps.org/
で勉強しる
542ご冗談でしょう?名無しさん:03/11/25 12:05 ID:???
>>540
君の訊いているのは、「料理のレシピを知りたいんだけど、どうしたら良いでしょう?」
というのと大して変わらない。
もっとジャンルを絞りこんでくれ。
543540:03/11/25 13:23 ID:3HgMCZJT
出来ればいろいろなジャンルのを知りたいのですが
例えば学会で公表された論文などをただの大学生が
見る事は出来ないでしょうか?
544ご冗談でしょう?名無しさん:03/11/25 13:39 ID:???
>>543
大学の図書館で学術雑誌を探せ。

つうより、まともな理系の大学生なら当該分野の雑誌を調べずに卒業することのほうがありえないよ。
545540:03/11/25 13:44 ID:3HgMCZJT
学術雑誌を調べてみます。どうもありがとうございます
546ご冗談でしょう?名無しさん:03/11/25 14:56 ID:3nJ0thIC
とっておきの情報です!ヤフーオークションの検索に

バブル崩壊!

と入れると凄い情報が見られます。
友達から聞き早速私も落札してしまいました!
落札しなくても見られる無料情報もありましたよ!!もちろん中身はもっとすごい情報でした。
とりあえずウォッチリストに入れておけばいつでも見られますね。
ただすごい人気なので、あっという間に落札されてしまうので早いもの勝ちですが・・・。
出品者の方も迅速で安心できる方でした。かなりお薦めです!

http://auctions.yahoo.co.jp/
547ご冗談でしょう?名無しさん:03/11/25 14:57 ID:nXg9yO9J
(y+δy)ln(y+δy)-ylny=δy(lny+1) において
|δy|<<y を証明せよと言う問題なのですが、
右辺と左辺が=になるのが良く解りません。
どなたか教えていただけないでしょうか。お願いします。


548ご冗談でしょう?名無しさん:03/11/25 15:03 ID:7UuKp82U
遂にみつけましたね!
※リクエストにお応えしてバラ売りも復活しました!

「片手間?NO!」「ほったらかし」等の高額(数十万円分)情報セットを格安で販売致します。
同じ情報にあなたは幾ら払いますか?
ここ最近、情報の値段が変動していることに気づいたあなたはもうお分かりですよね?
そうです、バブルが崩壊したのです。
とにかくもう何十万円も出す必要はありません!!500円即決早い者勝ち!
かなりの情報量です!!
たった500円であなたの人生観が変わります…

↑やってみたらこんなんでました
549ご冗談でしょう?名無しさん:03/11/25 15:48 ID:qf34NvYb
I=1/2の核スピン系における300Kでの低エネルギー(占有率N1)および高エネルギー
(占有率N2)準位の占有比R(=N2/N1)はいくらか?

ぜんぜん分からないのですけどどなたかお願いします。
550ご冗談でしょう?名無しさん:03/11/25 17:11 ID:CgmKMCJF
サイド7の人口重力地面において、ザクがジャンプしたらどうなりますか?
ちゃんと地面に戻って来れますか?
それともコロニー中心の無重力ポイントに吸い寄せられてしまいますか?
551ご冗談でしょう?名無しさん:03/11/25 17:28 ID:???
552497:03/11/25 17:40 ID:qjOExqcQ
>>503

レスさんくす。やっぱそうだよね。
いや、宿題の中にそういう問題があってさ…、ちと不安になったので聞いてみた次第。

※激しく遅レススマソ
553501:03/11/25 17:56 ID:qjOExqcQ
>>520

すいません、ちょっと勘違いしていたようです。
試行錯誤してみた結果こうなったのですが↓
http://ec.uuhp.com/~physics2/cgi-bin/upload250/NeoKiffg/siki.gif
これであってますかね?(これ以上整理できますか?)

それから、この式のγっていうのは、逆三角関数を使わずに書くことは
できないですよね?
554ご冗談でしょう?名無しさん:03/11/25 21:16 ID:wzPX+xwZ
555520:03/11/26 01:41 ID:???
>>553
 いいと思うぞ。これで何を説明or理解しようとしてるのかはよくわからんが。
 ちなみにγはtanγにした方がやや形が簡単になるな。
556ご冗談でしょう?名無しさん:03/11/26 09:56 ID:???
>>540
>>541は広告じゃないね
557501:03/11/26 13:34 ID:dElNLlEU
>>555

確認どうもです。

実際の問題としては、

・固定端での反射波の位相がπずれること
・反射波の振幅は入射波の振幅と同じになる

上記二つを説明する問題だったのですが。

この合成波の式が出せれば、あとの説明は容易に出来ると思ったので…
この形になれば説明できそうですね。(振幅の値が0になる場合を考えることによって)

ありがとうございました。
558カリン ◆NCKvTBmrj. :03/11/26 15:53 ID:???
カリン。
559ご冗談でしょう?名無しさん:03/11/26 18:43 ID:L/bXgcU4
水平と角度αをなす斜面の下端から初速度↑v_0(大きさv_0)、
仰角β(>α)で斜め上方に投げられた質量mの粒子の斜面上への到達距離がR=2v_0^2sin(β-α)cosβ/gcosαであり、
その最大到達距離がR_max=v_0^2/g(1+cosα)であることを示せ。

という問題なんですが到達距離は求まったんですが、最大到達距離がどうしても求められません。
dR/dβ=0を満たすβをRに代入したものがR_maxなのでしょうか?
560ご冗談でしょう?名無しさん:03/11/26 19:03 ID:???
>>559
そうですな。
561559:03/11/26 20:23 ID:twdlSFnF
>>560
あ、合ってるんですか。
それで計算してみたんですが、そのdR/dβが
2v_0^2[-sinβ{sin(β-α)}+cosβ{cos(β-α)}]/g(cosα)^2
となって、加法定理を使っても
2v_0^2[cosα{(cosβ)^2-(sinβ)^2}+2sinαsinβcosβ]/g(cosα)^2
となって、うまくβを抽出できないんですよ。
どうしたらいいんでしょうか・・・?
562ご冗談でしょう?名無しさん:03/11/26 20:31 ID:dElNLlEU
N個の理想気体分子からなる系について、
絶対温度Tを一定に保ったまま、体積をV1からV2に増加するとき、
体系のエントロピーの増加量ってどうやって求めるの?

吸収した熱エネルギーΔQはkNTlog(V2/V1)になると思うんだけど...
(kはボルツマン定数)

エントロピーの定義そのものはΔS=ΔQ/Tだよね?
だから、∫[V1,V2](ΔQ/T)dVでいいの?
563562:03/11/26 20:40 ID:dElNLlEU
というより、それができないから困ってるわけですが…

よろしくお願いします。
564ご冗談でしょう?名無しさん:03/11/26 20:41 ID:???
>>561
(cosβ)^2-(sinβ)^2とか2sinβcosβとか見たら
条件反射的に思い出す公式はないかい?
565ご冗談でしょう?名無しさん:03/11/26 20:48 ID:???
ΔQじゃなくてΔWがkNTlog(V2/V1)ですね。
ΔQはTが一定になる分量だけ外部から吸収されます。
566565:03/11/26 21:10 ID:???
あ、鵜呑みにした俺馬鹿だ。笑ってくだちぃ。
kNTlog(V2/V1)はΔWじゃないでち。
567ご冗談でしょう?名無しさん:03/11/26 21:22 ID:dElNLlEU
>>565
レスthanks


等温膨張の場合はΔQ=ΔWではないのですか?

でも、今良く考えてみると、積分なんかする必要ないのかな?
そのままかければ。

エントロピーって正直よくわからん。
568565:03/11/26 21:30 ID:???
U=1.5NkTとpV=NkTよりΔU=0,
ΔS=ΔU/T+pΔV/T=pΔV/T=ΔNklnV

…スマソ。やっぱりΔQで良かったのねん。
569565:03/11/26 21:35 ID:???
>>567
U(T,p)=1.5NkT
V(T,p)=NkT/p

二つ使わないとΔS計算出来ないです。
570559:03/11/26 22:51 ID:a+NIPoL2
>>564
思い出しました・・・。
でも、
2v_0^2[cosαcos2β+sinαsin2β]/g(cosα)^2
となって、カッコの中が0になればいいから
cosαcos2β+sinαsin2β=0
tanαtan2β=-1
tan2β=1/tanα
?ここから先へ進めないです・・・。
571 :03/11/26 22:55 ID:FueYyXMQ
なんかカッコいい名前の現象ないですか赤方偏移みたいな
572ご冗談でしょう?名無しさん:03/11/26 23:13 ID:???
573ご冗談でしょう?名無しさん:03/11/26 23:20 ID:???
>>570
そこまで行きゃぁ、tanβが出て、あとはイモヅル式にsinβ、cosβも出るだろ。
題意からβは0°〜90°の範囲としていいだろうから符号の不定性もない
574562:03/11/27 08:02 ID:???
>>569

最終的には
NkIn(V2/V1)
になりますか?

いや、もしかしたら何か勘違いしてるかも……俺。

遅レススマソ。
575ご冗談でしょう?名無しさん:03/11/27 15:08 ID:LcxdpxV2
すみません。文系学生です。
0<A<1
Aのx乗をxで1回微分した値は、正でしょうか、負でしょうか
576ご冗談でしょう?名無しさん:03/11/27 15:19 ID:???
>>575
微分ができないのか、微分はできるがその符号がわからんのか

577ご冗談でしょう?名無しさん:03/11/27 15:21 ID:ybCjDqm4
物理知識0から赤点をとらないようにしたいんですが10日ぐらいで
アドバイスください
578575:03/11/27 15:22 ID:LcxdpxV2
微分をすると、AのX乗logAになると思うのですが・・・。
これは、正でいいのでしょうか?
グラフを書いてみると、減少関数なので、負ではないのかと思うのですが。
579ご冗談でしょう?名無しさん:03/11/27 15:23 ID:???
>>575
580ご冗談でしょう?名無しさん:03/11/27 15:23 ID:???
>>578
0<A<1のときのlogAの符号は?
581ご冗談でしょう?名無しさん:03/11/27 15:24 ID:???
10日もありゃ中学生でもその関数を微分できるだろ…
582577:03/11/27 15:26 ID:ybCjDqm4
ちなみに高校2年のテストですが
2学期中間は6点でクラスビリでした
誰か勉強方法教えて

583ご冗談でしょう?名無しさん:03/11/27 15:28 ID:???
問題を解く。そんだけ。
584577:03/11/27 15:29 ID:ybCjDqm4
>>583
OK、やってみます
また質問があったら助けてください
585ご冗談でしょう?名無しさん:03/11/27 15:29 ID:???
手の込んだ釣りの予感・・・
586575:03/11/27 15:58 ID:LcxdpxV2
>>580
正ではないのですか?
高校の教科書引っ張り出して、勉強しなおします。
587ご冗談でしょう?名無しさん:03/11/27 16:05 ID:???
>>575
log(x)は単調増加関数. log1=0.
588575:03/11/27 16:12 ID:LcxdpxV2
では、(0<A<1)で、logAは負になるので、
Aのx乗をxで1回微分した時の符号は負になるのですね。
ありがとうございました。
589ご冗談でしょう?名無しさん:03/11/27 16:33 ID:gDPcPHlA
高校の物理のテストで素粒子の問題を出すとか言われたのですが、
問題集をつつこうがクォークだとかレプトンが出てくる問題がありません。
生徒にどのような事を素粒子の問題で問おうとすると思いますか?
590ご冗談でしょう?名無しさん:03/11/27 16:35 ID:???
>>589
そんなことを気にするより、テスト範囲の内容をしっかり理解することを考えろ。
591ご冗談でしょう?名無しさん:03/11/27 16:36 ID:???
>>573
そもそも普通はtanαtan2βなんぞに変形しない
592ご冗談でしょう?名無しさん:03/11/27 16:44 ID:???
>>589
素粒子というカムフラージュで電磁気とか力学で攻めてくるんだろうな。
593589:03/11/27 17:15 ID:gDPcPHlA
>>590 >>592
確かにメインは交流とか光電効果とかドブロイ波、ボーアの理論で
素粒子なんてのは出て10点ですけど出すとしたらどのような問題が
ありそうですか?
594ご冗談でしょう?名無しさん:03/11/27 17:42 ID:wiZwvbY7
何でうんこが臭いのか解りませんか?
教えてください。お願いします。
595ご冗談でしょう?名無しさん:03/11/27 17:46 ID:???
>>594
「うんこは臭い」と考える文化の中で育ったから臭く感じる。
一種の洗脳。
596ご冗談でしょう?名無しさん:03/11/28 01:03 ID:???
あれ、書いたのに消えた。。
フェルミ分布関数についてですが、exp(E-Ef/kbT)の
Efはフェルミ準位らしいですが、
Eはなんなのでしょう?
具体的に数値を求める問題でなにを代入すればいいのか困ってます。。

よろしくお願いします。

ちなみに問題は、温度が4.2、300、800Kにおいて、
フェルミ分布関数が0.1および0.9となるのは、
フェルミエネルギーからそれぞれ何eV上または下のエネルギーか。
またボルツマン分布の場合はどうか? です。

図書館休館かつぐぐってもわからなかったんでお願いします。すいません。
597ご冗談でしょう?名無しさん:03/11/28 01:48 ID:F5EETgiG
カーボンナノチューブのらせん構造について、それにバンドギャップがあって
半導体的な電子構造を持つことを説明したいのですが、それについて詳しく述
べた本はないでしょうか?
598569:03/11/28 07:15 ID:???
>>574
なります、なります。568でΔS=ΔNklnVと書いたのは、
ΔS=NkΔ(lnV)の方の意味で(klnV)ΔNの意味では無いです。
私はチョット悪癖を身につけてしまったので、回答者には不適格ですね。
申し訳ない事をしました。

とはいえ‥
S=Nk(lnV+1.5lnU)+f(N)

∵混合気体でなければf(N)はNに比例する。
599ご冗談でしょう?名無しさん:03/11/28 10:28 ID:???
>>596
Eはエネルギー
600ご冗談でしょう?名無しさん:03/11/28 14:01 ID:MWkJTu6Y
I=1/2の核スピン系における300Kでの低エネルギー(占有率N1)および高エネルギー
(占有率N2)準位の占有比R(=N2/N1)はいくらか?

ちなみに
N1=exp(u*H/kt) N2=exp(-u*H/kt)
uは核磁気モーメント Hは磁場 kはボルツマン定数 tは300K

ぜんぜん分からないのですけどどなたかお願いします。
601562:03/11/28 14:03 ID:l/en/thA
>>598
確認サンクス。

絶対エントロピーとなると、また話がややこしくなりますね。
この辺は、もう少し勉強してみます。

ありがとうございました。
602物理学科2年:03/11/28 14:09 ID:4yszbrIL
水平面から角αだけ傾いた斜面に底面の半径r、質量m、慣性モーメントIの
円柱をおき、静かに放す。そのときの時刻を時間tの原点に選び、
その位置から斜面に沿って下向きに測った円柱の質量中心の位置をxとする。
円柱は滑らずに転がり落ちるものとして、
円柱の運動に対して成立する力学的エネルギー保存則を求めよ。



_| ̄|○ 求めてくださぃ・・わからないです・・。
603ご冗談でしょう?名無しさん:03/11/28 14:19 ID:???
2年生か…
604ご冗談でしょう?名無しさん:03/11/28 14:24 ID:???
学科2年でこの程度の問題も。。。?
んなアホな ┐(´ー`)┌
605とある高校生:03/11/28 17:19 ID:PGBoCzuO
気柱の共鳴の実験にて
ガラス管内に水蒸気があるために、音速が速くなることを考慮すると、一番目の共鳴点や、二番目の共鳴点が変わると聞きました。
それはなぜ?また、補正のための式などあるのでしょうか?御回答よろしくお願いいたしますm(_ _)m
ちなみに、飽和水蒸気圧は、25℃のとき、3168[N/m^2]だそうです。
606ご冗談でしょう?名無しさん:03/11/28 21:14 ID:???
>>602
今すぐ大学を辞めて実家に帰れ
607ご冗談でしょう?名無しさん:03/11/28 23:07 ID:???
宿題をそのまま書き込まれてもなー
608ご冗談でしょう?名無しさん:03/11/28 23:11 ID:???
エネルギー保存法則とはいったい何でしょう。
暖房だけだと電気から熱にかわるけれど暖房と冷房だと打ち消しあって電気が消えてしまう。
エネルギーが消えてもこの世からエネルギーが少なくなるわけではない?
609ご冗談でしょう?名無しさん:03/11/29 00:06 ID:???
>>608
つーか、純粋な冷房機ってないからな。
クーラーには室外機があって、クーラーからの冷気以上に熱気を出している。

とりあえず、あなたは何年生ぐらいの方ですか?それがわかると答えやすい。
610物理学科2年:03/11/29 09:49 ID:yiOnmU8U
自分はヘボ2年生です!今から猛勉強しようと必死です!
運動方程式は、たぶんわかります。
斜面に沿って下向きにx軸、それに垂直にy軸をとって、
円柱を質点と考えた時の運動方程式と、回転の運動方程式が立ちます・・よね?
エネルギー保存則って、位置エネルギーがこのまんまの系だと困りますよね?
基準はやっぱり水平面?・・・_| ̄|○
611ご冗談でしょう?名無しさん:03/11/29 09:53 ID:???
>>610
運動方程式が書ければ保存則も分かるはずだが。積分して。
612ご冗談でしょう?名無しさん:03/11/29 11:09 ID:0P3sKy6F
学校で元素スペクトル測定の実験をしました。
水銀と水素のスペクトルをレコーダで連続的に記録したのですが、課題に
「測定した水銀スペクトルの波長より波長校正表をつくり、水素スペクトルの
波長を決定せよ」というのがありました。ところが波長校正表というのが
わかりません。実験の誤差を補正できるということなのでしょうか?
613ご冗談でしょう?名無しさん:03/11/29 11:22 ID:???
Hgスペクトルの実験結果と世界平均との差をリファレンスにして
Hスペクトルの実験結果をシフトさせろ、って事じゃないの?
614ご冗談でしょう?名無しさん:03/11/30 01:40 ID:???
ある本のproperties of X-raysというところを読んでいるのですが、
Balanced-Filterというのがよくわかりません。
日本語にどうやって訳したらよいでしょうか。
615ご冗談でしょう?名無しさん:03/11/30 05:55 ID:???
>>610
慣性モーメントの定義さえ教えれば高校生でも出来ることだよ。
もう一度なにをするべきか見直してみ。
問題文がわからないのなら日本語の勉強するとか。
ようはハミルトニアンを書けば姉妹なんだよ。
616ご冗談でしょう?名無しさん:03/11/30 21:56 ID:nIhc3ytC
ヽ(`Д´)ノボッキアゲ
617ご冗談でしょう?名無しさん:03/11/30 22:11 ID:2xEbqhAb
MOSトランジスタの特性についてですが、
n-MOSトランジスタのIC-VCE特性は
すぐ飽和状態になるのに、
なぜp-MOSのほうはすぐ飽和状態にならないのですか?
はっきりとした理由が説明できないので、どうか教えてください。
618ご冗談でしょう?名無しさん:03/11/30 23:24 ID:RtmRzu89
mmHgとPaの関係を導けっていう問題が、大学のレポートにでたんですけど、
高校のときの教科書もなく、大学の教科書にはぜんぜんのってないので、お手上げ状態です。
手がかりだけでも教えてください。
619ご冗談でしょう?名無しさん:03/11/30 23:30 ID:???

LRC回路に入力電圧e_o(t)が加わったときのコンデンサの端子間電圧e(t)は
LC*e_o''(t) + RC*e_o'(t) + e_o(t) = e(t) 
と表される。
この方程式をe_o(t)と回路電流i(t){ = C*e_o'(t)} を状態変数とする状態方程式に変換せよ。

こういう問題なのですが、解くにはどういった方針をとればいいのでしょうか?
教えてください。
620ご冗談でしょう?名無しさん:03/11/30 23:46 ID:dfGoEKH2
ふとした疑問なんですけれどよろしく
お願いします。

2つの物質があります。質量が同じで体積が異なった物質を水槽の中の水に入れると
水かさが増すのは体積が大きいほうなのでしょうか?

ずっと同じだと思っていたんですけれど・・・
わたしは馬鹿ですか?
621ご冗談でしょう?名無しさん:03/11/30 23:50 ID:nIhc3ytC
>>620
体積の分だけ水が排除されるんだから、その分かさが増でしょ。
お風呂で汲み桶とか洗面器とかを逆さにしてお湯に沈めたりした経験ない?
622ご冗談でしょう?名無しさん:03/11/30 23:55 ID:51ddbF7J
>>621さん
なるほどです。正直無知な自分が恥ずかしいですw
ありがとうございました。
623ご冗談でしょう?名無しさん:03/12/01 00:00 ID:???
>>620
・・・・・・(゚Д゚)ポカーン
ちょっとあんた・・・・・・・・・・・
無知とかいうレベルじゃないよ
そういうことって知識か?
624ご冗談でしょう?名無しさん:03/12/01 00:31 ID:i6Hi4Gx4
>>623
いや知識じゃないですね。
馬鹿です。
625ご冗談でしょう?名無しさん:03/12/01 00:32 ID:???
煽ってやるなよ。ここで知識が修正されたんだ。むしろ喜ばしい事だろ?
626ご冗談でしょう?名無しさん:03/12/01 00:38 ID:???
同意。>>623みたいなやつがいるから理系が虐げられる。
627早大一年:03/12/01 00:55 ID:gjRwjWnH
方形波と正弦波の聴覚特性の違いを考えろってプリントに書いてあるんですが、聴覚特性ってなんすか?!!


628ご冗談でしょう?名無しさん:03/12/01 01:02 ID:IZqPLOFD
>>614お願いできませんか?板違いでしたでしょうか。
629ご冗談でしょう?名無しさん:03/12/01 01:19 ID:???
漏れは分野違いだが、物理学辞典なり専門の和書なりで調べてみたら?
630614:03/12/01 01:23 ID:???
>>629
そうします。レスありがとうございました。
631ご冗談でしょう?名無しさん:03/12/01 01:24 ID:???
>>619をお願いいたします。
632ご冗談でしょう?名無しさん:03/12/01 01:30 ID:???
>>619
いまいちインデックスを理解できていないが、
同次系にして(右辺e(t)=0で考えて)
その微分方程式でも解いてみたら?
633ご冗談でしょう?名無しさん:03/12/01 03:17 ID:???
>>627
フーリエ級数を考えると簡単。矩形波は無限に高い高調波を含んで
いるので、「ジー」というかなり耳障りな音になる。vectorかどっかに
任意のフーリエ級数の音を出すソフトがあったと思うからダウソして
確かめてみるとよい。
634ご冗談でしょう?名無しさん:03/12/01 03:23 ID:???
>>627
暇だから探してきた。
あまり出力できる波の種類は多くないが、これで十分。
http://www.vector.co.jp/soft/dl/win95/art/se097634.html
635ご冗談でしょう?名無しさん:03/12/01 04:20 ID:0yq1njQc
相対性理論の本を読んでいてよくわからないところがあるので質問します。
空間の座標変換をする際、ベクトルの成分の変換を、反変、共変のように
あるルールに従って変化させるということが書かれているんですが
相対性理論での空間の座標変換って一体何から何に変化させるための
ものなのかがよくわからないのであまりぴんときません。
絶対座標から相対座標に変化させるとかそういうものですか?
どなたか教えてください
636ご冗談でしょう?名無しさん:03/12/01 05:22 ID:RCSbdot0
質量保存の法則は間違ってる
何もないとこからは何も生まれないんだろ?
なら世界は生まれないんじゃないか?
最初の始まりって何から生まれたの?
質量保存の法則って間違ってるよね。
637ご冗談でしょう?名無しさん:03/12/01 05:50 ID:???
>>635
別に相対性理論に限らず、テンソル解析を行う時には反変ベクトル
としての表現と、共変ベクトルとしての両方の表現を使うことがある。

相対性理論の場合は反変ベクトルがローレンツ変換、共変ベクトル
がローレンツ変換の逆変換に対応している。

マクスウェルの波動方程式ではガリレイ変換に対して不変ではな
かったのでアインシュタインはローレンツ変換なる物を定義して対応
した。これは光速不変の原理から導かれる。絶対静止系を一般慣性
系に変換する。当然逆変換はその逆を行う。

余談だが、アインシュタインは相対性理論を導く途中で重力法則は
一般共変性を持たないとする、いわゆる「穴の議論」に嵌って、これ
から抜け出すのに2年を費やしている。これでよいか?
638ご冗談でしょう?名無しさん:03/12/01 06:01 ID:???
>>636
質量保存の法則は広義にはエネルギー保存の法則の一例である。
だから最初にエネルギーがあれば質量なんかなくても構わない。

こんな事を言うと驚くかもしれないが、例えばガスを燃焼するとする。
するとそれが発熱反応で反応後二酸化炭素と水になったとする。
反応前後の質量を比べてみると、ガス>二酸化炭素+水なのだ。
ごくわずかだけどね。減った分は当然Δm=E/c^2です。

核反応ではわかり易いほど質量が減少するが、一般的な科学反応
でも実はエネルギー保存の法則は成り立っている。
639ご冗談でしょう?名無しさん:03/12/01 06:03 ID:???
×科学反応
○化学反応
640ご冗談でしょう?名無しさん:03/12/01 06:14 ID:???
>>638
化学屋さんに叱られそうだから一応訂正。

×ガス
○ガス+酸素

まあ酸素を必要としない燃焼もあるし、酸素以外の支燃性ガスも
あるし、いろいろだけどね♪
641ご冗談でしょう?名無しさん:03/12/01 06:19 ID:???
636は「一夜城を作ったのは豊臣秀吉じゃない。大工さんだ!」
と言いたいんじゃないかと思った。
642ご冗談でしょう?名無しさん:03/12/01 07:00 ID:???
うーんでも全く何もない所でもエネルギーさえ与えれば素粒子は
対発生するからねえ。そのエネルギーの所有者として豊臣秀吉
を考えてもいいんじゃないでしょうか。
643636:03/12/01 07:40 ID:RCSbdot0
エネルギー保存法則もまちがってる
保存も何も最初の段階ではエネルギーも何もないだろう。
というか物理や化学って地球限定のものなの?
644ご冗談でしょう?名無しさん:03/12/01 07:56 ID:???
足立区限定です
645ご冗談でしょう?名無しさん:03/12/01 08:05 ID:???
>>643
最初の段階って何?(ぷ
646ご冗談でしょう?名無しさん:03/12/01 08:07 ID:???
>>643
0=+1−1

>というか物理や化学って地球限定のものなの?
実験で確認できるとこ限定。
647ご冗談でしょう?名無しさん:03/12/01 08:09 ID:???
>>643
それはおかしい。もし最初にエネルギーがゼロだったら、ビッグバンも
起きなかったはず。最初の状態は4つの力が満ちた(分離せずに統一
した)高エネルギーで高密度高温、しかも極小の点だったとされている。
(もちろん誰もそれを証明した人はいない、超弦理論やM理論ででさえ
これを説明できない)

それから先はご存知の通り大爆発が起き、4つの力が次々と分離を
始め、宇宙は膨張を始めたと考えられている。

これは3K黒体輻射の観測と遠方の星からの光の赤方偏移(光のドッ
プラー効果、ハッブル定数)から推定されている。

そしてご存知の通り、急激に発生した素粒子と反素粒子が対消滅し、
CP対象性の破れにより素粒子の方が残り、それが原料となって原子
や物質ができたとされている。
648ご冗談でしょう?名無しさん:03/12/01 08:10 ID:???
>>643
そんなにいうなら論文かいて、どっかに投稿しろよ。
649ご冗談でしょう?名無しさん:03/12/01 08:13 ID:???
>>643
つまりこの宇宙の至る所で全く同じ原理で原子が出来ているので、
地球から離れた遠くの星に仮に行って見ても、そこで発見できるも
のは地球上にあるものと同じような物質(ただし同位体などの割合
は違うかもしれない)、物理や化学も地球と全く同じ規則に従って
いる。
650636:03/12/01 08:35 ID:RCSbdot0
ビックバンが始まりじゃないだろ?
4つの力が満ちた高エネルギーでもない。
その前だよ。いちばん最初。
そのエネルギーはどっから生まれたんだよ?
もしもともとあったと言うなら無から生まれた以外にはないだろ?
だから質量もエネルギー保存の法則も間違ってる。
651ご冗談でしょう?名無しさん:03/12/01 08:44 ID:???
時間が不連続だと言うつもりなら
既に物理じゃないからどっか行けば?
652636:03/12/01 08:50 ID:RCSbdot0
時間も物理の範囲じゃないのかよ!?
653ご冗談でしょう?名無しさん:03/12/01 09:09 ID:itDZdvsU
>>636
おまえのいっている「いちばんさいしょ」っていうところでは
きぞんのぶつりがくがはたんしてるんだよ
だからおまえのやっていることは あるほうそくがせいりつしないじょうけんかで
そのほうそくをつかっているだけ
654ご冗談でしょう?名無しさん:03/12/01 09:28 ID:???
百歩譲っても
物理と言うよりSFだな、「一番最初」なんて概念は。
655ご冗談でしょう?名無しさん:03/12/01 09:35 ID:???
>>647
原初のエネルギーはゼロだよ。
ただし、物質のエネルギーと空間(重力場)のエネルギーが相殺して、という意味でだけど。
「高い真空のエネルギー」が超高密度に詰まった空間がトンネル効果で出現した。
「高い真空のエネルギー」はものすごい勢いで、「低い真空のエネルギー」へと落ち込み(相転移)、
この時のエネルギーの差額がビッグバンの源となった。
つまり、宇宙は物質エネルギーをずっと「借金」し続けていると考えればよい。

656ご冗談でしょう?名無しさん:03/12/01 09:59 ID:???
>>655
やっぱり無からか・・・・

そう言えば11次元が4次元に崩れ落ちてそれが大爆発のエネルギー
になったんだっけ。

で、4次元の他の世界は「縮退」してカラビ=ヤウ多様体になっている
のかな?その縮退した空間には4つの力のうち重力しか入り込む事が
出来ず、そのため4次元の重力が弱くなっているという理論もあったな。

まるで石鹸の泡のような話だ・・・・
657ご冗談でしょう?名無しさん:03/12/01 10:04 ID:???
それにしても今入手できる本で超弦理論の本を本屋で手に取って
見たけど、本当にもう数式の羅列。純粋数学の世界ですよ。

でもこのまま手をこまねいているわけにも行かず、群論がどのよ
うに超弦理論に組み込まれているのか立ち読み。結果→頭あぼーん。
あいにくM理論の良書はまだ見たことないんだよなあ。
658ご冗談でしょう?名無しさん:03/12/01 10:13 ID:???
そもそも時間の一様性から導かれるエネルギー保存則が、
宇宙初期の、時空がダイナミックに変動しているときに
成り立っていないとして何か問題あるの?
659ご冗談でしょう?名無しさん:03/12/01 10:25 ID:???
それは、インフレーションが始まる前までは何でもあり、って事?
660ご冗談でしょう?名無しさん:03/12/01 10:29 ID:???
宇宙の全エネルギーが時刻の関数で表されないかって事でしょ?
それは僕も知りたい。
661ご冗談でしょう?名無しさん:03/12/01 10:43 ID:???
>>658
そりゃないでしょ。
そもそもこの今生きている宇宙に現行の物理学が適用できるように
なったのも「宇宙の晴れ上がり」からでしょうし。もっともまだまだ
アッチッチの世界だから厳密にはいろんな定数がずれていただろ
うな。
662ご冗談でしょう?名無しさん:03/12/01 11:57 ID:???
晴れ上がりは、宇宙誕生後30万年経過した、かなり進化した宇宙でしょ。
現行物理法則で予測可能なのは、もっとずっと先。
「地球から観測できる宇宙」という意味なら同意だが。
663ご冗談でしょう?名無しさん:03/12/01 12:50 ID:???
>>662
そしたら、電子が全て原子に捕まえられ光子が直進できるように
なった時としておきます。「現行物理法則」の範囲も明確にしてお
く必要がありますね。新しい理論ほどビッグバンの瞬間に近づく
ことができますから。
664ご冗談でしょう?名無しさん:03/12/01 12:51 ID:???
×原子
○原子核
665ご冗談でしょう?名無しさん:03/12/01 14:14 ID:???
>>662
む。今見たら「エネルギー保存則」となっている。これじゃあ宇宙の
晴れ上がりも何も関係ないや。相転移した直後からになるのかな?
666ご冗談でしょう?名無しさん:03/12/01 22:37 ID:sojlJfoT
CR積分回路と微分回路において時定数の実効値と理論値がなぜ異なるのでしょうか?
667ご冗談でしょう?名無しさん:03/12/01 22:39 ID:9bOj1v0A
 (健康体)  (喘息)

1.(神が喘息であるかないかを決める)
  Y        I
2.喘息でない人  喘息の人は
は体力がある   体力がない
Y I
3.        行動力、
         五感(嗅覚)が鈍り感性が変化
         する
  Y        I
4.神は異常な感性の人間は本来人に迷惑をかけるから外に出てはいけないと思っている。
Y I
5.変化なし    アトピーになる
Y I
6.正常な感性   外に出なくなりさらに異常な感         性になる
Y I
7.正常な人間    異常な人間(レッテル)
668ご冗談でしょう?名無しさん:03/12/01 22:39 ID:9bOj1v0A
Y I
8. 死
  Y        I
9.      来世
Y I
10.神は異常な人間は人に迷惑をかけるので行動を抑制する必要がある
Y I
11.神は異常な人間は人に迷惑をかけるので行動を抑制する必要があると思っている。
Y I
12.神が喘息であるかないかを決める
  Y        I
13.喘息でない   喘息である
  Y        I
     1.に戻る

神は事態の収拾に必死、頑張れよー。
669ご冗談でしょう?名無しさん:03/12/01 22:39 ID:9bOj1v0A

アトピー性皮膚炎の治し方

行動力=ガッツ=体力

アトピー性皮膚炎の患者は、
感覚の鈍い人間が多い。
それはなぜか。
幼少期喘息になるから、
体力がつかないため、
五感が弱るからである。
解決方法は、
五感を強くしてやればいい。
体力をつけることですぐに五感が正常になり、
行動力も湧くのである。
五感が正常になり、体力もつけば、
アトピー性皮膚炎も不思議と消えることが多い。
吸入器は一回の運動に一度か二度なら使っても
まるで問題なし。
670ご冗談でしょう?名無しさん:03/12/01 23:06 ID:???
何のコピペか知らんが、アトピーなめんなよ
671ご冗談でしょう?名無しさん:03/12/01 23:19 ID:lkEusL4L
昔、とある番組で『目の錯覚』を特集してて

白黒の渦が中心に向かってグルグル回る模様の中心を30秒くらい見つめて、

その後目線を他にやると世界が歪む、ってのがあったんですが、

グルグル動く画像ありませんか?久々にやってみたいので

ググって探してみたんですが、見つからず((;゚Д゚)
672ご冗談でしょう?名無しさん:03/12/01 23:54 ID:9Zi89/zN
        ∧_∧∩   / ̄ ̄ ̄ ̄ ̄ ̄ ̄ ̄ ̄ ̄ ̄ ̄ ̄ ̄ ̄ ̄ ̄ ̄ ̄
       ( ´∀`)/  < 先生!  誘電関数ε(ω, K)ってなんでつか?
  __ / /   /     \___________________
  \ ⊂ノ ̄ ̄ ̄ ̄ ̄\
  ||\           \
  ||\|| ̄ ̄ ̄ ̄ ̄ ̄ ̄||
  ||  || ̄ ̄ ̄ ̄ ̄ ̄ ̄||
     .||              ||

キッテル読んでたんですが、いきなり誘電関数がでてきて行き詰まってまつ
そもそもωとKってのは電磁波の周波数と波数でつか?
誘電関数の簡単な説明だけでもホスィでつ

初心者でスマソ
673671:03/12/01 23:58 ID:lkEusL4L
>>671

一応見つかりマスタ

Space Warperっていうんでつね


ttp://www.cmo.jp/users/wy/ik/ext/kaleidoscope/ks/ks_space_warper/ks_space_warper.html

貼っときますんで、一度お試しあれ

結構面白いでつ( ´∀`)

あとはDLしなくてもいつでもできるよう、gifアニメかなんかで保存できるとこないかなぁ〜

このサイトがアボーンしたらできなくなるし((;゚Д゚)
674ご冗談でしょう?名無しさん:03/12/02 00:01 ID:???
初読でキッテルはやめとき。。
675ご冗談でしょう?名無しさん:03/12/02 00:06 ID:???
>>673
今俺試してみたけど、ぐるぐる回る図形を見た後他の物体に目を
移したら、その物体から光のとげのようなものがたくさん突き出て
見えた。ぐにゃぐにゃとはしなかったな。
676ご冗談でしょう?名無しさん:03/12/02 00:08 ID:+p+x1n61
>>675

わたしは最初トゲしか見えませんでした

でも、何回もやってたらグニャグニャなってきましたよ☆





気持ち悪くなるのでやりすぎには注意ですが・・・
677ご冗談でしょう?名無しさん:03/12/02 00:21 ID:???
>>676
うわっ本当だ。大きくなったり小さくなったりする。
そして物体と物体の間に無数のクモの糸のような物が見えた。
678アヒィ:03/12/02 00:59 ID:AtwvM0Ad
すいません、質問です。

光速の99.5%の速さで地球から遠ざかるロケットに乗る人は、
地球にいる人に比べて時間の経ち方が10倍ほど遅くなると思います。

ここからです。ロケットから「いち、に、さん・・」と1秒刻みにカウントをとったものを
電波で地球に向けて飛ばすと、地球の人は「い〜〜〜ち、に〜〜〜、さ〜〜〜ん・・」
というように受信されるのでしょうか。

あたまがこんがらがりそうです。
識者の方、教えてください。。
679ご冗談でしょう?名無しさん:03/12/02 01:23 ID:???
>>678
電波が赤方変移するから周波数が下がるだけで情報の伝達速度は
不変では?
680ご冗談でしょう?名無しさん:03/12/02 01:36 ID:???
時間ののびや距離の変化も同時に考えろ
681ご冗談でしょう?名無しさん:03/12/02 01:41 ID:???
>>680
そうか。そうなると情報の伝達速度は不変でも電波に載せられる方の
情報の早さが遅くなるから「い〜〜〜ち、に〜〜〜、さ〜〜〜ん・・」
となるのか。
682ご冗談でしょう?名無しさん:03/12/02 02:43 ID:eK1POUHg
>>637
ありがとうございました
とても参考になりました
683ご冗談でしょう?名無しさん:03/12/02 05:04 ID:???
>>674
え、むしろ初読こそキッテルじゃないの?
684ご冗談でしょう?名無しさん:03/12/02 09:32 ID:???
>>678
非相対論的に考えた場合、ドップラー効果によって「い〜〜ち、に〜〜、さ〜〜ん・・」になる。
相対論的効果が加われば「い〜〜〜ち、に〜〜〜、さ〜〜〜ん・・」になるだろうな。
振動数自体も小さくなるので聞き取れないほどの低音になるだろうが・・・・
685ご冗談でしょう?名無しさん:03/12/02 10:43 ID:???
>>Kittel
読むに従ってどんどん定義説明のない式が出てくるようになる。
686ご冗談でしょう?名無しさん:03/12/02 19:29 ID:???
人工衛星が地球に落下するときは燃えるけど発射するときには燃えないのは、
燃える理由が大気摩擦ではなくて断熱圧縮だから?
687ご冗談でしょう?名無しさん:03/12/02 21:11 ID:1vkgBQKg
>>686
単にスピードが違うからだろ
688ご冗談でしょう?名無しさん:03/12/02 21:53 ID:VeJzzwDu
>>687 落下速度が第一脱出速度を越えるとでも?
689ご冗談でしょう?名無しさん:03/12/02 21:58 ID:???
>>688
aho
690ご冗談でしょう?名無しさん:03/12/02 21:58 ID:1vkgBQKg
>>688
越えませんが何か?
691686:03/12/02 23:09 ID:VeJzzwDu
で、どうなんよ
692ご冗談でしょう?名無しさん:03/12/02 23:13 ID:???
人工衛星が萌えるのは摩擦よりも断熱圧縮によるところが大きいのは事実。
693ご冗談でしょう?名無しさん:03/12/02 23:14 ID:???
>>691
>>687で不満か?
694ご冗談でしょう?名無しさん:03/12/02 23:19 ID:???
教科書スレが荒れているのでこちらでお願いします。
磁性のよい本を探しているのですが、金森、芳田は今は売ってません。
WhiteやMattisなどはどうでしょうか?他にも何かあれば和書でも洋書でも
いいので教えてください。特に金属磁性、遍歴電子系に詳しいのがいいです。
お願いします。
695ご冗談でしょう?名無しさん:03/12/02 23:26 ID:VeJzzwDu
>>693
ようは摩擦熱だけ?
696ご冗談でしょう?名無しさん:03/12/02 23:32 ID:???
>>695
は? どうしてそうなる? 打ち上げ直後の大気圧が大きいところでは
スピードが遅いってだけ。

まさかロケットは打ち上げ直後から第一脱出速度(第一宇宙速度のこと?)を
出してるとでも思ってるの?

697ご冗談でしょう?名無しさん:03/12/02 23:40 ID:???
>>696
もちつけ
>>695
おめでたいな。そんなに人工衛星を花火にしたいか。
698ご冗談でしょう?名無しさん:03/12/02 23:49 ID:VeJzzwDu
スピードが遅いのはわかっとる。
だからその遅さが摩擦熱に影響するのか
それとも急激な断熱圧縮に影響するのか。
どっちの原因によって打ち上げは無事で落下が燃えるのか。
699ご冗談でしょう?名無しさん:03/12/03 00:16 ID:P6OvsT72
打ち上げが無事なのは、ゆっくり上昇しつつ真空中で最高速に達するから。
再突入では逆に最高速で大気圏に突入し運動エネルギーを断熱圧縮で熱に
変換することで減速するから。
700ご冗談でしょう?名無しさん:03/12/03 00:43 ID:???
>>699
へぇ〜そうなんだ。
誰だ摩擦熱で燃えるとかほざいたDQN
701ご冗談でしょう?名無しさん:03/12/03 01:15 ID:???
>>694
その質問は2chではレベル高すぎるんじゃないの?
これに、なにげに答えられる人がいればすごい!

amazon.comのAdvance Searchで
Subjectを「Magnetism」、
Publication dateを「After the year 1990」、
Sort byを「Avg. Customer Review」にしてみるというのではいかが?
702ご冗談でしょう?名無しさん:03/12/03 01:22 ID:???
>>700
お前
703ご冗談でしょう?名無しさん:03/12/03 01:45 ID:???
>>698
>スピードが遅いのはわかっとる。
ププ
>>688書いたのどこのどいつだw
704晒し上げ:03/12/03 02:15 ID:???
>>703
越えますが何か??

ププッ
705ご冗談でしょう?名無しさん:03/12/03 02:17 ID:???
釣り注意報
706ご冗談でしょう?名無しさん:03/12/03 15:06 ID:???
単純な質問で申し訳ないのですが
メビウス巻きってなんか特殊なんですか?
707ご冗談でしょう?名無しさん:03/12/03 15:27 ID:???
そう。西海岸の奴らと来たら、何でも巻けばいいと思ってるんだよ。
もう少し調和とか考えて欲しいよな。
708ご冗談でしょう?名無しさん:03/12/03 21:17 ID:K6UZqeFT
ポケコンにちょっと長い計算をさせてると
ザーっていう音が聞こえてた記憶がある

アレは一体どういう原理で音が出ていたんだろう

きっとマザーボードからも同じ音が出ているだろう
709ご冗談でしょう?名無しさん:03/12/03 22:05 ID:???
中の人の悲鳴
710ご冗談でしょう?名無しさん:03/12/03 22:06 ID:XUsSEkC1
すみません、すっげー単純な質問で恥ずかしいんですけどおねがいします。

宇宙空間内って真空度はどれくらいですか?(何mmHgとか)
711ご冗談でしょう?名無しさん:03/12/04 04:38 ID:???
どこの宇宙よ?
712ご冗談でしょう?名無しさん:03/12/04 19:39 ID:mWBKXPhB
厨房の質問ですいません。
量子コンピューターってなんですか?
電子の重なりの状態を利用して0も1も同時に計算するというけれど
電子の状態の重なりというのは数学で取り扱いやすくするための便宜上のもので
実際に電子の状態が複数あるというわけではないのでは?
それとも実際、同時進行の別世界は観測されているのですか?

量子力学に詳しい人教えてください。
713ご冗談でしょう?名無しさん:03/12/04 20:48 ID:S9mBzH7e
位置センサー(超音波センサー)についての質問です。図をupしたので見て、お答え
頂けるとありがたく思います。
ttp://strawberry.atnifty.com/cgi/up/src/up0188.gif
超音波を出して壁に反射させて距離を測ると言うものです。質問の内容は、
「距離計測のカウンタの周期(VR)を調節によってカウンタパルスの何が
変化するのか?」
と言うものです。VRの値が大きいと周期が短いと言うことだけはわかりました。
この先どう考えて良いのかわかりません。お教え頂けるとありがたいです。
宜しくお願い致します。
714ご冗談でしょう?名無しさん:03/12/04 20:58 ID:ceKNlpua
コマや自転車はなぜ倒れないの?
肉食動物はなぜ野菜を食べないのに健康でいられるの?
715名無しSON:03/12/04 21:01 ID:hOwACkgM
コマ、自転車→力が釣り合っているうちは倒れない
肉食動物→持っている消化酵素の働きが違う
(`・ω・´)
716ご冗談でしょう?名無しさん:03/12/04 21:09 ID:33//Ra5G
>714
草食動物はなぜ肉を食べないのに早くはしれるの?
717ご冗談でしょう?名無しさん:03/12/04 21:57 ID:???
>>713
そのアドレス見れないよ
718ガイシュツ ◆atMKiSyUTU :03/12/04 22:18 ID:???
>712
量子コンピュータについて語るスレ
http://science2.2ch.net/test/read.cgi/rikei/1020566648/
719ご冗談でしょう?名無しさん:03/12/04 22:38 ID:LWdDwZpz
弦の振動について質問があります。

発振器と弦をつないで弦を振動させたとします。
では、弦を振動させる場所が、空気中と水中ではやはり違うのでしょうか?
もちろん、発振器で発振する振動数は一定とします。
具体的には、水中の場合は空気中より振動しにくいから、
振動数が空気中より小さくなるとか、振幅が変わり音量が小さくなるとか、
そのような違いはあるのでしょうか?
一見すると、空気中より水中のほうが、弦は振動しにくそうですが...

また、水に限らず、弦を振動させる周りの環境と弦の振動に関連はあるのでしょうか?
たとえば周りの物質の密度とかと振動数には規則性があるとか・・・

調べたのですが該当の資料が見つからず困っています。
どなたかご存知の方、ご教授よろしくお願いします!
720ご冗談でしょう?名無しさん:03/12/04 22:48 ID:???
>>719
ただのバネマス振動系の励起が非線形のバネマスダンパ振動系の強制振動になるわけで。

振動数は基本的に変わりませんが、振幅は小さくなるはずです。
発振機を切った後の残響は違う周波数にニョローンと動くはずです。(もしくは残響なし。)
後、水中だと音色(高周波成分っつーんですか?)が大分変わるはず。
721720:03/12/04 22:56 ID:CKYSv8q1
あ、>>720は発振機の振動数が空気中の弦の共振周波数と一致していた時の話。
722ご冗談でしょう?名無しさん:03/12/04 23:28 ID:33//Ra5G
水中で弦が振動するの?
振動数はどれくらい? MHz?
723ご冗談でしょう?名無しさん:03/12/04 23:30 ID:LRD2mhEk
ttp://homepage3.nifty.com/FLARE/UFO%20SIENCE.html

↑に超光速やら重力波電磁波変換やらの素敵技術が
語られていますが、これって実際なんなんでしょう。

なにやらリンクが張られていたり、natureに論文が載った
などとされていますが・・・
724ご冗談でしょう?名無しさん:03/12/04 23:32 ID:???
>>722
風呂の中で腰振ったらお前のちんぽはMHzで振動すんのかよ。
725ご冗談でしょう?名無しさん:03/12/04 23:52 ID:???
「水中の光の速度」でぐぐってみられ。あっと驚くような結果が待っているから。
726719:03/12/05 00:03 ID:QI5A37rJ
>>720
返信ありがとうございます!
高2の僕にはかなり難しい内容でしたが...
とにかく、ということは空気中でも水中でも振動数は変わらないということですか?

というのも、弦が縦波で振動しているのか横波で振動しているのか判別するのに、
水中を通すことによって判別できるかなと思ったのですが・・・・
水は空気より密度が高いので、横波だと水中では弦は振動しにくいかなと
思ったので、もし水中を通した後の振動数が発振振動数と違う場合は横波で、
同じだった場合は縦波かなと思い、判別しようと思ったのですが・・・
(縦波は進行方向の粗密波なので、水の影響は受けないかと思ったので・・・)

といっても、全部自分の思い込みでやっていたので・・・
やはりこの理論はめちゃくちゃでしょうか・・・?


727ご冗談でしょう?名無しさん:03/12/05 02:35 ID:PQ+6/AX6
LCR直列回路の交流応答で、
共振が起こる周波数よりも発振器の周波数が低いときって位相は進む?

実験で調べるのを忘れてレポートがやばい状況なので是非教えてください・・・
728727:03/12/05 02:38 ID:PQ+6/AX6
ゴメン。何に対してなのか書き忘れた。

電流の電圧に対する位相差が進む
のが正しいのかどうかを教えてください。
729ご冗談でしょう?名無しさん:03/12/05 08:08 ID:E+N7IyeW
>>717
申し訳ございません。頭に「h」が抜けておりました。
http://strawberry.atnifty.com/cgi/up/src/up0188.gif
こちらです。宜しくお願い致します。
730ご冗談でしょう?名無しさん:03/12/05 12:17 ID:???
>>726
弦の縦波って頭の腹痛、みたいな表現だな。
おまい頭の中でいろいろ混ざってるぞ。

・弦に発振機付けて周波数を下から上にゆっくりスイープして応答を見るとして、
 ピークの出る周波数は水中と気中で違いますか
 →違います。ちなみにピークの値も違い、特に横波だとピークは凄く小さくなるか無くなるかでしょう。


・100Hzで発振させたら103Hzで弦が振動することはありますか
 →気中だろうと水中だろうと定常状態ではありません。
731ご冗談でしょう?名無しさん:03/12/05 16:17 ID:FUDySW+O
相対性理論が成立しない高エネルギー物理学って何なんですか?
732ご冗談でしょう?名無しさん:03/12/05 16:30 ID:t4paXE26
なんか聞いたことあるな。
ローレンツ不変(共変)性が破れるとか。
まあ、トンデモの域を出ないだろうけど。
733ご冗談でしょう?名無しさん:03/12/05 20:17 ID:n2+iU1up
同じ重さの棒が2本あってそれを使い殴ったときの衝撃は
密度が高いほうが衝撃が大きいのでしょうか?
734ご冗談でしょう?名無しさん:03/12/05 20:31 ID:???
>>733
木刀は水に浮く。
735ご冗談でしょう?名無しさん:03/12/05 20:37 ID:???
>>733
mv = FΔt。
だからいくら密度が大きくても速度が速くないと衝撃は小さい。

密度が大きくかつ速度も速く、しかも力が働く時間が短かったら・・・・
そりゃ死ぬかもしれんね。車にはねられたようなもの。
736ご冗談でしょう?名無しさん:03/12/05 21:44 ID:iZS63pPf
素粒子、ニュートリノを子供に説明する時なんていったらよいですか。
何を研究してノーベル賞を受賞されたのかどう説明したらわかりやすいか
どなたか教えてください。


737ご冗談でしょう?名無しさん:03/12/05 22:47 ID:???
磁界に砂鉄を置くと模様ができるけども、
集まらない部分と集まる部分にはどんな差があるの?
738ご冗談でしょう?名無しさん:03/12/06 00:25 ID:vhbmgsx0
月はいつも同じ面が地球に向いていますが、これは地球に向いている面の方が重いってことですか?
739ご冗談でしょう?名無しさん:03/12/06 01:42 ID:zwUvNmrG
電磁気の質問なんですが
周期構造媒質に波が入射すると周波数は一定で様々な波数の空間高調波が発生するのですが
これは波数によっては光速を超えて伝播すると考えるのですか
740ご冗談でしょう?名無しさん:03/12/06 02:16 ID:c6uwSKCy
>>739
そう。でも媒質内での光速の話。
だから相対論には反しない。
741ご冗談でしょう?名無しさん:03/12/06 03:10 ID:???
便乗ですが、媒質内に入射する際屈折しますよね。
このとき光の向きが変わりますがこれは界面によって光子?が力?を受けているためと考えても良いですか。
742ご冗談でしょう?名無しさん:03/12/06 04:47 ID:???
入射光子が物質内部に吸収された後、放出されたと考える
743ご冗談でしょう?名無しさん:03/12/06 05:00 ID:???
>>738
no

月の公転周期と自転周期が(地球の朝夕力によって)
1:1になっているため。
744ご冗談でしょう?名無しさん:03/12/06 07:04 ID:???
>>743
潮汐力によって重いほうの半球が常に地球を向く(反対向きでもよいが)のが
安定だから自転と公転が同期したわけだが、なぜno?
745ご冗談でしょう?名無しさん:03/12/06 12:33 ID:???
>>738
月は出来た当初は自転周期が自身の公転周期と全く違っていたと
考えられる。というか同じである必要性は全くない。

しかし地球と月は互いに潮汐力を及ぼし続け、この力は公転速度
よりも自転速度が遅いと自転力を与え、公転速度よりも自転速度が
速いと自転を止めようとする。それはわずかに月が潮汐力により
変形する事により起きる。

その結果長い年月を掛けてついに月の自転速度と公転速度は
一致した。太陽系の惑星の回りを周回する衛星にもこれと同じ現象
が多く起きている事がわかっている。

もちろん地球も月から潮汐力を受け、自転速度がわずかずつでは
あるが遅くなりつつある。
746ご冗談でしょう?名無しさん:03/12/06 12:51 ID:msQ4qvjX
>>744
へ?月の半分って重いんですか?
回転の遠心力その他の力と地球に引かれる引力その他の力が釣り合ってるってだけじゃないの?
747ご冗談でしょう?名無しさん:03/12/06 13:13 ID:???
>>744
あなた潮汐力について根本的な誤解をしてますよ。
もう少し勉強しましょう。
748ご冗談でしょう?名無しさん:03/12/06 13:43 ID:???
大潮の反対側の海面はどうなってるでしょうか?とかな
749ご冗談でしょう?名無しさん:03/12/06 14:10 ID:???
>>736
ニュートリノは、通常の物体を簡単にすり抜けてしまう幽霊粒子で、
地球の厚みも平気で貫通できる。
人工的には原子炉での核燃料の燃焼や、加速器を使って作られる。
その極めて高い透過能力により、太陽内部を光より早く抜け出したりする。
逆に言うと、なまなかな方法では検出するのはまず不可能。
当時無理無茶無謀と言われながらも小柴教授率いる研究チームは、世界で初めて
巨大水槽型検出器カミオカンデを建設した。
(正確に言うと、もともとの目的は核子崩壊の検出でニュートリノはそのおまけ)

超新星爆発によって開放される重力エネルギーの99%はニュートリノによって持ち去られる為、
これを観測し、理論と照合する事は物理学・天文学にとって極めて重要。
この超新星爆発が銀河系近傍で発生する確率は数十年に一度。しかも爆発持続時間は
わずか10秒。
このタイミングを逃さず、カミオカンデチームは人類初の超新星爆発由来のニュートリノを
11例発見し、それらのエネルギーと、それから予測された全放出エネルギーは
理論とほぼ一致し、理論が正しい事を証明した。

この成功によって、ニュートリノを用いて宇宙の進化と天体現象を観測する
ニュートリノ天文学の重要性を世界に知らしめ、世界中で観測が行なわれるようになった。

…というのを噛み砕いて伝えるというのはどうでしょう?

750ご冗談でしょう?名無しさん:03/12/06 14:14 ID:k/60Be//
紐を垂らしたときにできる曲線ってカテナリー( y=a/2(e^(x/a)+e^(-x/a)) )ですよね
定規を曲げたときにできる曲線ってカテナリーなんですか?
それとも、別の曲線であるならばその式ってどんなんになるんでしょうか?
751ご冗談でしょう?名無しさん:03/12/06 15:27 ID:qdwrClFZ
レベルが低すぎるけど質問です。宿題なんですが全然わからない・・・。

170μCの電荷が一辺80cmの立方体の中心にとする。
(a)立方体の各面を通過する電束を求めよ。
(b)立方体の全表面を通過する電束を求めよ。

(a)はΦ=EA=k*(q/r^2)*A
=9*10^9*{170*10^(-6)/(0.4)^2}*(0.8)^2
=6.12*10^5
(b)はこの6倍と思ったのですがどうなのでしょうか?
未だに電場や電束という概念がよくわかっていないので自信がないです。
752ご冗談でしょう?名無しさん:03/12/06 17:25 ID:msQ4qvjX
>>750
よーーーく考えてみましょうね?
定規を曲げるには、力がいりますよね?とても重力じゃ曲がらないですよね?
その力の大きさによって、曲がり具合って変わりますよね?
だから、一概には言えませんよね?
753ご冗談でしょう?名無しさん:03/12/06 18:55 ID:???
>>750
単純梁の両端に逆方向のモーメントを掛けた場合に相当する。

http://homepage2.nifty.com/Pixy/calchtml/routine/04/iso/0428.html
754ご冗談でしょう?名無しさん:03/12/06 20:46 ID:???
>>746
月の重心は幾何学的中心より2〜3km地球側に寄ったところにあります

>>747
月にのった座標系でみると潮汐力は地球方向とその反対方向に引き伸ばす
向きに働きます。したがって同期したときに重い側が地球に正対するか
正反対方向を向くのが安定な位置になります。もし質量分布の偏りが
なければ、潮汐力による自転と公転の同期は起こりません。したがって、
月がいつも同じ面を地球に向けているのはそっちが重いから、というのは
理由の全てではないにせよ、理由の一部ではあります。少なくとも
>>743のようにnoと言い切るのは明白に間違いです。

755ご冗談でしょう?名無しさん:03/12/06 20:49 ID:???
>>749
いい加減にニュートリノを幽霊粒子と表現するのはやめてほすい
756ご冗談でしょう?名無しさん:03/12/06 20:59 ID:???
>>754
質量分布の偏りがなくても潮汐力によって月がわずかでも変形すれば、
その変形によって消費されたエネルギーによって時点が加速もしくは
減速するのではない?
757ご冗談でしょう?名無しさん:03/12/06 21:04 ID:???
>>755
では何と表現しよう?
758ご冗談でしょう?名無しさん:03/12/06 22:27 ID:???
やかんでお湯を沸かしたときに口から水蒸気が出てその後湯気になって白く見えますが
この水蒸気から湯気に変わったときの運動量の違いが
どのように光との関係に違いをもたらすのか教えてください
759ご冗談でしょう?名無しさん:03/12/06 22:34 ID:???
>>758
湯気は液体。水蒸気は気体。だから屈折率が大きく異なる。
白く見えるのは乱反射。
760ご冗談でしょう?名無しさん:03/12/07 00:03 ID:6vxtFRbq
>>756
???
なんで754にその質問?
761ご冗談でしょう?名無しさん:03/12/07 00:14 ID:???
>>760
いや、ちょっと「もし質量分布の偏りがなければ、潮汐力による自転と公転の
同期は起こりません。」という>>754さんの書き込みに疑問を感じましたので。
762ご冗談でしょう?名無しさん:03/12/07 00:43 ID:???
>>761
変形したら質量分布に偏りができた状態になるでしょう?
月の重心が地球に寄っているというのももちろん潮汐変形を
受けた状態での話ですよ。
「潮汐力が働いた状態で質量分布の偏りがなければ」、と
明記すればよかったですか?潮汐力のない状態での質量分布
などという空想世界での話をしているつもりはなかったもので
わざわざ断る必要を感じませんでした
763ご冗談でしょう?名無しさん:03/12/07 00:47 ID:???
>>762
「潮汐力が働いた状態で質量分布の偏りがなければ」
拙者、てっきりそのような前提で書いておられると受け取っておりました。
ひねくれ者なのでスマソ。
764ご冗談でしょう?名無しさん:03/12/07 01:00 ID:???
タバコの煙は
タバコから出てる煙は青白い
口から出した煙は白い なぜ色が変わるの?
765ご冗談でしょう?名無しさん:03/12/07 01:16 ID:???
>>764
ぐぐって調べればすぐわかるよ。
766ご冗談でしょう?名無しさん:03/12/07 13:54 ID:???
重力と潮汐力はどう違うんですか?
767ご冗談でしょう?名無しさん:03/12/07 14:16 ID:???
>>766
言葉が違えば定義が違い、これはほぼすべての場合に当てはまる。
重力、朝夕力ぐらいぐぐれよ。
768ご冗談でしょう?名無しさん:03/12/07 15:02 ID:???
じゃあ潮汐力と朝夕力も定義が違うのですか?
769ご冗談でしょう?名無しさん:03/12/07 15:04 ID:jk9BuEI5
300kgのバイクが20kmから70kmに加速する時の仕事を求めたんですけど
どうやって考えればいいんですかい?
770ご冗談でしょう?名無しさん:03/12/07 15:22 ID:???
求められたのに考えられないとはいったいどういう状況なのだろうか・・・
いったいどうやって求めたのか、何を考えたいのかさっぱりわからん。
771ご冗談でしょう?名無しさん:03/12/07 15:24 ID:jk9BuEI5
>>770
あぁ・・ すんません。指がいうこときいてませんでした。
「求めたいんですけど」でした
772ご冗談でしょう?名無しさん:03/12/07 15:45 ID:???
70km/hの時の運動エネルギーと20km/hの時の運動エネルギーの差が
加速するのにした仕事とおなじだよ
773ご冗談でしょう?名無しさん:03/12/07 15:49 ID:???
>>768
そういう揚げ足取りは感心しないねぇ。
>>767の朝夕は変換ミスなんだよ。
君は洞察力に欠けているようだな。
774ご冗談でしょう?名無しさん:03/12/07 16:17 ID:Fqg5RHon
細い糸に質量1.0kgの小球Aをつけた振り子の振動の最下点に
質量1.0kgの小球Bをおく。Aを最下点から0.2Mの高さまで
いとがたるまないように持ち上げて静かに放すと、AはBと
衝突した後、一体となって振れた。
〔1〕衝突後の振動の最高点は、最下点から何mの高さか?

                 解き方のヒントお願いします。
あと最近物理習い始めたのですが物理ってどうやって勉強したら
効率いいですか? 

最近
775ご冗談でしょう?名無しさん:03/12/07 16:37 ID:???
>>773
そういう態度は感心しないねぇ。
言葉が違えば定義が違うんだろ?ぷっ
君は社会性に欠けているようだな。

776_:03/12/07 16:38 ID:???
pn接合半導体のエネルギーバンド図で質問です。
なぜ電圧VをかけるとエネルギーバンドがeV分相対的に変化するのですか?
なぜ接合部のみで電圧の降下が生じるのですか?

 教えて下さい。
777ご冗談でしょう?名無しさん:03/12/07 16:39 ID:???
AとBが一体となって振れたということはAとBの間のはねかえり係数が0。
778774:03/12/07 16:42 ID:Fqg5RHon
>>777
ありがとうございます!!
779ご冗談でしょう?名無しさん:03/12/07 16:43 ID:???
>>774
ウンドウリョウホゾンソーク
780ご冗談でしょう?名無しさん:03/12/07 16:51 ID:AmFPM8Tp
http://www.elrosa.com/tisen/70/70232.html

たばこ直接から立ち登る煙はよく見ると青い。それに比べて口から吐き出された煙は白い。
タバコから立ち登る煙は、純粋に葉や巻紙が燃えた後の微粒子。
微粒子は波長の短い青い光を拡散する為に、煙の色が青く見える。
口から吐き出された煙は、体内で微粒子に水蒸気が混ざり大きな粒子に変質する。
大きな粒子はいろいろな光を拡散するので混ざり合って白く見える。
これは雨を含んだ雲が白く見えるのと同じ原理。
781ご冗談でしょう?名無しさん:03/12/07 20:32 ID:ecUt6Vyt
有効状態密度の問題やってて思ったのですが

kg(キログラム)/s(セコンド)がm-3になるのがなんでか解らないんですけど
m-3って1÷体積って事ですよね?
これが何を意味してるのかも解りません。

SI単位系が全部乗ってるようなサイトも幾つか回ったのですが解りませんでした。
知ってる方いらしたら教えて下さい。
782ご冗談でしょう?名無しさん:03/12/07 20:52 ID:mECpnUah
>749
ニュートリノの説明ありがとうございます。
わかったようなわからないようななんかむずかしい。
でもがんばって説明してみます。
783ご冗談でしょう?名無しさん:03/12/07 21:13 ID:???
有効状態密度ってなんの状態密度よ?
MKS単位系って知ってるでしょ。M,K,Sがそれぞれ独立な単位だから
kg/s=m^-3
なんてことは起こりえない。
784ご冗談でしょう?名無しさん:03/12/07 21:42 ID:???
特殊な単位系使ってんじゃないのか。hbar=1になってればhbarの単位が
省略された分そういうことが起きるかも。
785781:03/12/07 22:50 ID:ecUt6Vyt
キャリア濃度でございます。
786ご冗談でしょう?名無しさん:03/12/07 22:57 ID:???
個/m^3
787ご冗談でしょう?名無しさん:03/12/07 23:03 ID:NQ6wA2tj
1msecって、何秒ですか?
788ご冗談でしょう?名無しさん:03/12/07 23:05 ID:???
1×10^(-3)s
789ご冗談でしょう?名無しさん:03/12/07 23:06 ID:9/TwNQ54
AT車で、エンジンがかかっている時、セレクトレバーが、P・N以外の位置にあると動力がつながった状態になって、アクセルペダルを踏まなくても車が動き出します。その現象をなんというでしょう。
790ご冗談でしょう?名無しさん:03/12/07 23:06 ID:???
>>759
ありがとうございました
791ご冗談でしょう?名無しさん:03/12/07 23:54 ID:???
>789
クリープ現象
792ご冗談でしょう?名無しさん:03/12/08 00:00 ID:snrBhs+v
ばね定数k、自然長の長さL、単位長あたりの質量mのばねを伝わる
縦波の速さvはv=√((kL)/m)であることを示せ。

っていう問題なんですが、どこから手を付けたらいいのか分かりません。
運動方程式を立ててみようと思ったけどどうやって立てたらいいのかも分からない・・・
どなたかよろしくお願いします。
793ご冗談でしょう?名無しさん:03/12/08 02:15 ID:5TJWPmbW
質量mの球がなめらかな直線状を速度V0で進んでて、
それとは逆向きにbVという抵抗があるときに、球が止まるまでに進む距離は?
っていう問題なんですけど・・どうしても分からないので教えてください。
よろしくお願いします><
794ご冗談でしょう?名無しさん:03/12/08 02:30 ID:???
>>793
とりあえず運動方程式を書いてみ
795ご冗談でしょう?名無しさん:03/12/08 03:47 ID:???
院試に関してなんですが、
留年した受験生はつらいのでしょうか?

留年してまじめに通うようになってからもっと勉強したいと思いました。

続・平成16年度大学院入試統合スレッド
http://science2.2ch.net/test/read.cgi/sci/1062581992/
で質問しようかと思ったのですが、スレの進行が遅いため、こちらで質問させてください。
796ご冗談でしょう?名無しさん:03/12/08 03:56 ID:???
>>793
DQNの漏れがといてみた
運動方程式は
-bv = m dv/dt
積分すると、
v=v_0 exp[-bt/m]
v=0はt->∞なので、
vを[0,∞]で積分

x=(m*v_0)/b

と出たんだが
賢い香具師添削頼む。
797ご冗談でしょう?名無しさん:03/12/08 04:10 ID:???
>>795
正直知らんが、
熱意(と努力)があれば無問題。
一年世界を放浪した香具師とか社会人から舞戻ってきた香具師とか
他大学出身の香具師とかそれこそ普通に留年した香具師とか一杯知ってるが、
ちゃんと院生になれてるよ。

教授連中も、普通経歴より人を見る(はず)。少なくとも漏れの知ってる限りでは。
後、留年したって事は学部の勉強がぼろぼろになってる可能性が高いので、
とりあえずペーパーテストはきっちり点をとれるようになっとけ。
798ご冗談でしょう?名無しさん:03/12/08 04:25 ID:???
>>797
即レスありがとうございます。
現在私大なのですが、学費の関係で国立への進学を考えております。

現在の大学より少しでも良い研究をしてる大学に行きたいのですが、
「(ウチの大学よりレベルが低い)私大で留年してるんだから、ウチで勉強するのは無理」
と教授に思われないかと少し心配になっていました。

学部の成績はお察しのとおりぼろぼろですが、がんばります。
深夜にありがとうございました。
799ご冗談でしょう?名無しさん:03/12/08 09:07 ID:???
>>796
文句なし
800792:03/12/08 15:51 ID:8VUGrrWu
誰かお願い・・・m(_ _)m
801ご冗談でしょう?名無しさん:03/12/08 17:07 ID:???
>>792
m/Lとはなんだ?
802ご冗談でしょう?名無しさん:03/12/08 17:49 ID:???
>>792
ちょっと別解の部類になりますが、
有限個(n個)の質点の連成振動で考えていくとわかりやすいかもしれません。
この際、質点の質量と、質点間の「ばね定数」が変わりますが、注意してみて下さい。
波動方程式がでれば、答えが出たようなもの。
特殊相対性理論の光についてだが、「誰が見ても光の速さは一定に見える」とあるが、
その光って真空を通る状態のみですか?

水の中でも特殊相対論が成り立つなら、水の中の光は屈折率の関係によって空気より遅くなりますよね。
そうなると、水の中の方が空気より時間が遅く流れるんですかね?
804ご冗談でしょう?名無しさん:03/12/08 19:47 ID:/d63SYMt
灯油類などの熱膨張率を知りたいのですが、どこで調べられるでしょうか?
数時間ググってましたがわかりません。

灯油が1℃温度上昇すると、どれだけ体積が増えるのか?が知りたいです。
宜しくお願い致します。
805ご冗談でしょう?名無しさん:03/12/08 19:50 ID:???
>>804
ベンゼンとかなら理科年表にも載ってるけどな
806ご冗談でしょう?名無しさん:03/12/08 21:16 ID:???
>>803
勿論真空中。
というか、この宇宙で許される最大速度というのがあって、
真空中の光がそれを出しうるというだけの話。
物質中の光がいくら減速されようとも、それは時間の流れや相対論とは「無関係」。
そもそもバケツで有名なチェレンコフ光は、水中の光速度を荷電粒子が追い越した事で起こる現象だし。
807ご冗談でしょう?名無しさん:03/12/08 21:18 ID:WSdkr+aH
>>803
ん〜、まだ相対性理論やってないからよくわかんないスけど、
光の速さが小さくなる=時間の流れが遅くなる
なんですか?
にわか知識だと頭を混乱させますよ。
808ご冗談でしょう?名無しさん:03/12/08 21:19 ID:WSdkr+aH
(つд`)'カブった・・しかも>>806
さんのが詳しい・・_| ̄|○
809ご冗談でしょう?名無しさん:03/12/08 21:19 ID:???
ちょっと訂正。光速より遅い物体の速度の限界、と言い替えておくれ。
さらに細かい事を言うと、物質中で光速が遅くなるのは内部で吸収と再放出を繰り返すから。
810助けて:03/12/08 21:42 ID:AU4Lf9b+
タイヤのころがり抵抗値0.015が記述されている文献知りませんか?
811ご冗談でしょう?名無しさん:03/12/08 21:49 ID:???
任意の車体の任意のタイヤでか?
812助けて:03/12/08 22:03 ID:AU4Lf9b+
アスファルト舗装で普通タイヤです。一般的には0.01〜0.02と記憶していますが、それが記述されている文献知りませんか?
813ご冗談でしょう?名無しさん:03/12/08 22:05 ID:B0VckVyu
>>796 >>799
ありがとうございます!!!!
814ご冗談でしょう?名無しさん :03/12/08 23:39 ID:NLQ072Qi
フーリエ変換がわかりません、、、。いま読んでいる教科書に
「任意の正弦振動をしている量、例えば密度nは
n=n_0exp(k・r-ωt)と書ける。(kは波数、ωは振動数)」
とあるのですが、なぜこうあらわせるのでしょうか?
815ご冗談でしょう?名無しさん:03/12/08 23:51 ID:???
>>814
expの中iが抜けてるよね。
それはオイラーの公式を考えれば分かるかと。
816ご冗談でしょう?名無しさん:03/12/09 00:08 ID:???
>>812
ホンダとかの技術マニュアルに書いてあるんじゃないの?
よく知らないけど。
817ご冗談でしょう?名無しさん:03/12/09 00:09 ID:/OydZrSf
下の矛盾の謎を教えてください。

MSK単位      CGS電磁単位
1V         10^8emu
1Ω         10^9emu

ということは、
1V+1Ω
=1*10^8emu+10*10^8emu
=11*10^8emu
=11V
818ご冗談でしょう?名無しさん:03/12/09 00:14 ID:???
1V+1Ωってなによ?
819ご冗談でしょう?名無しさん:03/12/09 00:18 ID:/OydZrSf
>>818
ほんとは足しちゃいけないよねぇ…それはわかっているんだが、CGS単位系にすると足せてしまう気がして。
820ご冗談でしょう?名無しさん:03/12/09 02:21 ID:???
>>806サンクス
821ご冗談でしょう?名無しさん:03/12/09 03:56 ID:???
>>773
いや、「朝夕力」 ってのがこのスレでそれまで2回も出てきてたから
変換ミスじゃないのかもって思ったんですよ。
ボクのやったことが洞察力に欠けてると思うなら
それこそ洞察力に欠けてるのでは。
822ご冗談でしょう?名無しさん:03/12/09 04:04 ID:???
>>792
略解。
波の進行方向にx軸を取る
x軸方向の変位をuとおく。

弾性力はk du/dxとなるので、
[x,x+凅]部分の運動方程式は線密度をmとおくと、

m凅*d^2*u(t,x)/dt^2 = k du(t,+x凅)/dx - k du(t,x)/dt
凅で割って、凅 -> 0の極限をとれば、

m*d^2u(t,x)/dt^2 = k d^2u(t,x)/dx^2

上記の波動方程式より、
v = √(k/m)

見づらいが勘弁してくれ
あと、uはt,xの関数だからdじゃなく、偏微分の記号を使え


>v=√((kL)/m)
これは単位を計算したらおかしくないか?
823ご冗談でしょう?名無しさん:03/12/09 06:50 ID:???
>>822
ちょっと突っ込み。
弾性力は、k du/dx では次元が合わないのでは。
ばねの全長がLなので、kL du/dx となるはず。そうすれば題意に沿い、
>v=√((kL)/m)
も、速度の次元を持ちます。
824ご冗談でしょう?名無しさん:03/12/09 07:30 ID:wekSfI6h
なんでかな
普段オナラしてもそんなに臭いと思う事はないのに
風呂でボコッ!とするとめちゃめちゃ臭い

なんでかな?
825ご冗談でしょう?名無しさん:03/12/09 07:35 ID:???
>>824
さほど臭く感じないのは、空気中だと拡散してしまうからです。
一方水中でやると、拡散せず泡としてそのまま水上に届くため、めちゃめちゃ臭いと感じるのです。
826ご冗談でしょう?名無しさん:03/12/09 07:45 ID:???
>>824
風呂場で屁をこぐと臭くなるのは何故?
http://ebi.2ch.net/rikei/kako/1013/10130/1013084923.html
827ご冗談でしょう?名無しさん:03/12/09 08:19 ID:tFikwrfP
この宇宙は永遠に無限に広がりつづけます
であるからして中の物質は永遠にうすっぺらくなりつづけます

であるからして人類もうすっぺらくなりつづけないとエネルギーなくなってしんじまいます

ですから屁の臭さも永遠に無限に薄っぺらいにおいになり続けます
お前のおつむもうすっぺらくなりつづけます
828ご冗談でしょう?名無しさん:03/12/09 09:49 ID:???
風呂場でうんこを漏らすと汚なくなるのはなんでだろう
829ご冗談でしょう?名無しさん:03/12/09 10:29 ID:???
>>821
おまへ、2chのレスだけで判断してるの?w
830ご冗談でしょう?名無しさん:03/12/09 11:10 ID:???
くだらない質問なんですけど、無知なので教えてください。
・現在見える一番遠い星は150億光年先だとの事ですよね。
・ビッグバンは∞の速度で広がったけれど、それは空間の
 広がりなので、相対論には矛盾しないんですよね。
150億年前に発した光がいま、地球に届くためには、遠い星も
∞の速度で遠ざかったことにはならないのですか?
 宜しくお願いします。
831830:03/12/09 11:15 ID:???
すいません追加です。
ビッグバンの瞬間に遠い星はいま見える形に完成されていたことに
なってしまう様な気がするのですが、どうなのでしょうか?
832ご冗談でしょう?名無しさん:03/12/09 11:16 ID:???
>>829
悔しいのはわかったから、よそでやれ。
833ご冗談でしょう?名無しさん:03/12/09 11:28 ID:???
>>830
無限の速度(って何?)なんかでは広がっていない。現在主流のインフレーション理論によると
光速以上ではあったようだが、有限の速度で急激に膨張し、その後光速以下の
ゆっくりとした速度に転じた。現在再加速中ではないかという説もある。

また、宇宙が現在のように透明になったのは物質と光の混合状態が解消された
「宇宙の晴れ上がり」(宇宙歴30万年頃)以降で、この頃は既にゆっくりとした膨張になっている。
この後最初の天体が形成されるまで更に時間が必要で、
そうやって作られた天体の光が百数十億年たった今、地球に届いている。

つまり、ビッグバンの瞬間と今見えている星との間には長い時間が流れている。

834名無しです:03/12/09 11:30 ID:GbbZ6PRE
高校生の問題なのですが何故媒体中で光速が遅くなるのかが
分からないので教えてださい。(媒体の中は素粒子で詰まっていると
考えると何も無いところは30万km/sで素粒子に吸収されて放射されるまで
にタイムラグがあるのでしょうか?)
835ご冗談でしょう?名無しさん:03/12/09 11:47 ID:???
吸収放出のメカニズム

1.光子吸収
2.原子のエネルギー準位上昇
3.確率的に光子を放出
4.エネルギー準位戻る。

なので、物質密度が上がれば上がるほど遅くなる。

836ご冗談でしょう?名無しさん:03/12/09 11:55 ID:???
>>835
それは媒質中の光速と関係あるのですか?
837名無しです:03/12/09 12:06 ID:GbbZ6PRE
もし括弧内のが正しいとすると変な事があって
不確定性原理より僞冲≒hから高いエネルギーの光子は吸収されて
放射されるまでの時間が低いものに比べて短いはずなのに
現実には周波数の高い光の方が低い方に比べて光速は遅いそうなんです。
この事も835で書かれた事を使うとうまく説明出来るのでしょうか?
838名無しです:03/12/09 12:08 ID:GbbZ6PRE
>現実には周波数の高い光の方が低い方に比べて同じ媒体中
 では光速は遅いそうなんです
 上の事、間違っていたらごめんなさい
839ご冗談でしょう?名無しさん:03/12/09 13:01 ID:???
高エネルギーの光子は原子をより高励起状態にするから
元のエネルギーの光子を出して基底まで落ちる確率が減るんじゃない?
(別の低い励起状態に寄り道したりして)
840830:03/12/09 13:09 ID:???
>833さん
ご教示ありがとうございました。
841ご冗談でしょう?名無しさん:03/12/09 16:47 ID:???
失念&ググっても妥当に解説されたものが見当たらないので…

万有引力定数G=6.673*10^-11N・m^2/kg^2

はどのようにして求められるのでしょうか?
842ご冗談でしょう?名無しさん:03/12/09 17:09 ID:tq/TGwaC
最初から答えられないとわかっていたら
質問する意味がないな・・・
843ご冗談でしょう?名無しさん:03/12/09 17:17 ID:???
>>841
振り子使えば?
844ご冗談でしょう?名無しさん:03/12/09 19:38 ID:9uhtcc29
地球の磁束密度って何[T]ですか?(日本)
845844:03/12/09 19:55 ID:9uhtcc29
マルチをしてしまったので、ここでは終わります。
846822:03/12/09 21:30 ID:???
>>823
指摘ありがとう
寝ぼけてたみたいだ
847ご冗談でしょう?名無しさん:03/12/09 22:05 ID:/OydZrSf
>>844
 日本への地球による磁束密度は、水平方向に対して30000nTくらいです。
 理科年表に出てます。
848ご冗談でしょう?名無しさん:03/12/09 22:07 ID:cAqRTAG/
球面波の波動方程式の導出↓がわかりません。

d^2(rp)/dr^2 = (1/(c^2))(d^2(rp)/dt^2)

∇(p^2)の∇を球座標系と考えたとき、
∇(p^2) = d^2(rp)/dr^2(上の式の左辺)
となるのは分かったんですが、これ以上が分かりません。
849ご冗談でしょう?名無しさん:03/12/09 22:13 ID:9uhtcc29
>>847
有難うございます。( ̄ー ̄)ニヤリ
850ご冗談でしょう?名無しさん:03/12/09 22:48 ID:???
波動の問題なのですが

u(x,t)=f(x-vt)

で、u(x,0)のときの周期とはどうやったら求められるのでしょうか?
全くわからないので、やり方だけでも教えていただけたら嬉しいです。
よろしくお願いします。
851ご冗談でしょう?名無しさん:03/12/09 22:59 ID:???
>>850
それだけじゃわからない
852850:03/12/09 23:14 ID:???
>>851
すみません。f(z)=Asin2π(z/λ)という式もありました。
853ご冗談でしょう?名無しさん:03/12/10 01:16 ID:???
えと、t=0のときの周期とは?

普通に考えて
u(x,t)=f(x-vt)=Asin2π{(x-vt)/λ}
周期Tは、u(x,t)=u(x,t+T)より
Asin2π{(x-vt)/λ}=Asin2π[{x-v(t+T)}/λ]=Asin2π{(x-vt)/λ-vT/λ}で
T=λ/v
854ご冗談でしょう?名無しさん:03/12/10 03:00 ID:xksmhB6M
宇宙がめちゃ熱いころから、今の−273℃近くまで冷えたってことは、
もしかして数億年単位くらいで、摂氏100〜0度の時期があったんじゃ
ないのかなと思いました。

このころ、宇宙空間に生物が発生したっていう可能性はあるんですかね?
隕石にその痕跡が残ってないものかとか、研究しているところってあります?

面積的に地球の生物圏よりはるかに大きい空間なわけで、
ぜんぜんありえる話だとオモタんですが、間違いでしょうか。
855ご冗談でしょう?名無しさん:03/12/10 04:11 ID:???
別に温度がその位なら生命が発生するというわけではあるまい。
なぜ地球の原初の生命は、海で誕生したと思う?

それとは別に、地球型でない生命の発生については、SFの立派な1ジャンルとなっているな。

856841:03/12/10 08:28 ID:???
>842
観測値でしょうか?此れは決まった数値なのでしょうか?
定説みたいなもの?
>843
万有引力を求めるのではなくて…
857ご冗談でしょう?名無しさん:03/12/10 10:42 ID:???
>>856
物理定数。
万有引力 F∝m1*m2/r^2 の係数が万有引力定数で
観測で求めなければわからない。
858ご冗談でしょう?名無しさん:03/12/10 10:47 ID:???
>>854
物質の密度が低すぎて複雑な分子はほとんど生成しないかと。
859ご冗談でしょう?名無しさん:03/12/10 11:40 ID:nSbl/Myg
>>855
生命誕生って、月がまだ地球に近くて波が激しく、波打ち際の泡の中で
生成とかじゃなかったっけ?
860ご冗談でしょう?名無しさん:03/12/10 12:31 ID:KGHwAwT4
生命誕生説
 隕石説:隕石にタンパク質の形成源があった
 熱水こうしょう説:熱水中の硫化水素H2S
 オパーリン説:環境説→水、熱、光、酸素、炭素、窒素、雷・・・
861ご冗談でしょう?名無しさん:03/12/10 12:44 ID:wk3lxSPJ
おまえら・・・
862ご冗談でしょう?名無しさん:03/12/10 13:27 ID:???
愛してる
863ご冗談でしょう?名無しさん:03/12/10 13:56 ID:HGziq338
>>848おながいします。
訂正

×∇(p^2)
○(∇^2)p
864ご冗談でしょう?名無しさん:03/12/10 14:03 ID:???
顔文字ですか?
865850:03/12/10 16:19 ID:???
>>853
遅れてすみません。
お答えありがとうございますm(__)m
866ご冗談でしょう?名無しさん:03/12/10 16:25 ID:qdkVQmm0
>>856
「万有引力定数 測定方法」等で検索してみればよい
他の物理定数に比べ精密な測定は難しいが、ちゃんと測られているぞ
867ご冗談でしょう?名無しさん:03/12/10 18:33 ID:???
俺も顔文字にしか見えないな
868ご冗談でしょう?名無しさん:03/12/10 20:36 ID:???
>>848
出発点は何?
869848:03/12/10 21:23 ID:HGziq338
>>868
質問の主旨にあっているかわからないのですが、
ダランベールの波動方程式

(∇^2)p = d^2(p)/dx^2 + d^2(p)/dy^2 + d^2(p)/dz^2

というのを習いました。
で、div(grad p) = (1/(c^2))(d^2(p)/dt^2)
が成り立つので(運動方程式、気体の方程式、連続の方程式から)、

(∇^2)p = (1/(c^2))(d^2(p)/dt^2)

となるようです。さらに、球座標系でも似た形(>>848)で
書けることを示せ。という問題です。
870ご冗談でしょう?名無しさん:03/12/10 23:17 ID:FyCCdUTF
こんばんは

ちょっと気になるので、ここの皆さんの知識をお借りしようと思います。
0℃から凍り100℃から沸騰する温度の単位と言える水。

でも、これって厳密な条件がそろったときだけですよね?
大気温で湿度が異常に低いとき、湿度が高いとき
気圧が低いとき、高いとき。
風が流れているとき。

どれくらい違うものなのでしょうか?
まず、0℃で凍る定義から教えてください
871ご冗談でしょう?名無しさん:03/12/10 23:24 ID:79tgnpp4
このスレで代数を使った力学も出来ない人の質問に答えてくれるんですか?
872ご冗談でしょう?名無しさん:03/12/10 23:29 ID:???
>>870
現在は温度目盛りは水の三重点という厳密な条件がそろった状態で
定義されている
873さとみ:03/12/10 23:32 ID:yEM7XtTH
固定軸周りの剛体の回転の運動方程式についておしえてください☆
874さとみ:03/12/10 23:40 ID:yEM7XtTH
固定軸周りの剛体の回転の運動方程式てどんなものかおしえて下さい☆
875ご冗談でしょう?名無しさん:03/12/10 23:42 ID:???
>>874
ボコられる前に逃げた方が良いかもよ。
876ご冗談でしょう?名無しさん:03/12/10 23:53 ID:???
ぐーぐる>>2ch質問スレ
877ご冗談でしょう?名無しさん:03/12/11 00:01 ID:???
>>876
何故このスレが存在しているのか不思議だ。
878ご冗談でしょう?名無しさん:03/12/11 00:05 ID:???
暇だし答えてみる。
>>869
波動方程式を認めるなら話は楽で、pを球対称(∂p/∂θ=∂p/∂φ=0)とすると
連鎖律から

∂p/∂x=(∂p/∂r)*(∂r/∂x)+(∂p/∂θ)*(∂θ/∂x)+(∂p/∂φ)(∂φ/∂x)
 =(∂p/∂r)*(∂r/∂x)
   =x/r(∂p/∂r)

二回微分は

∂^2p/∂x^2=1/r(∂p/∂r)-x^2/r^3(∂p/∂r)+x^2/r^2(∂^2p/∂r^2)

∂^2p/∂y^2、∂^2p/∂z^2も同様にわかるので

∇^2(p)=2/r(∂p/∂r)+(∂^2p/∂r^2)
  =1/r{∂^2(rp)/∂r^2}

これを波動方程式に代入して

1/r{∂^2(rp)/∂r^2}=1/c^2(∂^2p/∂t^2)

両辺r倍して、右辺のrを時間微分の中に入れると

{∂^2(rp)/∂r^2}=1/c^2{∂^2(rp)/∂t^2}

で、rpが波動方程式を満たす。この解をf(r-ct)、g(r+ct)とすると

p=1/r{f(r-ct)+g(r+ct)}

となり、波面が球面で振幅が1/rで減少する波になる(球面波)。
879ご冗談でしょう?名無しさん:03/12/11 00:15 ID:???
なるほど、暇つぶしの為にこのスレは存在しているのか
880ご冗談でしょう?名無しさん:03/12/11 00:38 ID:???
何をいまさら
881ご冗談でしょう?名無しさん:03/12/11 00:43 ID:???
できるだけ答えを丸教えにしないようにしてる
自分で調べてわかることなら自分で調べる。
あくまで勉強の手助け、まあ内容にもよるが。
というのがおれが思ってるこのすれの存在。
>>874などは自分で調べてどこがわからないのかなど書けば誘導できる。
ナイスコンセプトなすれだと思う
882854:03/12/11 00:48 ID:FKXQT1gm
密度が低すぎるか・・。なるほど。

もしかして、生命の起源は原始宇宙で、それのタネが現在の地球で
花開いてるとしたら、同じような生命体が別の惑星系でも
発生している可能性があるかなぁと思ったんですが。

私、電波だったみたいですね。
すみませんでした。
883ご冗談でしょう?名無しさん:03/12/11 00:59 ID:???
暇つぶしだろ。
でなきゃナイスコンセプトなスレでイエスかノーで答えられる
質問しただけで袋叩きにあったのか?
884ご冗談でしょう?名無しさん:03/12/11 01:14 ID:???
>>883
どの質問のこと?どーせぐぐればすぐ分かるようなことなんじゃないの?
885ご冗談でしょう?名無しさん:03/12/11 01:17 ID:???
>>882
原始宇宙に複雑な生命を誕生させるには少々問題がある。
硫黄や鉄、マグネシウムなど重い原子がないと効率の良い
酵素ができない。また、岩石や金属でできた惑星ができないだろう。

それらは重い恒星の核融合で作られる。それらの恒星が核融合で作った
元素を放出しないと、生物の身体を作ったりするのは難しい。
原始宇宙には水素とヘリウムしかないと言う。

あと質問ならSF板の方がマシだぞ。
886ご冗談でしょう?名無しさん:03/12/11 01:18 ID:???
>>884
それで分かりゃ聞かないよ。

答える気もないのに反応すんな。
887ご冗談でしょう?名無しさん:03/12/11 01:28 ID:???
>>884
悪かったな。

>反応すんな
とか書いて。このスレが暇つぶし目的だもんな。スレ主でも
ないのに余計な事書いた。俺なんかは「疑問をかいてね」と
書いてあるからつい、>>885みたいな真面目な回答書いちゃった。
888ご冗談でしょう?名無しさん:03/12/11 01:48 ID:???
人間相手に質問してる事を忘れるなって事だ。
宿題丸投げなんか、遊んでくれと宣言してると同じ。
やる気があって、しっかり考えている事がわかればいくらでも手助けして貰える。
逆なら結果も真逆。

基本的に「知ってて」このスレを覗いてる香具師は「答えたい」タイプなんだから、
その辺の暗黙の了解を理解してから質問入れれば、まず普通に答が返ってくるはずだ。
889ご冗談でしょう?名無しさん:03/12/11 01:49 ID:???
とっくの昔に勉強して分かりきってる事だがその時と同じ質問をしてやる。
どう答える?

>1sの質量に相当するエネルギーは9*10^16で合ってますか?

だ。
890ご冗談でしょう?名無しさん:03/12/11 01:54 ID:???
>1sの質量に相当するエネルギーは9*10^16ジュールで合ってますか?
だった。
891ご冗談でしょう?名無しさん:03/12/11 02:06 ID:???
つまり答えないというわけだ。暇つぶしにならない問いになんか
答えないだけだろ。善人面してないで
>>880>>878みたいに正直に答えろよ。

俺は俺みたいにこの板を信じて質問してくる人が袋叩きに合わないように
って仏心で質問者の誘導をしてんだよ。
892ご冗談でしょう?名無しさん:03/12/11 02:10 ID:???
おれは884ではないがいきなり聞かれたら答えに困る質問だな。
E=mc^2を疑ってるのか、次元の心配をしているのか、
それとも算数を確認してるのか分からん。

特に算数と解釈されたら叩かれるのは無理も無いな。
893ご冗談でしょう?名無しさん:03/12/11 02:20 ID:???
ほう、質問によっちゃ袋叩きにあうのか。
他の板じゃ根本的な勘違いをしてても親切に教えてくれたよ。
E=mc^2は独学だがな。

単位について混乱してたんだがそれでも袋叩きにあうんだろうな。
894ご冗談でしょう?名無しさん:03/12/11 02:25 ID:???
単純に真面目に質問するだけで住人の気分で
暇つぶしに袋叩きに合ったり、トンチンカンな
答え返したり気が向けば答えることもあるスレって事で
OKだな。
895ご冗談でしょう?名無しさん:03/12/11 02:31 ID:???
算数と解釈されたら「さっさと電卓でも叩けよ」という話になる。
居合わせた奴も厨だったんだろうが、聞き方も悪い。
896ご冗談でしょう?名無しさん:03/12/11 02:39 ID:???
>>894
何でそんなに顔真っ赤にしてるかわからんが、
ここだって2chの一部なんだからさ、
嘘吐かれたり煽られたりしたっておかしくないでしょ。
んで、君が(たぶん)3時間張りついてる間にも人は刻々と入れ替わってるし、
タイミング悪ければ普通にスルーされたり、煽られたりするだろう。

つうか煽り耐性の無い香具師は、2chに来ない方がいいと思う。寿命が縮むよ。
897ご冗談でしょう?名無しさん:03/12/11 02:49 ID:???
>>895
じゃあどう聞けば良いんだ?

他の板で常識ともいえる質問しちゃった事合ったけど丁寧で
親切に敬語で解説してくれたよ。後で「ググるのを忘れてましたって」
誤ったけどな。

>んで、君が(たぶん)3時間張りついてる間にも人は刻々と入れ替わってるし、
それだけ多くの人がこのスレの実態を知ってくれるわけだ。
898ご冗談でしょう?名無しさん:03/12/11 02:56 ID:???
式も使う定数も分かってる算数の結果を聞くのはただの怠慢だろ。
単位が分からなかったんなら、そうはっきり書けばいいじゃん。
899ご冗談でしょう?名無しさん:03/12/11 03:16 ID:???
笑わすな。ここでの他の質問の行く末も見てたが
まともに答えなんか返って来やしねえじゃん。

>>870の反応がないな。
>>872の答えなんか
俺なら「水の三重点」でググれば良いと言うのが分かるが、
物理の完全に素人の人が分かるかね。せめて
>水の三重点でググれ
って書けば分かるだろうが。

こっちでトンチンカンな答えしか書いてなかったので
別の板で俺が分かりやすく説明してやった事もあった。

>ジュールの単位は s*(m/s)^2だそうですがそうすると
>1sに相当するエネルギーは9*10^16ジュールで合ってますか

と聞こうと思ったが、努力の後を見せろみたいな事が書いてあるから
単純な文章にしたんだがな。怠慢か!
900ご冗談でしょう?名無しさん:03/12/11 03:36 ID:???
教科書を読んだ記憶しかない奴には分からないだろうが、
単位は実際計算して使いこなさなきゃ意味がない。
俺みたいに独学の人間には一度混乱すると不安になる。

単位の情報が混乱してたから一番単純な変換式
E=mc^2の算数の結果を照らし合わせようとした訳だが、
どこを探しても見つからない。1円玉で全国の風呂を
沸かせるってどれだけの熱量だよ。それで専門家が
集まってるここを頼ってきた訳だが
それだけで袋叩きに合うんだからなここは。
901ご冗談でしょう?名無しさん:03/12/11 03:52 ID:???
>>899
よほどヴァカじゃないかぎり、>>872を見りゃ「水の三重点」がキーワードだ、
くらいわかるだろ。おまいは素人を擁護するフリして実は素人を見下してる
態度をとってることに気付け
902ご冗談でしょう?名無しさん:03/12/11 04:10 ID:???
>>901
ずいぶん、マシな事を言うようになったな。
「言ってみるもんだ」
とよく言われるが喚いてりゃ玄人なら結局理詰めするわけだ。
あの時にそうしてくれりゃもっとラクが出来たのにな。素人に
解説する姿勢ってのが伺えるよ。暇つぶしってのがな。

世代の差かね、一年くらいの間なのに。
一年前は小学生みたいな質問や電波な質問にも
親切丁寧に教えてくれたよ。一緒に答えられるものは
答えたし、困ったときは質問したもんだ。
903ご冗談でしょう?名無しさん:03/12/11 04:35 ID:???
この人は未来技術板の質問スレで物理板が役に立たないことに
したくて(そんなものは本人次第なのにね)暴れてるバカです。
反論されて我慢できずにここまでいちゃもんつけに来たようです。
ほどほどに相手しておきましょう。
904ご冗談でしょう?名無しさん:03/12/11 04:38 ID:???
わからん。君の言いたい事がよくわからん。
古参ぶりたいのかい?
素人ぶりたいのかい?

どうして欲しいの?

>世代の差かね、一年くらいの間なのに。
だから何度も言ってるが、ここは人間の集合体であって(ry
fj世代だって多数流れ込んでいるっつーの…

905ご冗談でしょう?名無しさん:03/12/11 04:38 ID:???
>>903
今ごろ分かったか。

>役に立たないことにしたくて
事実役に立たないだろう。

いちゃもん付けに来たんじゃない。仕返しにきたのだよ。
906904:03/12/11 04:39 ID:???
>>903
ああ、未来技術板でしたか。なるほど…
907ご冗談でしょう?名無しさん:03/12/11 07:55 ID:???
自分の思ったとおりの反応でないと満足できないお子ちゃまが
紛れ込んでいるようだな
908ご冗談でしょう?名無しさん:03/12/11 07:57 ID:???
つうか仕返しって…
909ご冗談でしょう?名無しさん:03/12/11 08:09 ID:???
あとは未来技術板のほうでよろしく…
つきあってらんない
910ご冗談でしょう?名無しさん:03/12/11 09:17 ID:???
>仕返しにきたのだよ
江戸のカタキを長崎で討つってやつでつか?
カッコワルッ
911ご冗談でしょう?名無しさん:03/12/11 10:40 ID:???
未来技術板か・・・。最初に言ってくれりゃあよかったのに。
そんな仕返しなんて季節もの
まぁいろんな人がいるさ、答えてくれる人も答えてくれない人も。
ここだって大学教授、研究者、院生、学部生、工房、厨房、ヒッキーetc
いろんな人がいるんだ。話そうとすればそれなりの話ができるはず。
912ご冗談でしょう?名無しさん:03/12/11 12:03 ID:???
今更ながら

>>869
(∇^2)p = d^2(p)/dx^2 + d^2(p)/dy^2 + d^2(p)/dz^2
↑これはただの微分演算子。あと偏微分の記号使いましょう。
div(grad p) = (∇^2)p = (1/(c^2))(d^2(p)/dt^2)
↑こっちが波動方程式ね。
913ご冗談でしょう?名無しさん:03/12/11 15:53 ID:???
>>912
唐突な展開にワラタ
914ご冗談でしょう?名無しさん:03/12/11 22:02 ID:???
(1)半径aの球内に負電荷-Qが一様に分布し、その中心に正の点電荷Qが
あるとき、球内外の電場を、球の中心からの距離rの関数で表せ。

(2)平面電荷(面密度σ>0)と平行に、面密度-σの平面電荷を
距離Lだけ隔てて置かれたときの電場を求めよ。

(1)は電荷密度を考えてればいいのでしょうが、
(2)は電場の合成を考えなきゃいけないのでしょうが、
この場合はガウスの法則をどう使えばいいのだろう・・・
915ご冗談でしょう?名無しさん:03/12/11 22:19 ID:???
1も2も電荷分布の対称性から電場の取りうる成分を
ちゃっちゃと見極めてしまえばあとは至極簡単な問題
916ご冗談でしょう?名無しさん:03/12/11 23:18 ID:???
ガウスの法則から電場の法線成分には閉曲面内部の電荷しか寄与しない。
あとは対称性を考えて。
1)点電荷を中心とする球面を考えて、その内部にある電荷を考えましょう。
2)一枚の面電荷を囲む長方形、両方の面電荷を囲む長方形を考えましょう。
 (ほんとは無限に広いんだろうけど)
917ご冗談でしょう?名無しさん:03/12/12 00:35 ID:???
>>916
2)は前者と後者、両方考えなきゃダメですか?
それとも、片方ずつ考えて、最後にかさね合わせればいいのでしょうか?
918ご冗談でしょう?名無しさん:03/12/12 00:53 ID:???
前者を考えると、それぞれの面電荷が作る電場がわかる。
後者を考えると、領域内での全電荷が0なのでトータルで電場が出来ない場所がわかる。

ま、重ね合わせてもいいけど。
919ご冗談でしょう?名無しさん:03/12/12 01:25 ID:jq5Vi7NZ
どうして時間というものは止まったり、戻ったりしないんですか?
時間ってどういうものなんですか?
920ご冗談でしょう?名無しさん:03/12/12 01:31 ID:???
>>919
思考がエントロピー増大の方向に進行するから。
数学上は時間は後戻りもできる。
空間が収縮に向かうときに時間は後戻りするのかどうかは不明。

と、学者は言っているのだと思う。
921ご冗談でしょう?名無しさん:03/12/12 01:36 ID:???
質問です。

宇宙はビッグバンによって拡がったということですが
ビッグバンによって「空間」が育成されたのか、
それとも元からある空間に「物質」が拡がっただけなのか、
どちらでしょうか?

また、重力によって宇宙が収縮に向かうとした場合、
「物質」だけでなく「空間」も重力によって縮まるのでしょうか?
922919:03/12/12 01:37 ID:jq5Vi7NZ
うーん、エントロピーで説明できるんですか・・。
そもそも時間ってどういう概念なんでしょうか?
自分は化学屋ですから全く関係はないのですが・・。
物理屋さんに聞く質問ではないのでしょうか?
923ご冗談でしょう?名無しさん:03/12/12 01:43 ID:???
とりあえず止まったり戻ったりは今のところ観測されてないでしょ?
「時間とは何か?」に明確な答えがあるかどうかはオレは知らないし、
「ちょっとした」どころのテーマではないと思う。

スレ一覧とか探せば専用のスレがあるんじゃないかな。
924ご冗談でしょう?名無しさん:03/12/12 02:01 ID:???
>>921
現在の理論では宇宙空間は「無」の揺らぎから生まれ、急激な膨張
(インフレーション)を起こし、その時開放されたエネルギーでできた超高温宇宙が
ビッグバンということらしいです。

収縮するときは空間も縮まるんでしょうが、最終的には特異点(密度∞)に
なるんで現在の物理ではわからないかと。
925ご冗談でしょう?名無しさん:03/12/12 02:19 ID:???
計算物理っぽいことなんですが、こちらでよろしいでしょうか?
世に一般固有値問題
Ax = sBx (A,B.....既知行列 x......未知ベクトル s.....未知スカラー)
を解くプログラムっていうのは沢山ありますが、定数項みたいのが加わった
Ax = sBx + q (q......既知ベクトル)
を解くものっていうのは存在するんでしょうか?
LAPACKにはなかったんで、こちらに聞きに参った次第です。
御存知の方、情報頂けないでしょうか?
926ご冗談でしょう?名無しさん:03/12/12 04:17 ID:???
>>924
>収縮するときは空間も縮まるんでしょうが、

ということは、「空間」も重力の作用(引力)を受けるのですか?
927わかんないんです(><):03/12/12 07:01 ID:???
わかんないんです(><)
928ご冗談でしょう?名無しさん:03/12/12 07:10 ID:???
>>925
LAPACKにないとなりますと、
調べてないので、あるかどうかはわかりませんが、Numerical RecipesやBLASは探してみましたか?

多分なさそうだけど
929ご冗談でしょう?名無しさん:03/12/12 12:33 ID:???
>>926
アインシュタインというじいさんによれば、時空は重力で歪むそうですよ。
930ご冗談でしょう?名無しさん:03/12/12 12:38 ID:???
>>919
>>920
CP対称性は破れているので、CPT対称性が保たれる宇宙ではT対称性も破れています。
(T対称性の直接観測結果はまだだっけ?)
すなわち、時間の矢はエントロピー以外でも方向性を持つという事がわかっています。
>>922
哲学的な疑問には答えられないまでも、現代物理は「時間」というものについて
かなり切り込んでいますよ。もっとも、あくまで物体の運動に付随するパラメータとして、ですが。
931ご冗談でしょう?名無しさん:03/12/12 13:10 ID:???
T-violation観測されたみたいだよ。
ttp://www-ps.kek.jp/jhf-np/CPT/
932ご冗談でしょう?名無しさん:03/12/12 13:14 ID:???
thx.
CPLEARの成果だったのか。
933ご冗談でしょう?名無しさん:03/12/12 13:20 ID:???
>>929
空間が歪むのは知っています。
知りたいのは宇宙が収縮するときに空間の広がりも縮むのか、
あるいは、空間を残して物質だけが宇宙の中心に引き付けられていくのかということです。
934ご冗談でしょう?名無しさん:03/12/12 13:26 ID:???
宇宙というのは空間そのものだ
935ご冗談でしょう?名無しさん:03/12/12 13:42 ID:???
>>925
q=Cxと書いてやればいいだろ。
Cは対角行列でC_ii=q_i/x_i。
Ax = sBx + q = ( sB + C )x = s( B + 1/s C )x
936ご冗談でしょう?名無しさん:03/12/12 14:40 ID:???
>>935
だとしますと、Cは未知となりますが・・・
937ご冗談でしょう?名無しさん:03/12/12 15:32 ID:???
>>936
ウゲ
スマソ
938ご冗談でしょう?名無しさん:03/12/12 16:44 ID:tKJNWLUR
Cq=0なるCを左から作用させて
CAx = CsBx → A'x = s'Bx
とするとかできないんですかね?
939ご冗談でしょう?名無しさん:03/12/12 16:48 ID:???
A'x = sB'xか。
940925:03/12/12 18:16 ID:???
みなさま、お返事ありがとう。
>>938
なるほど。手動でCを演算ですか。
でもこれだと、仮にAやBが対称・疎・帯行列とかだったりしたら
その性質が壊れて計算的に効率落ちてマズーとかならないでしょうか?
そのあたりは仕方がないかもしれないですが・・・
941ご冗談でしょう?名無しさん:03/12/12 20:06 ID:???
ごめん938はうそ
942ご冗談でしょう?名無しさん:03/12/12 20:56 ID:???
うそつきはうそつきの始まり
943ご冗談でしょう?名無しさん:03/12/12 22:51 ID:wj7zJ4Ns
外側の半径がa、内側の半径が0.8aの中空の金属球がある。
小さな穴を通して球内部の空洞の中心に電荷Qを入れた。
(1)球の内側の面(2)球の外側の面の単位あたりの電荷を求めよ。
って問題なんですが、(1)では答えが-Q/2.56πaa
(2)ではQ/4πaaとなるのですが、なんで、電荷が内側だと負になっちゃうんですか?
944ご冗談でしょう?名無しさん:03/12/12 23:10 ID:???
中心の正電荷で内側に負電荷が誘導されるから。
そして元々導体は電気的に中性だから内側に誘導された分を
打ち消す正電荷が外側に生じる。
945ご冗談でしょう?名無しさん:03/12/13 00:20 ID:phXVxeCt
>>872
ググってみました。
>>899
失礼しました。調べてみます。
946ご冗談でしょう?名無しさん:03/12/13 09:32 ID:???
>>945
899はバカなのであいてしなくてもいいですよ
947ご冗談でしょう?名無しさん:03/12/13 18:21 ID:???
1: クルーグマンって何者?2 (499) 2: ●売国奴●円売り介入しまくり財務厨房● (577) 3: 円高?円安?Part359 (781)
4: 【経済から政治を語るスレ】その11 (157) 5: 【経済から軍事を語るスレ】その1 (179) 6: 今後のノーベル経済学賞を予想する (785)
7: 【西村和雄を】経済学検定試験の進退【救え】 (101) 8: 2003年、平成大恐慌! part4 (856) 
9: 政府が「日本は生き残れない悪寒」 (164) 10: ダイマン (3) 11: 円高? 円安? Part 316 (76)
12: インタゲは日本を滅ぼす (701) 13: もの凄い勢いで誰かが質問に答えるスレ (356)
14: 【プレステとは】 ゲーム理論 【関係ない】 (22) 15: なんで共産主義にしないの

経済板のスレの内容ですが、ここと比べると物理板って幼稚に感じてしまうのですが。。
みなさん、どう思います?
948ご冗談でしょう?名無しさん:03/12/13 18:23 ID:???
age
949ご冗談でしょう?名無しさん:03/12/13 18:24 ID:???
>>947
どこをどう比較したのかわからないけど、どっちも幼稚じゃんw
950ご冗談でしょう?名無しさん:03/12/13 18:25 ID:???
>>949
納得w
951ご冗談でしょう?名無しさん:03/12/13 22:21 ID:XDaxOl+U
質問なんですが無知なためなんとか以下のことを教えてください。
 
投げられた物体の運動が継続することについて、古代ギリシャから17世紀の
ガリレオ・ガリレイまでどういった理論が唱えられていたか。

上記の問題なんですが参考書をいろいろ見てはみたんですが、
分らなくて困っているんです。
なにとぞよろしくお願いします。
952ご冗談でしょう?名無しさん:03/12/13 22:36 ID:???
動くのは力が加わり続けるからだという解釈がガリレオ以前。
力が加わらなくても動き続ける(こともある)というのがガリレオ以後。

この発想の転換はまさに奇跡としか言いようが無い。
953ご冗談でしょう?名無しさん:03/12/13 22:45 ID:XDaxOl+U
うん?どういうことですか?もう少し詳しくお願いできませんか?
そのへんが書かれている参考文献はありますか?
954ご冗談でしょう?名無しさん:03/12/13 23:32 ID:???
広末涼子って妊娠してるの!?
ということは・・・
広末のアソコ見た奴がいるってこと?
しかも生で入れて中出ししたってこと?信じられない許せない!
955ご冗談でしょう?名無しさん:03/12/13 23:48 ID:+cru0PQU
>>953
クーンの本にそのあたりのこと書いてなかったかな
『科学革命の構造』でもぱらぱらめくってみれば?
956ご冗談でしょう?名無しさん:03/12/13 23:49 ID:???
10ヶ月もしたら他人にも見られることだろうよ。
957ご冗談でしょう?名無しさん:03/12/13 23:59 ID:???
>>953
インペトゥスでググってみれ
958ご冗談でしょう?名無しさん:03/12/14 01:38 ID:VctQrlYx
∇×F=0をわかりやすく説明してくれ!!!
959ご冗談でしょう?名無しさん:03/12/14 12:31 ID:/+erZWlS
人間って地球上にいるときは空気の浮力を受けてるんですよね?
ということはたとえば体重計に乗って50kgと出ても、
自分の体重は50kgよりも重いってことですか?
960ご冗談でしょう?名無しさん:03/12/14 12:33 ID:uE5AqAgf
>>958
回転なし。
961ご冗談でしょう?名無しさん:03/12/14 12:38 ID:TYNldqen
>>959
体重計も浮力を受けてる
962ご冗談でしょう?名無しさん:03/12/14 12:39 ID:???
>>959
体重計の中の人が浮力分差し引いて表示してくれるから気にしないでいいよ
963ご冗談でしょう?名無しさん:03/12/14 12:46 ID:???
>>961
ワロタ
964ご冗談でしょう?名無しさん:03/12/14 12:49 ID:???
Fは保存力だよな?
保存力ならポテンシャルΦを使ってF=-∇Φと書ける。
で、∇×F=-∇×∇Φ=0 (∵∇×∇は恒等的に0)
965ご冗談でしょう?名無しさん:03/12/14 12:57 ID:???
>>964
むしろ∇×F=0であるような力Fを保存力と呼ぶのだが
966ご冗談でしょう?名無しさん:03/12/14 12:59 ID:???
>>962
体重計の人には上の人の体積なんてわかんないじゃん!
967ご冗談でしょう?名無しさん:03/12/14 13:01 ID:???
それもそうだな。
∇×F=0から恒等式でこれを満たせる形としてF=-∇Φで表されるんだな。

スマソ、顔洗って吊ってくる。
968ご冗談でしょう?名無しさん:03/12/14 14:23 ID:???
>>959
 浮力が小さいから無視してるだけだろ。人間だったら大体10gf位だし。
969ご冗談でしょう?名無しさん:03/12/14 16:05 ID:???
>>959
大体、それは中学校で習うだろよ。
体重計で計った値はkg重、気圧に関係なく重さが量れる。
天秤で計った場合はkgでこれは気圧による浮力を考慮しなくてはならない。

勉強しなおせ。
970ご冗談でしょう?名無しさん:03/12/14 16:10 ID:10Eeczit
>>969
ならたとえば海中で体重計で体重を量った場合は?同じ50kgが出るのですか?
実験しろというのは勘弁してください。
971ご冗談でしょう?名無しさん:03/12/14 16:28 ID:???
>>969
????????
972ご冗談でしょう?名無しさん:03/12/14 16:58 ID:9Y3DiUmC
>>971
969の言いたいのはおそらく、浮力で重力加速度gが補正されたようになっていて
補正されたgでのmgをはかるんだから問題ない・・・ってかいてて思ったが逆ジャン。
gの浮力依存がバネの縮みなりなんなりに線形にでるからそれを補正しなくてはならない。
ただ、空気と人間の(平均)比重を比べればそんなことはきにせんでいい。
973ご冗談でしょう?名無しさん:03/12/14 17:41 ID:???
重量の定義には、浮力の寄与も含まれるのか?ってのが問題では。
http://nippon.zaidan.info/seikabutsu/1996/00269/contents/004.htm
によると

>重量とは、正確には、
>その物体に働く重力の大きさで、即ち、「重量は、物体の質量に9.80665m/sec2の加速度を加える力である。」と定義されている。
974ご冗談でしょう?名無しさん:03/12/14 17:42 ID:2CT7Meqh
しょうもない事かもしれないですけど、人間が音を聞くことができるのは、
鼓膜が音の波を受けて振動するからですよね?
という事は、鼓膜は音の波を受けて共振していると言うことですか?
共振するということは、鼓膜は固有振動数が常に変化しているという事ですよね?

ググッたんですけどいまいち解りません。
どなたか助けて下さい〜。
975ご冗談でしょう?名無しさん:03/12/14 17:54 ID:???
 別に狂信してるわけじゃないと思うが。つーか共振するような振動数が
可聴範囲にあったらほとんど役に立たないぞ。
 スピーカとかでかんがえてみぃ。
976ご冗談でしょう?名無しさん:03/12/14 17:54 ID:???
>969
kg重は力の単位だアホ
977974:03/12/14 18:05 ID:2CT7Meqh
>>975
確かにそうですね。
マイクとかで音を拾えるのは、別にわざわざ固有振動数を変えているわけではないですものね。
ちょっと自分の考えが足りなかったですかね。
何はともあれ、ありがとうございました。
978ご冗談でしょう?名無しさん:03/12/14 18:36 ID:YtMuK2dT
本に、直交して置けって書いてあったのですが、電流が流れているそばに、平行して電流を流すと、お互いの電流の
大きさなどに影響が出るのでしょうか?
979ご冗談でしょう?名無しさん:03/12/14 20:14 ID:???
同じ方向に電流が流れる二本の導線は引き合い、逆方向に流れる導線は反発しあう。
高校の物理の教科書に書いてある。
980ご冗談でしょう?名無しさん:03/12/14 20:23 ID:???
物が切れるのはどういう仕組みで切れているのですか
981ご冗談でしょう?名無しさん:03/12/14 20:41 ID:???
例えば紙をハサミで切る時は、ハサミの構造上、紙の一点にせん断応力という物凄い力がかかるように
なっていて、紙を紙として維持している水素結合の力にせん断応力が打ち勝ったときにどんどん水素結合が
切れていって紙が二つに分かれるというマクロな現象として観測されている。

こんな感じで、実はかなり複雑な物理現象がからんでモノが切れているわけです。
982980:03/12/14 20:53 ID:???
>>981
何となく分かった
ありがとう
983ご冗談でしょう?名無しさん:03/12/14 20:54 ID:kAultYjn
ニュース速報板の進行が速すぎるのでここに書かせてもらうけどさ

フセインが捕まったとしたら、フセイン解放を要求して
余計、テロが烈しくなるんじゃないか?
984ご冗談でしょう?名無しさん:03/12/14 21:59 ID:???
>>983
板違い
985978:03/12/14 22:25 ID:YtMuK2dT
>>979 はい、それは分かってますが、それ以外に電流を平行に
流すことによって起こる現象というのはあるのでしょうか?
986ご冗談でしょう?名無しさん:03/12/14 22:28 ID:mmG5C6fj
>>985
トランスを考えてみましょう。
987ご冗談でしょう?名無しさん:03/12/14 22:33 ID:10Eeczit
>>969に勉強し直せと言われた私は一体・・・

もう1つ質問です。
マイクの音量を上げすぎるとハウリングが起きますよね。
あれって何なんですか?
988ご冗談でしょう?名無しさん:03/12/14 22:35 ID:???
マイク→スピーカ→マイク→スピーカ→ヽ(`Д´)ノ ウワァァァ---ン!!
989ご冗談でしょう?名無しさん:03/12/14 22:37 ID:mmG5C6fj
>>988
イイ!!ヽ(`Д´)ノ ウワァァァ---ン!!
990ご冗談でしょう?名無しさん:03/12/14 22:42 ID:???
そろそろ次スレだね。
991アフォ厨房:03/12/14 23:10 ID:???
絶縁体と不導体と誘電体は同じものでしか?
992ご冗談でしょう?名無しさん:03/12/14 23:17 ID:???
>>991
違います。

↓次どうぞ。
993978:03/12/14 23:23 ID:YtMuK2dT
>>986 トランスなんて知らないんで、検索してみたのですが
よく分かりません。
平行にすることによって、電圧値が変わるとかないですかな?
994ご冗談でしょう?名無しさん:03/12/14 23:33 ID:mmG5C6fj
ある。ただし時間的な磁場変化(電流変化)がないとだめ。
それを使ったのが変圧器(トランス)。
995ご冗談でしょう?名無しさん:03/12/14 23:53 ID:YtMuK2dT
>>994 そうですか。
   時間的変化は無いので、力が働く事によっておこるこで考えて行きます。
   ありがとうございました!
996996:03/12/15 01:32 ID:???
996
997997:03/12/15 01:33 ID:???
997
998998:03/12/15 01:33 ID:???
998
999999:03/12/15 01:34 ID:???
999!?
1000ご冗談でしょう?名無しさん:03/12/15 01:35 ID:4UJqLUjr
せん
10011001
このスレッドは1000を超えました。
もう書けないので、新しいスレッドを立ててくださいです。。。